Internal Medicine Cases

Ace your homework & exams now with Quizwiz!

A 52-year-old female comes to the clinic for a health maintenance visit. Past medical history is significant for hypertension, hyperlipidemia, and diabetes. Her body mass index (BMI) from three months ago was 31.5 kg/m2. For the past three months, she has restricted calories and increased her aerobic activity. Current medications are lisinopril, aspirin, atorvastatin, and metformin. Her pulse is 76 beats/min, blood pressure is 138/76 mmHg, and BMI is 31 kg/m2. The physical exam is normal. Her hemoglobin A1c (HgbA1c) is 7.6%. The patient is frustrated, as her weight has not improved significantly. Involvement of which of the following individuals would be most appropriate in the care of this patient? A. Cardiologist B. Endocrinologist C. Physical therapist D. Registered nutritionist E. Social worker

The correct answer is D. This patient's BMI is still in the obese range, despite her attempts at lifestyle modification on her own. A multidisciplinary approach has been shown to improve glucose control and weight and decrease the risks of further morbidity. Registered nutritionists can counsel patients on daily food choices and portion sizes that fit their lifestyle. Referral to a cardiologist is not necessary at this time as the patient has appropriate blood pressure control and there is no evidence from her history or exam of cardiovascular abnormalities. Endocrinology referral would be appropriate if the patient was having recurrent episodes of hypoglycemia or if her A1C was 8% or more for more than six months, despite intensive treatment; for initiation of a complex multiple daily injection insulin regimen; or for initiation of continuous infusion insulin therapy (i.e. insulin pump). None of these are present in this patient. Physical therapy referral would not be appropriate as she does not have an injury or other indication for referral. Social worker referral is not indicated, as there is no evidence that the patient is uninsured or requires assistance in getting her medications.

A 20-year old male presents to the internal medicine clinic with concerns of right knee pain and an inability to bear weight. He states that he was practicing speed drills (accelerating, decelerating, and changing directions), when he heard a "pop" in his right knee followed by immediate right knee instability and swelling. What is the MOST likely diagnosis? A. Anterior cruciate ligament (ACL) tear B. Lateral collateral ligament (LCL) tear C. Medial collateral ligament (MCL) tear D. Patellar subluxation E. Posterior cruciate ligament (PCL) tear

The correct answer is A. ACL tears commonly occur when a person rapidly decelerates and pivots OR after direct trauma with knee hyperextension. People commonly complain of knee instability and associated knee swelling. An exam will be pertinent for a positive "anterior drawer" sign and "Lachman" test. A lateral collateral ligament tear is secondary to a laterally directed force on the knee and commonly presents with lateral knee pain and joint line tenderness. An exam will be pertinent for pain with "varus" stress A medial collateral ligament tear is secondary to a medially directed force on the knee and commonly presents with medial knee pain and joint line tenderness, swelling, and instability. An exam will be pertinent for pain with "valgus" stress. Patellar dislocation or subluxation can also occur in the setting of acute, non-contact injury as seen in pivoting or "cutting" directions which causes the patella to become displaced, usually laterally. Upon observation, there may be lateral asymmetry of the effusion and the person may resist extending the leg. Upon palpation, the patella may felt displaced from the trochlear groove Posterior cruciate ligament tears are commonly due to posteriorly directed forces on a flexed knee. Examples include: during a motor vehicle accident when the knee is flexed and strikes a dashboard or when a football player is tackled with a flexed knee. An exam will be pertinent for a positive "posterior drawer" sign

A 35-year-old female presents with urgency, frequency, and pain when she urinates. Other associated symptoms include left-lower-back pain, fever, chills, nausea, and vomiting. She has no medical problems and does not take any medications. On physical examination her vital signs show her temperature is 38.9 C (102 F), blood pressure is 90/55 mmHg, pulse is 120 beats/minute, respiratory rate is 18 breaths/minute, and oxygen saturation is 98% on ambient air. She has left flank tenderness and lower abdominal tenderness. Laboratory studies show an elevated white blood cell count, but her basic metabolic panel is unremarkable. Urinalysis reveals pyuria with white blood cell casts. Blood and urine cultures are sent. In addition to intravenous fluids and hospital admission, which of the following is the most appropriate next step? A. Azithromycin B. Cefpodoxime C. Ceftriaxone D. Moxifloxacin E. Vancomycin and piperacillin/tazobactam

The correct answer is C. This patient's presentation is consistent with pyelonephritis. It is important to distinguish cystitis and pyelonephritis since initial antibiotic treatment and duration of therapy will be different. The classic symptoms of cystitis are dysuria, frequency, urgency, suprapubic pain, and/or hematuria. The urinalysis often shows pyuria and the patient is often treated for three days. They may also not require hospitalization. Pyelonephritis may have similar presentation to cystitis, but also includes fever, costovertebral angle tenderness, and nausea/vomiting. The urinalysis shows pyuria and may show white blood cell casts. The treatment duration for patients with pyelonephritis is often 7-14 days. The most common organism in both cystitis and pyelonephritis are gram-negative organisms. Ceftriaxone is the best answer. Broad-spectrum parenteral antimicrobials should be used for empiric treatment of complicated pyelonephritis (patient with sepsis). Ceftriaxone will cover the most likely cause of pyelonephritis, which are gram-negative organisms. Azithromycin is not an antibiotic that is used for pyelonephritis or even cystitis. Cefpodoxime is incorrect because it is important that this patient is placed on intravenous antibiotics since her clinical picture is consistent with sepsis in the setting of likely pyelonephritis and she is currently having nausea and vomiting. Moxifloxacin is not a preferred antibiotic in the management of pyelonephritis or cystitis due to very low urinary penetration. The combination of vancomycin and piperacillin/tazobactam is unnecessarily broad for most patients with pyelonephritis. Neither MRSA nor pseudomonas are common causes of pyelonephritis.

A 45-year-old male comes to clinic because of a four-day history of increasing back pain and one week of fever, chills, fatigue, and generalized malaise. Medical history is significant for a T10 spinal cord injury causing paraplegia following a motor vehicle accident 10 years ago. He has required self-catheterization for bladder decompression since the accident. Temperature is 38.7 C (101.7 F), pulse is 102 beats/minute, respiratory rate is 20 breaths/minute, and blood pressure is 98/55 mmHg. Oxygen saturation is 99% on room air. He appears tired and slightly pale. He has mild tenderness to palpation over the thoracic spine between his shoulder blades. His bladder is palpable in his lower abdomen. He has loss of sensation at the level of his umbilicus and paralysis of his lower extremities with increased tone. The remainder of his physical examination is normal. Which of the following is the most likely cause of this patient's back pain? A. Abdominal aortic aneurysm B. Cauda equina syndrome C. Infective endocarditis D. Spinal abscess E. Urinary tract infection

The correct answer is D. The most-likely cause of this patient's back pain is a spinal abscess (D). The patient has had fevers and has focal tenderness along his thoracic spine above his prior area of injury. He is at risk for systemic infection that could seed his vertebrae due to his history of neurogenic bladder and need for catheterization. Vertebral or epidural infections are usually accompanied by fever and symptoms of systemic illness plus the finding of focal pain. An aortic abdominal aneurysm (A) may present with back pain, tachycardia, and hypotension, but fever and focal tenderness on back examination would not be expected with this diagnosis. The patient has long-standing paralysis due to spinal cord injury at T10, so it would be hard to distinguish compression at the lower lumbosacral nerve roots with his baseline focal neurologic deficits. While cauda equina syndrome can be caused by any process that invades the subarachnoid space below the spinal cord, it is most commonly caused by neoplastic processes rather than infections. The patient's symptoms of systemic infection and focal tenderness over the upper thoracic spine are not typical of cauda equina syndrome (B). Patients with infective endocarditis (C) may present with similar symptoms as this patient, but they generally have a murmur on a cardiac exam and generally do not have focal back pain with tenderness to palpation, unless the spine has also been seeded as a potential complication of infective endocarditis. Patients with a neurogenic bladder requiring self-catheterization for bladder decompression are prone to urinary tract infections (E), as has been the case for this patient, but that would be less likely to result in focal midline back pain and thoracic spine tenderness on exam than spinal abscess.

A 61-year-old male recently diagnosed with small cell lung carcinoma presents with confusion. Head imaging is negative for brain metastases, but his serum sodium is 120 mEq/L. You know that syndrome of inappropriate antidiuretic hormone (SIADH) secretion is a common paraneoplastic syndrome associated with small cell lung carcinoma. If SIADH is causing his hyponatremia, what results would you expect on his clinical and laboratory evaluations? A. Euvolemia/low serum osmolarity/high urine osmolarity B. Hypervolemia/high serum osmolarity/low urine osmolarity C. Hypervolemia/low serum osmolarity/high urine osmolarity D. Hypovolemia/high serum osmolarity/low urine osmolarity E. Hypovolemia/low serum osmolarity/high urine osmolarity

The correct answer is A. An antidiuretic hormone (ADH) release is normally increased when volume status is low, to promote retention of free water and restoration of a normal volume. ADH release normally decreases when volume status is normal. A hallmark of the syndrome of inappropriate antidiuretic hormone (SIADH) is inappropriately high levels of ADH, regardless of volume status. ADH acts on receptors in the renal collecting ducts, promoting the resorption of water. This causes dilution of serum solutes (low serum osmolarity) and concentration of urine solutes (high urine osmolarity). However, it does not typically cause hypervolemia; most patients are euvolemic on exam.

A 58-year-old male experiencing homelessness presents to the emergency department with a decreased level of consciousness. Urine microscopy is performed, and numerous calcium oxalate crystals are present. What is the most likely explanation for the calcium oxalate crystals seen in his urine? A. Ethylene glycol ingestion B. Excessive ethanol intake C. Kidney stones D. Methanol poisoning E. Tricyclic antidepressant overdose

The correct answer is A. Calcium oxalate crystals can be a normal finding or appear in the urine of patients with increased calcium excretion for any reason (e.g., hyperparathyroidism), including those with recurrent kidney stones (C). However, the presence of numerous crystals in this man with a decreased level of consciousness strongly suggests ethylene glycol ingestion (A) and is one of the criteria for initiating treatment. Methanol poisoning (D), excessive ethanol intake (B), and tricyclic antidepressant overdose (E) can all cause a decreased level of consciousness but would not result in calcium oxalate crystals in the urine.

A 51-year-old healthy male comes to the clinic to review the findings from his recent screening colonoscopy. He had multiple diverticula in the sigmoid colon and internal hemorrhoids, but no colonic polyps. Family history is negative for colon cancer. Physical examination is normal. Which one of the following recommendations for future colorectal cancer screening would be most appropriate for this patient? A. Colonoscopy in 10 years for secondary prevention B. Fecal immunochemical testing in five years for secondary prevention C. Flexible sigmoidoscopy in five years for tertiary prevention D. Yearly fecal occult blood for tertiary prevention E. Yearly flexible sigmoidoscopy for primary prevention

The correct answer is A. Colonoscopy in 10 years would be an appropriate screening modality at the correct interval, and it constitutes secondary prevention. Other modalities that can be used for secondary prevention of colorectal cancer include annual stool-based testing (fecal occult blood test (FOBT) or fecal immunochemical test (FIT)) or flexible sigmoidoscopy every five years. An example of primary prevention of colorectal cancer would be the promotion of healthy lifestyle choices known to reduce the risk of developing cancer, e.g. regular exercise. Tertiary preventative measures would include more frequent colonoscopy for a patient who was already found to have neoplastic polyps or with a history of colon cancer resection.

A 53-year-old female comes to the clinic to follow up on blood work obtained last week. Due to signs and symptoms of anemia, a complete blood count was ordered. Results are shown. White blood count (WBC): 7.4 cells x 103/μL Hemoglobin: 9 g/dL Hematocrit: 27% Mean corpuscular volume (MCV): 75 μm3 Platelets: 338,000 /mm3. Which test would be most helpful to diagnose iron deficiency in this patient?

The correct answer is A. Ferritin is an indicator of body iron stores and is the best test to assess iron deficiency. When the ferritin is less than 15 ng/mL (15 μg/L), it is pathognomonic for iron deficiency. Although ferritin is an acute phase reactant and may be falsely elevated during acute illness, a diagnosis of iron deficiency is very unlikely if the serum ferritin is normal or elevated (greater than 100 ng/mL (100 μg/L)). Serum iron is low in iron deficiency anemia but is also low in anemia of chronic disease, so a low iron is not diagnostic of iron deficiency anemia. Serum iron can be used to aid in the diagnosis of iron deficiency anemia when other tests are considered, such as total iron binding capacity (TIBC) and transferrin saturation. TIBC can be calculated by multiplying transferrin concentration in mg/dL by 1.389. It is increased in iron deficiency and decreased in anemia of chronic disease. Transferrin saturation is the ratio of serum iron to TIBC and would be decreased in iron deficiency anemia.

A 23-year-old male is brought to the hospital on a hot summer evening with an altered mental status. He has no chronic medical conditions and takes no prescribed medications. He is known to drink alcohol regularly and use street drugs. He reportedly attended a party hours before being found confused and combative in his apartment, which is without air conditioning. Laboratory examination reveals a serum sodium of 160 and a creatinine of 2.1 mg/dL. What is the most likely cause of his hypernatremia? A. Alcohol intoxication with vomiting B. Diuresis from excessive beer intake C. Excess salty food intake D. Heroin overdose E. MDMA ("ecstasy") intoxication

The correct answer is A. Given that this patient was found on a hot day, the most likely explanation for his hypernatremia is through gastrointestinal (GI) losses combined with excessive sweating due to the heat, resulting in hypovolemia with a free-water deficit. Alcohol intoxication leading to vomiting (A) best fits with his laboratory findings. Diuresis from beer intake (B) is unlikely to cause hypernatremia. So-called "beer potomania" occurs in malnourished beer drinkers, and these patients develop hyponatremia An otherwise healthy young adult would not become hypernatremic from excessive salt intake (C), as he would be able to drink adequate water in response to thirst and can regulate his free-water balance with antidiuretic hormone (ADH) adjustments. Overdosing on heroin (D) is unlikely to cause hypernatremia. MDMA ("ecstasy") intoxication (E) often results in hyponatremia as it drives an excessive thirst mechanism as well as inappropriate ADH secretion.

A 72-year-old male presents with one hour of chest pain. The pain is vague, difficult to localize, gives a "squeezing" sensation, and is associated with dyspnea and nausea. On examination, his pulse is 110 beats/minute and regular, respiratory rate is 22 breaths/minute, blood pressure is 160/95 mmHg, and oxygen saturation is 92% on ambient air. He appears to be in moderate distress from pain. The rest of the exam is normal. What is the most likely diagnosis? A. Acute coronary syndrome B. Aortic dissection C. Pericarditis D. Pneumothorax E. Pulmonary embolism

The correct answer is A. Ischemic chest pain is often described as squeezing, tightness, pressure, or a crushing sensation. It's classically substernal and difficult to localize to a specific spot. Often there is radiation to the shoulders or lower jaw. Common associated symptoms include dyspnea, diaphoresis, and nausea. Aortic dissection typically presents with sudden onset of severe pain. The location of pain can depend on the location of dissection—tearing or ripping back pain is characteristic of a descending aorta dissection, whereas pain radiating to the jaw or neck is more typical of an aortic arch dissection, and severe anterior chest pain is common with aortic root or ascending aorta dissection. Acute pericarditis typically presents with sudden onset of sharp, stabbing pain localized over the center or left side of the chest. Pain is worse with deep breathing and laying down, but improved with sitting up or leaning forward. A pneumothorax typically presents with sudden onset of sharp unilateral pain associated with dyspnea. A pulmonary embolism characteristically presents with sudden onset of unilateral pleuritic chest pain. It is often associated with dyspnea, and cough with hemoptysis may also be present. A central pulmonary embolism can present without much chest pain, but with predominant signs and symptoms, including dyspnea, tachycardia, and lightheadedness.

A 36-year-old female without any past medical history presents to the emergency department with severe dehydration. Which of the following urinalysis results would be most likely found in this patient? A. Decreased urine sodium (Na), decreased fractional excretion of sodium (FENa), increased urine osmolality B. Decreased urine sodium (Na), increased FENa, decreased urine osmolality C. Increased urine Na, decreased FENa, increased urine osmolality D. Increased urine Na, increased FENa, decreased urine osmolality

The correct answer is A. Measurement of urine electrolytes is useful in the assessment of arterial blood volume. Severe dehydration leads to an attempt to conserve sodium and volume. This leads to a combination of decreased urine Na (<20 mmol/L), decreased FENa to <1%, and increased urine osmolality. However, there are two common situations where the FENa may be >1% in the setting of volume depletion: (1) severe volume depletion (pre-renal) which leads to acute tubular necrosis (intra-renal) and acute kidney failure; and (2) use of diuretics which cause sodium excretion despite ongoing dehydration. In the setting of diuretic use, the fractional excretion of urea (FEUrea) is more reliable than the FENa.

You are seeing a female in your clinic with unilateral leg swelling and pain as her chief concern. On exam, she has right-lower leg tenderness, edema, erythema, and a positive Homan's sign (calf pain with passive dorsiflexion of the ankle). Of her signs and symptoms, which, if any, is the most sensitive and specific for deep-vein thrombosis (DVT)? A. None B. Pitting lower-extremity edema C. Positive Homan's sign D. Tenderness on physical exam E. Unilateral leg pain

The correct answer is A. No individual sign or symptom is very sensitive or specific for DVT. You should review all the risk factors and signs and symptoms to determine a pretest probability of DVT and decide if further testing is needed. Validated clinical decision rules are available to help determine pretest probability and decide on next steps.

You are seeing a 45-year-old female in the primary care clinic with concerns of joint pain. She reports a six-month history of symmetrical pain and swelling of her metacarpophalangeal (MCP) and proximal interphalangeal (PIP) joints of both hands as well as her wrists, which are more painful and stiff in the morning. She also endorses difficulty holding her coffee mug due to weakness and generalized fatigue. On exam, you notice tenderness and edema of the wrists, MCPs and PIPs bilaterally. There are no oral lesions, she has no skin changes, no muscle tenderness, and no other weakness appreciated. Her exam is otherwise normal. Which of the following would be the most-appropriate diagnostic laboratory test to order? A. Anti-citrullinated peptide antibodies (anti-CCP) B. Anti-double stranded DNA (anti-dsDNA) C. Anti-neutrophil cytoplasmic antibodies (ANCA) D. Antinuclear antibody (ANA) E. Erythrocyte sedimentation rate (ESR)

The correct answer is A. Rheumatoid arthritis (RA) typically presents as symmetric pain and swelling of small joints, such as MCPs, PIPs, and metatarsophalangeals (MTPs). Other synovial joints may also be affected, as can the axial skeleton (cervical spine) later in the disease. This patient is presenting with classic symptoms of RA, so a test that is both sensitive and specific for RA would be the best choice. Anti-CCP antibody (A) has a sensitivity of 67% and specificity of 96% for RA. This will typically be sent in combination with a rheumatoid factor (RF), though early in the disease course the sensitivity of this test is much lower, around 30%. Anti-dsDNA (B) is a specific test for systemic lupus erythematosus (SLE). This patient is lacking certain clinical diagnostic criteria for lupus, including rash, oral ulcers and serositis, making this diagnosis less likely. Anti-neutrophil cytoplasmic antibodies (ANCA) (C) are seen most commonly in systemic vasculitis. Antinuclear antibodies (ANA) (D) and erythrocyte sedimentation rate (ESR) (E) can be elevated in a variety of diseases and would not be helpful in the diagnosis of RA.

A 67-year-old male comes to the clinic for a health maintenance visit. His past medical history is significant for chronic allergic rhinitis, severe chronic obstructive pulmonary disease (COPD), osteoporosis, psoriasis, atrial fibrillation, and benign prostatic hypertrophy. Vital signs show his temperature is 36.8 C (98.2 F), pulse is 76 beats/minute, respiratory rate is 12 breaths/minute, and blood pressure is 118/70 mmHg. His weight is 129.2 kg (285 lbs) and his body mass index (BMI) is 41. Which of his co-morbidities is most likely to be associated with his BMI? A. Atrial fibrillation B. Benign prostatic hypertrophy C. Chronic allergic rhinitis D. Osteoporosis E. Psoriasis

The correct answer is A. Obese patients are at a significantly increased risk for developing atrial fibrillation. This is thought to be related to increased left-atrial volume. Weight loss may reduce the burden of atrial fibrillation in obese patients. Also, obesity has been associated with COPD and asthma, although the nature of the association has not been fully elucidated. Additional related health risks include atherosclerotic cardiovascular disease (including stroke, coronary artery disease, and peripheral vascular disease) and heart failure. Psoriasis, chronic allergic rhinitis, and benign prostatic hypertrophy are less likely to be associated with obesity. Osteoporosis is associated with low BMI, not obesity.

A 52-year-old female comes to the clinic to discuss weight loss. Her medical history is significant for obesity; her body mass index (BMI) is 41; hypertension; hyperlipidemia; and obstructive sleep apnea. She knows that losing weight will help her hypertension and hyperlipidemia, but she doesn't feel like these things bother her. Her only other concern is fatigue; she doesn't use her continuous positive airway pressure (CPAP) machine, because she doesn't like the mask. What additional information can you provide her to help motivate her weight loss? A. Her obstructive sleep apnea may improve with weight loss. B. Her risk of cardiovascular disease is similar to that of a female with a normal BMI. C. Obesity is mainly a cosmetic issue. D. Surgery should be considered before diet and exercise.

The correct answer is A. Obesity is associated with a number of medical comorbidities affecting multiple organ systems, including the cardiovascular (atherosclerotic cardiovascular disease, atrial fibrillation, heart failure, venous thromboembolism), pulmonary (obstructive sleep apnea, obesity hypoventilation syndrome), gastrointestinal (gastroesophageal reflux disease, cholelithiasis, hepatic steatosis), endocrine (diabetes), and renal (chronic kidney disease). In patients with medical comorbidities related to obesity, weight loss is a cornerstone of therapy. In this patient, improvement in hypertension, hyperlipidemia, and sleep apnea can be expected with weight loss. Therefore, counseling regarding lifestyle interventions, weight loss medications, and possibly surgery if non-surgical interventions are unsuccessful, is warranted to manage obesity-related co-morbidities.

A 68-year-old male with a history of peptic ulcer disease, diabetes mellitus, and hypertension is admitted to the hospital with a 24-hour history of abdominal pain and three episodes of loose stool. Initial vital signs reveal temperature is 38.9 C (102.02 F), pulse is 102 beats/minute, respiratory rate is 16 breaths/minute, blood pressure is 132/78 mmHg, and oxygen saturation is 98% on room air. White blood count (WBC) is 13,100 cells/μl. He has tenderness to palpation of the left lower quadrant of his abdomen, with guarding but no rigidity. Renal function is normal. Computerized tomography (CT) scan of the abdomen and pelvis shows sigmoid diverticulitis without any sign of abscess or perforation. In addition to ordering intravenous (IV) antibiotics, IV fluid, and pain medication, what is the next best step in management? A. Blood cultures B. Colorectal surgery consult C. High-fiber diet D. Stool culture E. Stool test for occult blood

The correct answer is A. Obtaining blood cultures prior to starting antibiotics is essential, as this patient is showing signs of sepsis (fever, tachycardia, and leukocytosis) in the setting of a known infection. Patients who have recurrent episodes of diverticulitis, or who have acute complicated diverticulitis with perforation, abscess formation, or obstruction, should see a surgeon. However, a first-time episode of uncomplicated diverticulitis would not warrant a surgical consult. A high-fiber diet can help decrease constipation and intraluminal pressure, and may decrease the chance of recurrent diverticulitis, but in the acute setting, nothing by mouth or a clear liquid diet is indicated. Sending stool for culture is only useful if an acute bacterial diarrheal infection is suspected. In this case, CT scan shows diverticulitis. Bleeding is not typical with acute diverticulitis, and a positive stool guaiac without gross blood or melena is a non-specific finding. Diverticular bleeding usually presents independent of diverticulitis, and typically presents as painless, bright-red blood per rectum.

While on call in the hospital, you are called to the bedside of a female who was admitted four hours ago with severe pneumonia. Her breathing has become more shallow, she is increasingly lethargic and her oxygen saturation has dropped to 85% despite high-flow oxygen. You request an arterial blood gas (ABG) to help determine your next steps. Which of the following ABG results is most consistent with this patient's clinical presentation? A. pH: 7.15p CO2: 70 mmHg pO2: 60 mmHg HCO3: 28 mEq/L B. pH: 7.30p CO2: 30 mmHg pO2: 90 mmHg HCO3: 12 mEq/L C. pH: 7.37p CO2: 40 mmHg pO2: 90 mmHg HCO3: 22 mEq/L D. pH: 7.50p CO2: 60 mmHg pO2: 90 mmHg HCO3: 44 mEq/L E. pH: 7.65p CO2: 20 mmHg pO2: 70 mmHg HCO3: 16 mEq/L

The correct answer is A. Option A reflects an acute respiratory acidosis which is the acid-base disorder that best supports an acute respiratory decompensation. Note that the pCO2 is elevated. However, pCO2 cannot be the only variable to be relied on as elevations of pCO2 could be expected in someone with a chronic respiratory condition such as interstitial lung disease, or chronic obstructive pulmonary disease (COPD). We can infer that there has been a rapid decompensation if the bicarbonate is not significantly elevated in efforts to correct the pH. The hypoxia in this example also matches the clinical scenario. In addition, metabolic alkalosis (option D) would also typically lead to elevations in pCO2 as lungs retain CO2 to help correct the bicarbonate elevation that is acting to raise pH. Option B reflects a metabolic acidosis with low pH and bicarbonate, and a correspondingly low pCO2 due to respiratory compensation for the acidosis. Option C reflects the normal state. Option D represents a metabolic alkalosis. Option E reflects an acute respiratory alkalosis that can arise with hyperventilation due to drugs, anxiety, pulmonary embolism, acute asthma, pneumonia, etc. The hypoxia drives an increased respiratory rate, but because CO2 diffuses across the alveoli faster than oxygen (O2), the pCO2 decreases, and the pH subsequently increases.

A 52-year-old female presents for a follow-up visit for hypertension. Her aunt recently died from breast cancer, her mother is alive and without a history of cancer, and her father died 10 years ago from prostate cancer. The patient does not smoke cigarettes. She exercises regularly. Vital signs are pulse is 76 beats/minute, blood pressure is 138/72 mmHg, and body mass index (BMI) is 24.6 kg/m2. Her examination shows no abnormalities. Which U.S. Preventive Services Task Force (USPSTF) measures would you recommend to her to decrease her risk of breast cancer death? A. Biennial screening mammography B. Breast magnetic resonance (MR) now, then yearly screening mammography C. Clinical breast examination every six months by a clinician D. Monthly breast self-examination E. Yearly screening mammography

The correct answer is A. Per current USPSTF guidelines, for a female between 50-74 years of age, of average risk for breast cancer, routine mammography every two years is recommended. The USPSTF recommends against teaching breast self-examination. In addition, they conclude that there is insufficient evidence to make a recommendation regarding MRI and clinical breast examination as a screening modality. The USPSTF breast cancer screening guidelines are more conservative than those of some other organizations. This is based on concern that beginning mammography screening at a younger age and screening more frequently may increase the risk for false-positive results, unnecessary biopsies, overdiagnosis, and subsequent overtreatment.

A 68-year-old male with hypertension and non-ischemic cardiomyopathy is admitted to the hospital with an acute exacerbation of his congestive heart failure (CHF). An echocardiogram (ECG) one month ago showed left-ventricular hypertrophy and a left-ventricular ejection fracture (LVEF) of 60%. He takes ramipril and chlorthalidone at home, and these are continued. His pulse is 85 beats/minutes, blood pressure is 134/88 mmHg, and oxygen saturation is 95% on two liters of oxygen by nasal cannula. His exam is notable for jugular venous pulse and pressure (JVP) of 12, bilateral leg edema, and crackles at his lung bases. What will be the next step in management of this man? A. Add furosemide intravenously B. Add metoprolol orally C. Add valsartan D. Increase dose of chlorthalidone E. Increase dose of ramipril

The correct answer is A. The acute management of a congestive heart failure (CHF) exacerbation severe enough to require hospitalization includes loop diuretics such as furosemide. Because a CHF exacerbation can cause splanchnic edema, oral diuretics may be poorly absorbed, so the initial doses should be administered intravenously. Adding an angiotensin receptor blocker (ARB) such as valsartan is not indicated, as he is already taking an angiotensin-converting-enzyme (ACE) inhibitor. An increased dose of chlorthalidone would be insufficient to diurese a patient in the setting of an acute CHF exacerbation. A beta blocker or increased dose of ramipril will not help his symptoms acutely, though they should be considered after he is euvolemic.

A 70-year-old female is seen in the emergency department for palpitations. She states that over the last day she felt her heart beating "funny". She denies any chest pain, shortness of breath, cough, or fever. The paramedics obtained the electrocardiogram (ECG) prior to arrival. What is the diagnosis based on the ECG? A. Atrial fibrillation B. Atrial flutter with variable block C. Sinus arrhythmia D. Sinus rhythm with frequent premature atrial contractions E. Wandering atrial pacemaker

The correct answer is A. The answer is atrial fibrillation. The ECG shows a narrow, complex, irregularly irregular rhythm and there are no discernible p waves. Atrial flutter with variable block should have p waves in a saw-tooth fashion (usually running 280-300 beats/minute) with variable conduction resulting in QRS complexes after some of the p waves. Sinus arrhythmia varies in rate, but it has normal p waves before every QRS. Sinus rhythm with frequent premature atrial contractions would have sinus beats and frequent premature beats with a different p wave morphology. With a wandering atrial pacemaker, the ECG must show three different morphologies of p waves, which come from random ectopic atrial foci.

A 61-year-old male with a history of chronic obstructive pulmonary disease (COPD) is seen for a follow-up visit. He has smoked one pack per day for 45 years but recently cut back to half-a-pack per day after a recent upper respiratory infection. On examination, vital signs are within normal ranges. He has mild diffuse wheezing on lung exam, and the remainder of his exam is normal. What is the most appropriate next step in managing his nicotine dependence? A. Assess his readiness to quit smoking B. Prescribe a long-acting anticholinergic inhaler C. Prescribe nicotine replacement therapy D. Prescribe varenicline E. Refer for smoking cessation counseling

The correct answer is A. The most-appropriate next step is assessing this patient's willingness to quit smoking. It is not yet clear whether he is ready to quit smoking, so it would be inappropriate to prescribe smoking cessation products or to refer him for counseling if he was not ready to do so. If he is ready to stop smoking, then it would be appropriate to provide or refer him for smoking cessation counseling. It would also be appropriate to discuss the efficacy, side effects, and cost of various pharmacological treatment options and then prescribe whichever option seems most appropriate for him. With his previous diagnosis of COPD and wheezing on exam, a long-acting anticholinergic inhaler, such as tiotropium bromide, might be indicated, but prescribing this medication would not address his nicotine dependence.

A 22-year-old male comes to your office with fatigue and shortness of breath on exertion. On exam, you notice conjunctival pallor. You obtain a hemogram, which shows the following: Hemoglobin: 10 g/dL Hematocrit: 29% Mean corpuscular volume (MCV): 74 μm3 Red blood cell distribution width (RDW): 14% What is the most likely diagnosis? A. Beta thalassemia B. Hemolytic anemia C. Hypothyroidism D. Iron deficiency anemia E. Sideroblastic anemia

The correct answer is A. The patient has a low hemoglobin and hematocrit for his age and gender and therefore has anemia. The mean corpuscular volume is low so this is microcytic. Iron deficiency anemia and beta thalassemia minor are microcytic anemias. Sideroblastic anemia, hemolytic anemia, and anemia due to hypothyroidism are either normocytic or macrocytic. The red cell distribution width (RDW) is at the upper limit of normal; it should be elevated in iron deficiency anemia and is normal in beta thalassemia minor.

A 66-year-old male presents to his primary care provider with concern for intermittent chest pain. He describes several episodes a week of pressure under his sternum that comes on with exertion, lasts for about 5 to 10 minutes and is improved with rest, over the last two weeks. He notes some shortness of breath with the pain, but denies any associated diaphoresis, nausea, or jaw pain. He has a history of hypertension and hyperlipidemia treated with lisinopril, simvastatin, and aspirin. He exercises at a gym for 30 minutes, two to three times per week. Physical examination shows his pulse is 78 beats/minute, respiratory rate is 16 breaths/minute, and blood pressure is 145/80 mmHg. His lungs are clear, heart sounds are normal, and there is no lower-extremity edema. His electrocardiogram (ECG) is normal. Which of the following is the most-appropriate next step in his workup? A. Cardiac catheterization B. Computed tomography (CT) scan C. Echocardiogram D. Pharmacologic stress test with imaging E. Treadmill stress test without imaging

The correct answer is A. The patient should proceed directly for a cardiac catheterization. His symptoms meet the criteria for angina (substernal chest discomfort with a characteristic duration and features, exertional in nature and relieved with rest or nitroglycerin) and since it is relatively new and started within the month, it should be considered unstable angina. In addition, he has considerable risk factors, thus his pretest probability is high for cardiac disease and cardiac catheterization is recommended. A CT scan is not the first line for diagnosing angina. An echo alone would not be sufficient to diagnose coronary artery disease, or determine its severity and the need for treatment. Pharmacologic stress tests with imaging should be reserved for patients who are not able to exert themselves. If he had been experiencing symptoms for more than six weeks, he would be considered intermediate probability and a treadmill stress test without imaging would be indicated given his normal ECG and ability to exercise.

A female with human immunodeficiency virus (HIV) and a CD4 count of 24 cells/mm3 presents with fever and diarrhea for two months. She states no blood or mucus in her stool. She has not had any problems with diarrhea in the past. She reports no change in her diet. She does not take any medications. Which of the following would be the most likely diagnosis in this patient? A. Cytomegalovirus (CMV)-associated colitis B. E. coli O157:H7 infection C. Gluten intolerance D. Lymphocytic colitis E. Ulcerative colitis

The correct answer is A. The patient's CD4 count is below 100 cells/mm3, which increases her risk for CMV colitis. Gluten intolerance and lymphocytic colitis are usually not associated with fever. The diarrhea in an E. coli O157:H7 infection and ulcerative colitis is most often bloody.

A 23-year-old female with diabetes mellitus type 1, hypertension, and hypothyroidism is admitted to the intensive care unit (ICU) with lethargy, nausea, and diarrhea. She is somnolent on admission, however, her boyfriend provides some history. She has not taken her insulin for the past three days due to nausea and vomiting, she has had a poor appetite and diarrhea. She is difficult to arouse. On exam, she is afebrile. She has no other neurologic deficits. Her admission labs are shown: Sodium: 123 mEq/ Potassium: 4.8 mEq/L Chloride: 91 mEq/L Bicarbonate (HCO3): 10 mEq/L Blood urea nitrogen (BUN): 48 mg/dL Creatinine (Cr): 1.5 mg/dL Serum glucose: 540 mg/dL White blood cell (WBC): 8 cells x 103/μL Serum osmolality: 285 mosm/kg Urinalysis is pending. What is the most likely cause of her clinical presentation? A. Diabetic ketoacidosis B. Diarrhea C. Pseudohyponatremia D. Sepsis with lactic acidosis E. Surreptitious consumption of isopropanolol

The correct answer is A. The patient's anion gap and serum osmolol gap must be calculated: Anion gap = [Na - (Cl + bicarb)]. Values over 12 are considered elevated. Calculated osmolality = (2 x Na) + (glucose/18) +(BUN/2.8). This patient has an anion gap of 22, a calculated osmolality of 293, and an osmolol gap of 8.1 (both values elevated). Causes of an elevated anion gap and elevated osmolality gap include diabetic ketoacidosis, lactic acidosis, paraldehyde ingestion, and uremia. While the patient does have diarrhea which can cause a low serum bicarbonate, her other lab abnormalities and lethargy are explained by her diabetic ketoacidosis. Her low sodium is explained by her hyperglycemia, which causes osmotic shifts and a dilution hyponatremia. Pseudohyponatremia, on the other hand, is due to elevated serum lipids or proteins, which interfere with the sodium assay. She has no signs, symptoms, or clinical history to suggest lactic acidosis, uremia, or toxic ingestions.

In the emergency department, you are evaluating a 64 year old previously healthy male presenting with fever, altered mental status and shortness of breath. At admission, his temperature is 103 degrees Fahrenheit, heart rate is 128 beats per minute, respiratory rate is 27 breaths per minute, oxygen saturation 94% on 15L non-rebreather. On exam, he is hypotensive and appears acutely ill, with somnolence and respiratory distress with diffuse crackles. While your attending prepares to intubate the patient, bloodwork is collected. Labs show the following: Na 134 mEq/L Cl 100 mmol/L HCO3 14 mmol/L Lactic acid 4.9 mmol/L ABG: pH 7.33 / PaO2 61 mm Hg / PaCO2 31 mm Hg How do you interpret his acid/base status? A. Anion gap metabolic acidosis B. Anion gap metabolic alkalosis C. Anion gap respiratory alkalosis D. Non-anion gap metabolic acidosis E. Non-anion gap metabolic alkalosis

The correct answer is A. The patient's bicarbonate is low, which indicates a likely primary metabolic acidosis, which is more common than metabolic compensation for a respiratory alkalosis. In addition, based on the ABG collected, the patient does not have a respiratory alkalosis. Once a metabolic acidosis has been determined, the next step is to calculate the anion gap (Na - [Cl + HCO3]); for this patient it is 20 mEq/L, which is above the normal range of 8-16, indicating an anion gap metabolic acidosis. A helpful pneumonic for etiologies of anion gap metabolic acidosis is MUDPILES (see case for description). For this patient, his acidosis is likely due to severe sepsis from his acute illness.

Which of the following physical examination findings is most consistent with right-sided heart failure? A. Abdominojugular reflux B. Bibasilar crackles C. Pitting pedal edema D. Sustained apical impulse E. Systolic murmur at right upper sternal border

The correct answer is A. The positive predictive value for the abdominojugular reflux in right-heart failure is 91% (sensitivity 0.81; specificity 0.80). Upon applying 25-30 mmHg of pressure to the midabdomen, increased splanchnic return increases intracardiac pressure and causes a sustained (>/= 10 second) 4+-cm rise in the neck veins A sustained apical impulse and bibasilar crackles are more commonly associated with left-ventricular dysfunction. Pitting pedal edema is fairly nonspecific for heart failure and can be seen in both right and left sided failure.

An 85-year-old male undergoes evaluation for syncope that occurred while climbing up a flight of stairs. He has a history of hypertension and takes hydrochlorothiazide. He has been feeling lightheaded with exertion for about six months, but has never had a syncopal event until today. He denies any chest pain. On examination, his vital signs reveal his temperature is 36.6C (98F), pulse is 85 beats/minute, respiratory rate is 14 breaths/minute, supine blood pressure is 125/79 mmHg, and standing blood pressure is 119/74 mmHg. His neurologic exam is normal. Lungs reveal normal breath sounds bilaterally. A cardiovascular exam reveals diminished carotid pulse upstroke and a grade II/VI systolic murmur heard best at the right-upper sternal border. His abdomen is soft, and the extremities reveal trace edema bilaterally. His electrocardiogram (ECG) is shown below. What should you do next? A. Echocardiogram B. Exercise stress testing C. Serial troponin testing D. Stop the hydrochlorothiazide and bolus 500 ml normal saline E. Tilt-table testing

The correct answer is A. This male is older with exertional syncope and an exam consistent with aortic stenosis, so the next-appropriate step is to obtain an echocardiogram. Exercise stress testing is not indicated yet, and if the aortic stenosis is severe it could be dangerous. Serial troponin testing is indicated if ongoing ischemia or infarction is suspected, but the ECG shows only left ventricular hypertrophy (LVH), and the echo would be more important at this time. The patient is not orthostatic (defined as a systolic decrease of 20 mmHg or diastolic decrease of 10 mmHg with positional change), so it is less likely he has volume depletion as his cause of syncope. Tilt-table testing can sometimes help diagnose a patient with neurocardiogenic causes of syncope, but this patient is more likely to have aortic stenosis.

You are asked to evaluate a 75-year-old female with a history of coronary artery disease and hypertension who presents with forgetfulness and difficulty caring for herself. She is accompanied by her son who has noticed a progressive functional decline in his mother over at least the past year. She has been increasingly forgetful, including several incidents wherein she would leave water for tea boiling for hours. In the past two months, he has taken over the management of her bills, as she had missed several payments. She is able to ambulate without assistance. Two days ago, her neighbors found her wandering around the neighborhood in the middle of the night. When they tried to redirect her, she became agitated, yelling that strangers were trying to rob her. She calmed down when her son arrived to the scene. She states that she does not feel depressed. Her vital signs are normal. Her physical exam was unremarkable and a detailed neurological exam demonstrates no significant abnormal findings except for a Montreal Cognitive Assessment (MoCA) score of 10. She completes a "Get Up and Go" test in 15 seconds, rising from the chair and ambulating without difficulty. A comprehensive metabolic profile, complete blood count, thyroid stimulating hormone, and vitamin B12 level are normal. A non-contrast computed tomography (CT) of her head shows increased sulci. No other structural abnormalities are seen. What is the most-likely diagnosis? A. Alzheimer's dementia B. Frontotemporal dementia (FTD) C. Lewy body dementia D. Pseudodementia secondary to depression E. Vascular dementia

The correct answer is A. This patient likely has Alzheimer's dementia. She is demonstrating a gradual memory loss, short-term memory loss, and a progressive decline in function. While the diagnosis can only be confirmed at autopsy, a clinical diagnosis is 90% accurate. Vascular dementia is characterized by abrupt onset and step-wise progression of a patient's memory impairment often in the setting of a history of a stroke or with focal neurologic symptoms. Given her history of coronary artery disease and hypertension she has risk factors for this, but overall the history is more consistent with Alzheimer's​ dementia. Frontotemporal dementia is characterized by behavior and personality changes and language impairment. This can manifest as disinhibition, apathy, hyperorality, and compulsive behaviors such as hoarding or cleaning. This patient does not demonstrate this constellation of symptoms. Patients with Lewy body dementia typically have motor parkinsonism and visual hallucinations, this patient does not have these symptoms. In addition,​ no symptoms of depression are noted making pseudodementia secondary to depression unlikely.

A 48-year-old male with no significant past medical history presents to your office in the summer with eight days of fever, sore throat, rash, and myalgias. His nine-year-old daughter was treated for strep throat recently. He is a non-smoker, drinks four to five alcoholic drinks on the weekends, and does not use intravenous drug use (IVDU). He is sexually active with women. Exam results show temperature is 39.4 C (103 F), pulse is 116 beats/minute, blood pressure is 120/72 mmHg. He is ill-appearing, and your exam confirms swollen and erythematous pharynx, palpable and tender cervical and axillary lymph nodes, a diffuse maculopapular rash, and a palpable spleen. Examination of his genitalia is normal. Which of the following is the next best step in evaluating and managing this patient? A. Obtain a human immunodeficiency virus (HIV) viral load B. Obtain HIV antibody testing C. Perform a rapid strep test (RST) D. Perform darkfield microscopy on the rash E. Recommend hydration and rest

The correct answer is A. This male is presenting with signs of a systemic illness, and the differential diagnosis remains broad, but his sexual history is concerning for exposure to HIV. Acute HIV can present with lymphadenopathy, pharyngitis, rash, and myalgias. HIV viral load would be helpful in confirming or ruling out the diagnosis. While the HIV antibody should also be obtained, it would not be helpful in ruling out acute HIV given the up-to-three-month window period for seroconversion. One would not expect axillary lymphadenopathy and splenomegaly with streptococcal pharyngitis. Secondary syphilis is among the differential diagnoses and testing via non-treponemal and treponemal blood tests would be helpful, in addition to the HIV test. However, darkfield microscopy is useful for diagnosing syphilis if a chancre or condyloma lata is present.

A 36-year-old female is receiving a blood transfusion after having an abdominal hysterectomy. She has never had a blood transfusion previously. Both she and donor blood are type A+. As the first of two ordered units of packed red blood cells (PRBCs) is being infused, she develops fever and rigors. On exam, temperature is 37.9C (100.2F), pulse is 95 beats/minute, blood pressure is 122/68 mmHg, and oxygen saturation is 97% on room air. She appears diaphoretic. Lungs are clear with non-labored breathing. Abdominal exam is benign. There is yellow urine in her Foley catheter bag. Laboratory testing reveals hemoglobin of 6.5 g/dL, up from 6.0 g/dL postoperatively. Repeat blood type and crossmatch shows no incompatibility. What is the next best step in management for this patient? A. Use leukoreduced blood products for future transfusions B. Administer empiric broad-spectrum antibiotics C. Administer intramuscular (IM) epinephrine D. Give irradiated blood products for the next transfusion E. Give washed red blood cells for the next transfusion

The correct answer is A. This patient has likely experienced a febrile non-hemolytic transfusion reaction, with symptoms limited to fever and chills, along with normal urine output. This type of reaction is not threatening to her and can be managed with acetaminophen. It can be prevented by using blood products that are leukoreduced. IM epinephrine would be indicated for symptoms of anaphylaxis, which she is not experiencing. Anaphylactic reactions to transfusion are most common in immunoglobulin A (IgA)-deficient patients, and can be prevented by using washed red blood cells. Irradiated blood products are used when there is the concern for graft versus host disease (GVHD). GVHD develops 4 to 30 days after transfusion, and presenting symptoms are fever and maculopapular rash. Patients at highest risk of GVHD are those with hematopoietic cell transplant, those receiving blood transfusion from a family donor, and those given human leukocyte antigen (HLA)-matched platelets. Irradiated blood products are not needed for solid organ transplant recipients or those undergoing chemotherapy. Transfusion-related sepsis can cause fever, rigors, and hypotension. Transfusion-related sepsis is very rare, and patients are much more likely to have a febrile non-hemolytic transfusion reaction than sepsis. Transfusion-related sepsis is due to bacteria in the transfused unit of blood.

A 62-year-old male presents to his physician for a follow-up of his type 2 diabetes mellitus. He is asymptomatic and is currently taking metformin and aspirin. His pulse is 76 beats/minute, blood pressure is 142/92 mmHg, and body mass index (BMI) is 29 kg/m2. The remainder of his physical examination is within normal limits. Laboratory results are shown as: Glucose 132 mg/dL Urinalysis: Negative leukocyte esterase, negative nitrite, negative bili, negative blood, trace protein, Microalbumin 275 mcg/g. Initiation of which a drug from which of the following classes would be most appropriate for this patient? A. Angiotensin converting enzyme inhibitor B. Beta-blocker C. Calcium channel blocker D. Thiazide diuretic E. Vasodilator

The correct answer is A. This patient has microalbuminuria, and his blood pressure is above the goal (American Heart Association and American College of Cardiology guidelines suggest pharmacologic treatment of blood pressure >/= 130/80 mmHg in patients with diabetes). In this situation, treatment with an angiotensin-converting-enzyme (ACE) inhibitor or angiotensin receptor blocker is recommended, as these medications will effectively manage the man's elevated blood pressure and slow the progression of diabetic nephropathy. Although beta-blockers, calcium-channel blockers, thiazide diuretics, and vasodilators can all be used to attain blood pressure goals, they are not first line for this patient with microalbuminuria.

A 75-year-old male comes to the clinic for a review of the workup of his chronic kidney disease (CKD). His past medical history is significant for hypertension, diet-controlled diabetes mellitus type II, coronary artery disease, and benign prostatic hyperplasia (BPH). His blood pressure and diabetes have been well-controlled by antihypertensive medications and following a diabetic diet. Urinalysis and urine microscopy are normal. His creatinine is 1.5 mg/dL with a GFR of 55mL/min. Renal ultrasound shows normal-sized kidneys with good corticomedullary differentiation, mild hydronephrosis, and normal blood flow per dopplers. What is the most likely cause of his chronic kidney disease? A. Benign prostatic hyperplasia B. Diabetes mellitus C. Glomerulonephritis D. Hypertension E. Renal artery stenosis

The correct answer is A. This patient has multiple risk factors for CKD, including diabetes mellitus, hypertension, and BPH. In addition, he has coronary artery disease, which increases his risk for renal artery stenosis. However, his urine is bland, and his kidneys appear normal on an ultrasound except for the hydronephrosis. These findings support BPH as the cause for his CKD. Diabetic nephropathy tends to cause larger-than-normal kidneys with poor corticomedullary differentiation and proteinuria.

A 58-year-old female presents to the clinic with concern for chest pain over the past three months. She describes the pain as sharp and stabbing, in the mid-sternal region, lasting for one to two minutes, occurring a few times a day. The pain can come on at rest or with exertion and resolves on its own. It has not become worse since it began. There is no associated diaphoresis, shortness of breath, nausea, jaw pain, or pain with movement, eating, or laying supine. She has a 10-year history of obesity and hypertension for which she takes chlorthalidone and lisinopril. She was recently diagnosed with diabetes that has been controlled by diet. Physical examination shows her pulse is 86 beats/minute, respiration rate is 16 breaths/minute, and blood pressure is 135/85 mmHg. Her lungs are clear, heart sounds are normal, and there is no chest wall tenderness to palpation or abdominal tenderness. There is no peripheral edema. How would you best characterize her chest pain? A. Atypical angina B. Gastroesophageal reflux C. Musculoskeletal D. Stable angina E. Unstable angina

The correct answer is A. This patient is experiencing atypical angina. She does not meet the criteria for angina which includes substernal chest discomfort with characteristic duration and features, is exertional in nature and relieved with rest or nitroglycerin. This is considered atypical angina because the pain does not follow the classic pattern of angina; however, it is still possible that the pain is cardiac in origin, especially since atypical features are more common in women and patients with diabetes. Gastroesophageal reflux typically occurs after meals or while laying flat and is often described as having a burning quality. Musculoskeletal pain is typically worse with certain movements and associated with chest wall tenderness. Stable angina would meet the criteria and follow a predictable pattern with exertion. Unstable angina is characterized by chest pain at rest or with progressively less exertion. Angina with worsening features or new within the past four to six weeks is also considered unstable.

A 63-year-old male with no past medical history is hospitalized after a fall and inability to care for himself at home. On his fourth day in the hospital, he is transferred to the intensive care unit (ICU) for sepsis due to a urinary tract infection (UTI). Which of the following would be the most-appropriate empiric antibiotic treatment for this patient? A. Ampicillin and gentamicin B. Azithromycin and trimethoprim-sulfamethoxazole C. Ceftriaxone and cefipime D. Ciprofloxacin and nitrofurantoin E. Vancomycin and metronidazole

The correct answer is A. This patient requires empiric antibiotics that will cover gram-negative bacteria, including E. Coli, Proteus, Enterobacter, Klebsiella, and Pseudomonas, and gram-positive organisms such as Enterococci. The combination of ampicillin and gentamicin is the best option; the ampicillin would cover for Enterococci, and the aminoglycoside for Enterobacter and Pseudomonas. Azithromycin and trimethoprim-sulfamethoxazole would not be effective in sepsis due to the UTI; Enterococcus is often resistant to trimethoprim-sulfamethoxazole and would not cover Pseudomonas. Ceftriaxone and cefepime are in the same antibiotic class (cephalosporins), and such a combination should be avoided. Ciprofloxacin and nitrofurantoin do not provide broad-spectrum coverage. Vancomycin and metronidazole would not cover gram-negative organisms.

A 56-year-old female is being evaluated in the hospital for anemia. She was admitted five days ago with a myocardial infarction. She underwent stent placement and was doing well until last night when she developed lower abdominal cramping and started passing blood clots in her stools. Her medical history is significant for type II diabetes mellitus, hypertension, hypothyroidism, and rheumatoid arthritis. On physical examination, she appears to be in mild distress. Her pulse is 100 beats/minute and blood pressure is 110/54 mmHg. Abdominal exam reveals tenderness in bilateral lower quadrants with no guarding or rigidity. The remainder of her exam is normal. A complete blood count (CBC) obtained today is shown. White blood count (WBC): 5 cells x103/μL Hemoglobin (Hgb): 8 g/dLHematocrit (Hct): 24% Mean corpuscular volume (MCV): 84 μm3 Red blood cell distribution width (RDW): 12 Platelets: 280,000 /mm3A complete blood count obtained at the time of hospital admission is shown. WBC:11 cells 103/μL Hgb:13 g/dL Hct: 39% MCV: 86 μm3 RDW: 11 Platelets:150,000 /mm3 What is the most likely cause of this patient's anemia? A. Acute blood loss B. Anemia of chronic disease C. Hemolysis D. Hypothyroidism E. Iron deficiency

The correct answer is A. This patient was admitted with a normal CBC, which suggests that her chronic diseases such as hypothyroidism or rheumatoid arthritis were not contributing to anemia. She developed an acute gastrointestinal bleed during her hospitalization which was likely due to antiplatelet medications used for the management of her myocardial infarction. Her hemoglobin and hematocrit have dropped to 8 g/dL from 13 g/dL in five days, suggesting an acute process. Although hemolysis may cause a similar pattern for the CBC, there are no clues to suggest that there is a cause of hemolysis occurring in this patient at this time.

You are working at a skilled nursing facility, rounding on your patients. The nurse tells you that Mrs. Viera, an 83 year-old resident with a history of heart failure, has been having vomiting, diarrhea, and fever since the previous afternoon. The nurse also reports a decrease in urine output since early that morning. On your assessment, you notice that Mrs. Viera is slightly more confused than baseline, with dry mucous membranes. On chart review, you see that her baseline creatinine is 1.6 mg/dL, and that her medication list includes furosemide. You order stat labs which come back with the following results: Serum sodium: 134 mEq/L Serum blood urea nitrogen (BUN): 17 mg/dL Serum creatinine: 2.1 mg/dL Urinary sodium: 200 mEq/L Urinary blood urea nitrogen (BUN): 385 mg/dL Urinary creatinine: 220 mg/dL How does the fractional excretion of sodium (FENa) help you identify the etiology of the patient's acute kidney injury (AKI)? A. FENa cannot be reliably interpreted in this patient B. FENa is <1%, indicating a pre-renal etiology C. FENa is <1%, indicating likely acute tubular necrosis (ATN) D. FENa is >1%, indicating a pre-renal etiology E. FENa is >1%, indicating likely acute tubular necrosis (ATN)

The correct answer is A. This patient's calculated FENa is 1.4%. FENa <1 has a 96% sensitivity and 95% specificity for distinguishing pre-renal causes from acute tubular necrosis (ATN). However, since the equation uses the urinary sodium, this calculation cannot be reliably interpreted in patients taking loop diuretics, which cause increase in urinary sodium excretion. Therefore, in patients taking loop diuretics presenting with AKI, the fractional excretion of urea (FEUrea) is the preferred equation: FEUrea ≤35% indicates pre-renal etiology with 78% sensitivity and 88% specificity, whereas FEUrea >50% indicates a diagnosis of ATN. This patient's calculated FEUrea is 21.6%, which supports the presumptive diagnosis of pre-renal AKI based on the patient's history and exam.

A 58-year-old female with chronic kidney disease stage III, secondary to type II diabetes mellitus, presents to the clinic to establish care. Her blood pressure is 154/86 mmHg. Her body mass index (BMI) is 30 kg/m2. Her hemoglobin A1C is 9.8 mg/dL, and urine protein/creatinine ratio is 1.5 mg/dL. She takes long-acting insulin. In addition to tighter glucose management, which of the following would be the best treatment for diabetic nephropathy? A. Lisinopril B. Pentoxifylline C. Protein restriction D. Simvastatin E. Spironolactone

The correct answer is A. Treatment of diabetic nephropathy focuses on glycemic and blood pressure control. Intensive glycemic control can delay the development of proteinuria and decrease in glomerular filtration rate (GFR). Treatment of hypertension, particularly with blockers of renin-angiotensin, slows loss of renal function. Both angiotensin receptor blockers (ARBs) and angiotensin-converting-enzyme (ACE) inhibitors (such as lisinopril) decrease the development and worsening of proteinuria and slow the loss of GFR. However, using an ACE inhibitor and ARB together does not slow loss of renal function further and is associated with hyperkalemia and increased incidence of acute kidney injury. Some studies have shown that adding spironolactone (mineralocorticoid receptor agonist) to lisinopril decreases proteinuria, but there is no long-term data regarding the slowing loss of GFR. Pentoxifylline has been shown to decrease proteinuria and slow loss of GFR in small non-randomized studies. Additional studies are needed to verify the effect of pentoxifylline. Lipid lowering is important in the overall management of diabetes, as diabetes is considered a coronary heart disease equivalent, but it has not been shown to prevent diabetic nephropathy. Low-protein diets have been studied in diabetic nephropathy and it is uncertain if protein restriction slows decline in GFR. Additionally, diabetics are at increased risk for protein malnutrition, so protein restriction should not be recommended to the patient.

A 77-year-old female is brought to the office by her daughter with complaints of fatigue, poor appetite, and forgetfulness that have been getting progressively worse over the past three months. Conversing with the patient is difficult, as she only provides one-word answers to your questions; her daughter provides most of the history. The patient has had weight loss and constipation for which she has been taking glycerin suppositories. She has had more difficulty bathing and decreased motivation to practice good hygiene. She lives alone, but her daughter is nearby and has been visiting her with increasing frequency over the past month. On physical examination, she is afebrile. Her pulse is 120 beats/minute. Her blood pressure is 140/90 mmHg. She appears frail and disheveled. A mini-mental state examination (MMSE) is performed, and her score is 7. What would be the ​next best step in management? A. No further lab testing needed. Start donepezil. B. Order a serum folate level C. Order a thyroid stimulating hormone (TSH) level D. Order a transthoracic echocardiogram E. Order an erythrocyte sedimentation rate (ESR)

The correct answer is C. Goal: Identify atypical manifestations of hyperthyroidism as a reversible cause of dementia in older adults. In an older adult patient with a new diagnosis of dementia, a thorough evaluation of reversible causes of dementia should be conducted. The American Academy of Neurology (AAN) recommends that patients be screened for depression and be tested for possible underlying thyroid disease and vitamin B12 deficiency. While hypothyroidism is more common as a cause for dementia, as it can lead to cognitive slowing, hyperthyroidism is also associated with dementia. Graves disease is the most likely underlying cause of​ hyperthyroidism. Hyperthyroidism may present atypically in older adult patients. Referred to as "apathetic hyperthyroidism," it can present without the common symptoms of hyperactivity, tremor, or other manifestations of sympathetic overdrive. A goiter is less likely in older patients. Tachycardia may be present in about 60 percent of older hyperthyroid patients. Folate deficiency is not associated with dementia syndromes. While she may ultimately require an echocardiogram to diagnose the etiology of her tachycardia, an initial evaluation should involve an electrocardiogram to define her underlying rhythm. ESR is a non-specific marker of inflammation and will add little to the evaluation of secondary causes of dementia.

A 26-year-old male with human immunodeficiency virus (HIV) presents to the clinic with one week of blurry vision and floaters in his right eye. He has not had any exposure to bacterial conjunctivitis at his work in an elementary school. He does not wear corrective lenses. He had a fever this morning to 38.2 C (100.8 F) with associated malaise and mild abdominal pain and one watery stool. He admits to nonadherence with his HIV medication regimen. A funduscopic exam was performed, which showed the following. What is the most-likely organism responsible for these findings? A. Chlamydia trachomatis B. Cytomegalovirus C. Herpes zoster D. Retinitis pigmentosa E. Toxoplasmosis

The correct answer is B. Acute cytomegalovirus (CMV) infection often presents with visual symptoms due to retinitis, and colitis. Usually the CD4+ count will be less than 100 cells/mcl. This patient has been non-adherent with his medication, increasing the risk that his HIV will progress to AIDS. CMV infection involving the eye is an AIDS-defining opportunistic infection. Chlamydia and zoster do not affect the retina. Retinitis pigmentosa is a genetic disease. Toxoplasmosis is also an opportunistic infection, but the retina findings demonstrate necrotizing retinochoroiditis and are usually bilateral.

Two victims of a multiple-vehicle car accident are being treated in the emergency department trauma bay. Both have severe injuries resulting in blood loss, and four units of packed red blood cells (PRBCs) are ordered from the blood bank for each patient. The stickers to label each patient's blood samples were mixed up, so these two patients each mistakenly receive the blood intended for the other. The patient with type O+ blood ends up receiving a transfusion of B+ blood, and you are concerned that she will develop an acute hemolytic transfusion reaction. What findings would be most consistent with this type of transfusion reaction? A. Fever and nausea within hours of the transfusion, with normal urine output B. Fever, dyspnea, hypotension, and dark urine within minutes of starting the transfusion C. Fever, spherocytosis, and hyperbilirubinemia 2 to 10 days after the transfusion D. Hypotension and angioedema within minutes of starting transfusion E. Pulmonary edema within hours of transfusion

The correct answer is B. Acute hemolytic transfusion reaction can occur due to mis-labeling of specimens and ABO incompatibility. Classic findings include fever, dyspnea, hypotension, and dark urine within minutes of starting the transfusion. A febrile non-hemolytic transfusion reaction can cause fever and nausea within hours of a transfusion, but the urine is unaffected. A delayed hemolytic transfusion reaction can cause fever, spherocytosis, and hyperbilirubinemia 2 to 10 days after transfusion. Hypotension and angioedema within minutes of starting the transfusion may be due to anaphylaxis. Transfusion-related acute lung injury (TRALI) can present with pulmonary edema within hours of a transfusion.

A 42-year-old female presents to the emergency department with concern for mild chest pain lasting three to four minutes with vigorous exercise, three times over the past week. She has no past medical history and is not taking any medications or supplements. She has no family history of cardiac or pulmonary disorders. She follows a vegetarian diet, exercises regularly, and is training for a half-marathon. Physical examination shows her pulse is 66 beats/minute, respiration rate is 16 breaths/minute, and blood pressure is 110/70 mmHg. Her lungs are clear, heart sounds are normal, and there is no lower-extremity edema. Which of the following is the most likely laboratory study in the acute setting to assist with the diagnosis? A. Brain natriuretic peptide (BNP) B. Complete blood count (CBC) C. Hemoglobin A1c D. Low-density lipoprotein (LDL) E. Triglycerides

The correct answer is B. Although cardiac causes of chest pain should be considered, it is important to consider other sources of pain. Anemia may cause chest pain by decreasing oxygen carrying capacity. BNP should be ordered when congestive heart failure is suspected; however heart failure is unlikely in this case, given her clear lungs, lack of edema, and lack of risk factors. Hemoglobin A1c would be helpful for the diagnosis of insulin resistance and diabetes, which are significant risk factors for cardiac disease, but do not cause chest pain on their own. LDL and triglycerides would be helpful to characterize her cardiac risk, but would not help determine the source of her symptoms.

A 65-year-old female presents to the emergency department with one week of progressive confusion, fatigue, and global weakness. Her history is significant for widely metastatic renal-cell carcinoma. She has not had a bowel movement in five days. Laboratory studies are shown. Sodium: 142 mEq/L Potassium: 4.2 mEq/L Chloride: 105 mEq/L Bicarbonate: 18 mEq/L Calcium: 15 mg/dL Albumin: 3.9 g/dL Goals of care are discussed with her wife, and she requests inpatient therapy. Along with saline resuscitation, which medication should be given to help lower her calcium levels? A. Acetazolamide B. Calcitonin C. Furosemide D. Hydrocholorthiazide E. Spironolactone

The correct answer is B. Calcitonin is given to patients with severe hypercalcemia (>15 mg/dL), along with intravenous (IV) hydration. It has multiple mechanisms of action to lower serum calcium, including increasing urinary calcium excretion, antagonism of parathyroid hormone, and inhibiting osteoclast bone resorption. Bisphosphonates, such as zoledronic acid and pamidronate, are potent medications that treat hypercalcemia by inhibiting osteoclastic bone resorption. These medications take effect in two to four days, and are therefore used in conjunction with IV and/or calcitonin. Furosemide is a loop diuretic which can induce calcium excretion in the urine, however, due to complications (such as dehydration and hypokalemia) and more-effective medications, this is no longer routinely recommended. Hydrochlorothiazide increases calcium reabsorption in the distal convoluted tubule of the nephron, so would be contraindicated in the setting of hypercalcemia. Sprinolactone and acetazolamide do not significantly impact serum calcium levels so are not used as treatments for hypercalcemia.

A 67-year-old female with a history of diet-controlled gastroesophageal reflux disease presents to her general internist for her annual well visit. She has been doing well since her last visit one year ago. Her only medication is omeprazole as needed, which she takes rarely. Six months ago, she had an unremarkable mammogram and four years ago she had an unremarkable colonoscopy. She has a history of regular cervical cancer screenings without any abnormal results. She exercises regularly, does not drink, and has never smoked. She works as a part-time librarian. Her mother and father both lived into their 80s, and both died of cardiac disease. Physical exam, including vital signs, are normal. Which of the following screening modalities is most appropriate for this patient? A. Abdominal ultrasound B. Dual-energy x-ray absorptiometry (DEXA) scan C. Papanicolaou (pap) test D. Transvaginal ultrasound E. Upper endoscopy

The correct answer is B. DEXA is the screening modality for osteoporosis screening. Osteoporosis is commonly seen in older adults, particularly in post-menopausal females. It can carry significant mortality and morbidity (for example, fracture) if undiagnosed. The U.S. Preventive Services Task Force (USPSTF) recommends screening every female 65 years old and over for osteoporosis. This patient is 67 and has not yet been screened for osteoporosis. Papanicolaou test is the screening modality for cervical cancer screening. The USPSTF recommends cervical screening is dependent on age and other risk factors and prior screening. Age Recommendation Females who are under 21 years of age No screening for cervical cancer Females who are 21-29 years old Papanicolaou test every 3 years Females who are 30-65 years old Screening with high risk human papillomavirus (HPV) testing alone or in combination with cytology every five years Females who are over 65 years old with no history of abnormal pap and three negative pap tests (or two negative HPV tests) within 10 years prior Do not require further screening This patient is 67 years old with three prior negative PAP tests and hence does not require further screening. Abdominal ultrasound is the screening modality for aortic abdominal aneurysm. It is not recommended in females. It is recommended as a one-time screening in males who are 65-75 years old who have ever smoked. Transvaginal ultrasound can be used to screen for ovarian cancer in select high-risk patients. In this patient without any risk factors for ovarian cancer, the USPSTF recommends against screening. Upper endoscopy is used to evaluate the upper gastrointestinal (GI) tract. In this patient with mild gastroesophageal reflux disease (GERD) that is well-controlled with rare pharmacotherapy, there is no indication or red flags for endoscopy.

Mary is an 18-year-old female with a past medical history significant for type 1 diabetes mellitus who was admitted to the intensive care unit (ICU) yesterday with nausea, vomiting, and a decreased level of consciousness. Her labs were consistent with diabetic ketoacidosis, and she was started on an insulin drip and intravenous fluids. Today, she is more alert and has no further nausea or vomiting. Her anion gap has closed. Prior to stopping the insulin drip, which type of insulin is most likely to prevent a recurrence of ketoacidosis? A. Intermediate-acting insulin B. Long-acting insulin C. Premixed insulin (combination of short- and long-acting insulin) D. Rapid-acting insulin E. Short-acting insulin

The correct answer is B. During diabetic ketoacidosis, an insulin drip using rapid- or short-acting insulin is necessary to close the anion gap. Once the anion gap has normalized, and if the patient is alert and able to eat, subcutaneous insulin may be started. A long-acting insulin is the preferred subcutaneous insulin because it has no peak and covers basal insulin needs for a full day. Because long-acting insulin does not start working for one to one-and-a-half hours, the insulin drip should continue for one to two hours after the subcutaneous insulin has been administered.

Ms. H is a 68-year-old female with a medical history significant for obesity, type II diabetes, hypothyroidism, hypertension, and recently diagnosed hyperlipidemia. Her most-recent lipoprotein (LDL), three months ago, was 197 mg/dL. At that time, atorvastatin was initiated. Other medications include metformin, insulin glargine, amlodipine, hydrochlorthiazide, and levothyroxine. Which of the following may be contributing to her elevated LDL? A. Amlodipine B. Hydrochlorthiazide C. Insulin glargine D. Levothyroxine E. Metformin

The correct answer is B. Dyslipidemia is typically familial, although there are secondary causes of hyperlipidemia that clinicians should be aware of. These include type II diabetes, cholestatic or obstructive liver disease, nephrotic syndrome, acute hepatitis, alcohol, and medications including hydrochlorthiazide, beta-blockers, oral contraceptives, and protease inhibitors. In addition to a possible familial cause of hyperlipidemia, this patient's type II diabetes, hypothyroidism, and hydrochlorthiazide all may be contributing to dyslipidemia.

Six weeks ago, a 67-year-old female presented to her primary care physician with dyspepsia and epigastric pain. She did not drink alcohol and took no medications. Testing revealed a positive stool-occult blood test and mild microcytic anemia. She underwent upper and lower endoscopy and was found to have a duodenal ulcer. Biopsy was positive for Helicobacter pylori (H. pylori), and she completed the prescribed course of proton-pump inhibitor and antibiotics. Now she is returning to the clinic and reports that her symptoms have not resolved. What is the best next step in diagnosis and management for this patient? A. Perform computed tomography (CT) of the abdomen B. Perform H. pylori stool antigen test C. Repeat upper endoscopy with biopsy D. Resume proton-pump inhibitor indefinitely E. Test serum gastrin level

The correct answer is B. H. pylori infection and non-steroidal anti-inflammatory drugs (NSAIDs) cause the vast majority of peptic ulcer disease. In this patient's case, it would be appropriate to do testing for eradication of H. pylori since her symptoms did not respond to appropriate treatment. This could be done with the H. pylori stool antigen test or urease breath test - invasive testing with repeat endoscopy is not necessary. It would be best to confirm eradication of the underlying cause of the ulcer before committing her to lifelong acid suppressant therapy. If testing confirms H. pylori eradication, then it would be appropriate to consider investigating for less-common causes of peptic ulcer disease, such as gastrinoma with a gastrin level. A CT scan of the abdomen is unlikely to help you currently. Resuming the proton pump inhibitor may be reasonable but only after checking for H. pylori eradication.

A 57-year-old male presents with a five-day history of productive cough with purulent sputum, fatigue, and fever. He has a 40-pack-per-year smoking history. On examination, he is moderately ill-appearing. Temperature is 38.6 C (101.4 F), pulse is 110 beats/minute, respiratory rate is 22 breaths/minute, blood pressure is 118/76 mmHg, and oxygen saturation is 92% on room air. Chest exam reveals tachycardia without murmurs and crackles in the left-lower lung field. What is the most appropriate next diagnostic step? A. Chest computed tomography (CT) B. Chest x-ray C. Complete blood count (CBC) with differential D. Influenza testing E. Pulmonary function testing

The correct answer is B. In this ill-appearing patient who has a fever, tachypnea, and hypoxemia, as well as crackles on examination, a chest x-ray will be the first line to diagnose pneumonia. A CT scan would also show pneumonia, but exposes him to more radiation and is more expensive than a chest x-ray. A CBC with differential may show an elevated white blood cell count, but will not help clarify his diagnosis. While influenza can cause high fevers and may result in pneumonia, given his abnormal lung examination, imaging would be first line for evaluation. Pulmonary function testing may be indicated in the future to evaluate for chronic obstructive pulmonary disease (COPD) given his smoking history, but will not help in management of this acute illness.

A 78-year-old male is evaluated in a subacute rehabilitation center after sustaining a fall. The fall occurred last night as he was getting out of bed. He does not recall the circumstances of the fall. Medical history is significant for Parkinson disease for which he has taken carbidopa-levodopa for four years. He was hospitalized for community-acquired pneumonia two weeks ago and completed his course of levofloxacin four days ago. During his admission, diphenhydramine was started and he continues to take it at bedtime. Today, his temperature is 37.6C (99.7F), pulse is 85 beats/minute, and blood pressure is 145/80 mmHg. His physical exam reveals lungs clear to auscultation, regular heart rate, no appreciable heart murmur, and mild ecchymosis on his left hip. He scores 27/30 on a mini-mental state examination (MMSE). He is able to stand and ambulate 20 feet without the use of assistance devices. He is cooperative during the exam. Which of the following most likely triggered this fall? A. Arrhythmia due to levofloxacin B. Delirium due to diphenhydramine C. Dementia secondary to Parkinson disease D. Limited mobility due to Parkinson disease E. Sepsis due to healthcare-associated pneumonia

The correct answer is B. Medications can be a major risk factor for falls in older patients. For example, postural hypotension can be related to the use of antihypertensive medications. Medications used for other reasons may also cause hypotension or other adverse reactions (e.g. tamsulosin given for benign prostatic hypertrophy). Medications such as diphenhydramine, benzodiazepines, and zolpidem are sometimes prescribed as sleep aids, but these medications may have additional undue effects on older adult patients. Diphenhydramine has been associated with drug-induced delirium, and it is likely that his fall occurred as a result of delirium. His underlying Parkinson disease may be a contributing factor to the fall, but as he is able to stand and ambulate without assistance, it is unlikely that this condition is severe enough to be the main factor. While levofloxacin can cause prolonged QT and increase the risk for cardiac arrhythmias, he is no longer on the medication. His examination is not suggestive of sepsis or dementia.

A 30-year-old fair-skinned female presents to the office for her first skin check. She has multiple lesions on her back; each of them is slightly different. Which lesion would be most concerning for melanoma? A. A brown lesion measuring 4 mm of uncertain duration B. A symmetric black lesion which has doubled in size since childhood C. A uniformly black lesion present since childhood D. An asymmetric black lesion present since childhood E. An asymmetrical brown pigmented lesion present since childhood

The correct answer is B. The ABCDE criteria are used to identify pigmented lesions which are more likely to be diagnosed as melanoma. They include asymmetry, border irregularity, color variation, diameter >6 mm, and evolving changes in shade, size, symptoms, and ulceration. Of the five ABCDE criteria, evolving character has the highest specificity for melanoma (90%) making the lesion that has doubled in size the most concerning for melanoma.

A 24-year-old male comes to the clinic to establish care as a new patient. His medical history is significant for intermittent asthma. Social history reveals that he is a graduate student. He says that he drank alcohol heavily when he was in an undergraduate fraternity two years ago, but that he has cut back significantly on his alcohol use and no longer drinks every day. He does have intense cravings for alcohol at times and has missed a few classes because he was hungover from a binge and unable to get to class. He received a driving-under-the-influence (DUI) citation two months ago, which he admits was a "really stupid mistake." On physical exam, he is well-groomed and pleasant. His vital signs reveal pulse is 80 beats/minute, respiration rate is 12 breaths/minute, blood pressure is 138/82 mmHg. His physical examination is normal. What is the best description of the alcohol consumption in this patient? A. Alcohol abuse B. Alcohol use disorder C. Occasional alcohol use D. Previous alcohol dependence E. Risky alcohol behavior

The correct answer is B. The best characterization of his alcohol use is alcohol use disorder, based on the Diagnostic and Statistical Manual of Mental Disorders, Fifth Edition (DSM-5) criteria. Alcohol use disorder must meet at least 2 of 11 criteria. This patient meets three criteria, including (1) craving or strong desire to use alcohol; (2) recurrent alcohol use resulting in a failure to fulfill major role obligations at work, school, or home; and (3) recurrent alcohol use in situations in which it is physically hazardous. Alcohol abuse is an older term. The DSM-5 has changed the name to alcohol use disorder, which includes criteria for abuse and dependence. Occasional alcohol use does not fully describe his use. While he may have had worse issues in the past, characterizing his use as "previous alcohol dependence" minimizes his ongoing problem. His use could be described as "risky," as he had a DUI, but that is not the most accurate way to describe how it has affected his behavior in other ways. Being accurate about the diagnosis will be the best way to help him get the care he needs.

Mr. York is a 44-year-old male presenting for evaluation of an eyelid lesion. He noticed the lesion about one year ago. There is no associated itching, discharge, or other bothersome symptoms. Which of the following is the next best step in the management of the eyelid lesion? A. Low potency topical corticosteroid B. Measurement of serum cholesterol levels C. Measurement of serum uric acid levels D. No further management E. Skin biopsy

The correct answer is B. The eyelid lesion is most likely a xanthelasma associated with hyperlipidemia. Xanthelasma are cholesterol-filled, soft, yellow plaques which may appear on the medial aspect of the eyelid or on extensor surfaces. They are benign findings, and removal is typically only pursued for cosmetic reasons. Despite the benign nature of the lesion itself, measurement of serum cholesterol levels should be pursued to identify hyperlipidemia in patients with xanthelasma.

A 72-year-old male with a history of a right knee replacement 14 days ago returns to the emergency department with pain at the site of the knee replacement, and a new effusion. An arthrocentesis performed at his bedside reveals bloody fluid with low viscosity. Lab analysis reveals 5,000 white blood count (WBC)/mm with 30% polymorphonuclear cells (PMNs) and 900,000 red blood count (RBC)/mm in the fluid. What is the most-likely diagnosis? A. Gout B. Hemarthrosis C. Osteoarthritis D. Rheumatoid arthritis E. Septic arthritis

The correct answer is B. The fluid is most consistent with a hemarthrosis. A hemarthrosis will have red/bloody colored fluid that has RBCs >WBC with a variable percentage of PMNs. Septic arthritis will commonly have a WBC count >100,000 WBC/mm with >75% PMNs. A synovial fluid is consistent with an inflammatory condition (like rheumatoid arthritis or gout) if WBC count in the fluid is between 2,000-75,000 WBC/mm and >=50% PMNs. Synovial fluid is consistent with a non-inflammatory condition (like osteoarthritis) if WBC count in the fluid is between 200-2,000 WBC/mm and <25% PMNs.

A 50-year-old male with history of alcohol use disorder, type 2 diabetes mellitus, and cirrhosis presents to the emergency department with hematochezia and lightheadedness. He reports a large amount of painless bright-red rectal bleeding for the past one hour. He has not experienced this before. He denies vomiting. His home medications include metformin and propranolol. He drinks half a pint of vodka daily. On physical exam, his temperature is 37.3C (99.2F), pulse is 120 beats/minute, and blood pressure is 82/60 mmHg. His abdomen is soft and non-tender. He is jaundiced and has distended veins on his anterior abdominal wall. Laboratory testing reveals a hemoglobin of 8.8 mg/dL and platelet count of 69,000/µL. Two large-bore peripheral intravenous (IV) medications are placed, a fluid bolus is started, and a blood type and crossmatch is ordered. What diagnosis is the most likely cause of this patient's hematochezia? A. Alcoholic gastritis B. Esophageal varices C. Internal hemorrhoids D. Mallory-Weiss tear E. Meckel diverticulum

The correct answer is B. The most likely cause of bleeding in this patient with cirrhosis and hematochezia, is esophageal varices. In order to produce bright-red blood from an upper gastrointestinal (GI) source, the hemorrhage must be brisk enough for blood to pass through the intestines without oxidating. Because varices bleed at arterial pressure, they often (but not always) produce brisk hemorrhages and hemodynamic instability, as with this man. His medication list includes a non-selective beta-blocker, which may indicate a known diagnosis of varices. Gastritis does not typically cause large-volume bleeding. Internal hemorrhoids and Meckel diverticulum both could present with painless hematochezia, but hemorrhoidal bleeding is typically mild, and Meckel diverticula present more commonly in younger patients. Mallory-Weiss tears typically cause hematemesis after vomiting.

A 42-year-old male presents to the office with concerns of low back pain and lower extremity weakness. He has no significant medical history and his only medication is occasional ibuprofen for pain. He is very active and has recently joined a recreational golf league. This current pain has persisted since his last golf game. Physical exam reveals a fit male in no distress with vitals showing his pulse is 56 beats/minute, respiratory rate is 16 breaths/minute, and blood pressure is 110/75 mmHg. You are preparing to perform a neurologic examination to diagnose what is likely a radicular etiology of his back pain. Which combination of motor, sensory, and reflex neurologic deficits would be consistent with an L5-S1 radiculopathy? A. Big toe dorsiflexion difficulty, third metatarsal sensory deficits, and decreased medial hamstring change B. Foot eversion difficulty, lateral malleolus sensory deficits, and decreased Achilles reflex C. Foot eversion difficulty, medial malleolus sensory deficits, and decreased Achilles reflex D. Foot inversion difficulty, lateral malleolus sensory deficits, and decreased patellar reflex E. Foot inversion difficulty, medial malleolus sensory deficits, and decreased patellar reflex

The correct answer is B. The neurologic examination of back pain should include a motor and sensory evaluation. A Deep-tendon reflex assessment should also be performed if a radicular nerve compression at the lumbar level is suspected. For an L5-S1 radiculopathy, foot eversion difficulty, lateral malleolus sensory deficits, and a decreased Achilles reflex (B) would be present. An L3-L4 radiculopathy would result in foot inversion difficulty, medial malleolus sensory deficits, and a decreased patellar reflex (E). An L4-L5 radiculopathy would result in big toe dorsiflexion difficulty, third metatarsal sensory deficits, and decreased medial hamstring change (A). The constellations of symptoms described in answers (C) and (D) contain findings consistent with more than one nerve root involvement.

A 44-year-old male presents with a three-week history of nasal congestion, rhinorrhea, and watery eyes. He notes an occasional dry cough and mild sore throat as well. On physical examination, vital signs are within normal limits. Nasal exam reveals mild mucosal edema that is boggy and pale. Pharynx has a cobblestoning appearance. What is the next best step in management? A. Antibiotic therapy B. Antihistamine therapy C. Complete blood count D. Rapid Strep test E. Sinus computed tomography (CT)

The correct answer is B. The patient is demonstrating symptoms and signs of allergic rhinitis. The optimal management would be to proceed with antihistamine therapy. The timeframe and features of his symptoms and physical exam are not suggestive of a bacterial infection, so antibiotic therapy is not warranted. The patient has mild upper respiratory symptoms with normal vital signs and no fever, chills, or other systemic symptoms to suggest infection. Given the low likelihood of infection and his non-toxic presentation, a complete blood count (CBC) is not necessary. The patient does not have any of the clinical features of Group A Streptococcal pharyngitis, which typically include sudden onset of severe sore throat, fever, headache, nausea, vomiting, and abdominal pain. Additionally, the exam does not reveal oropharyngeal erythema or exudates, palatal petechiae, or tender anterior cervical adenopathy which are characteristic of this infection. Given the low likelihood of Group A Streptococcal pharyngitis, a rapid Strep test is not indicated. While the duration of the patient's upper respiratory infection (URI) symptoms is long, they are mild and non-progressive. The course for typical rhinovirus infections (cold infection) shows 50% are better in one week; 85% better at two weeks and 97% better at three weeks. The persistence of this patient's symptoms without worsening is inconsistent with a URI. A sinus CT may reveal sinus inflammation, but it would not help differentiate allergic versus infectious etiologies better than the history and physical exam and would expose the patient to unnecessary radiation.

A 32-year-old female presents to the office with fever, a petechial skin rash, and migrating joint pain for the past month. She states that her symptoms started a few days after having an unprotected sexual encounter while on vacation. Two weeks ago, she had pain and erythema in her right wrist, which spontaneously improved. She now has pain in her left knee with an associated effusion. You perform an arthrocentesis in the office, which shows the following: Straw-colored, cloudy fluid low viscosity 25,000 white blood count (WBC)/mm 80% polymorphonuclear cells (PMNs) Gram stain and crystal evaluation is pending. What is the most likely diagnosis? A. Acute rheumatic fever B. Gonococcal arthritis C. Osteoarthritis D. Reactive arthritis E. Staphylococcal arthritis

The correct answer is B. The patient presented with migratory polyarthralgia, fever, dermatitis, and tenosynovitis, consistent with a presentation of disseminated gonococcal infection. Arthrocentesis should not be delayed in patients who are presenting with a possible septic joint. The synovial fluid is consistent with inflammation secondary to gonococcal arthritis. If Staphylococcal or other non-gonococcal septic arthritis were considered, one would expect the WBC count in the synovial fluid to be >100,000 WBC/mm with >75% PMNs. Reactive arthritis is less common in women and normally presents with the triad of urethritis, conjunctivitis, and axial arthritis. The onset is commonly subacute, and fever is usually absent. Poststreptococcal arthritis (acute rheumatic fever) may also present with a (fleeting) rash and polyarthritis and have an inflammatory synovial fluid. However, this patient had no evidence of a recent streptococcal infection.

A 72-year-old male presents to the hospital with a two-day history of bright-red blood per rectum. He reports no weight loss, fevers, chills, nausea, vomiting, or diarrhea. He has a history of hypertension, diabetes, hyperlipidemia, and chronic constipation. Temperature is 37.1C (98.8F), pulse is 98 beats/minute, respiratory rate is 20 breaths/minute, and blood pressure is 132/94 mmHg. Abdominal examination reveals slight tenderness in the left-lower quadrant, without guarding or rebound. The remainder of the physical examination is normal. Which of the following is the most likely diagnosis for this patient? A. Colon cancer B. Diverticulosis C. Gastritis D. Inflammatory bowel disease E. Perforated colon

The correct answer is B. The patient presents with hematochezia and slight left-lower quadrant pain. The most likely diagnosis is diverticulosis. His history of chronic constipation increases his risk of diverticulosis, which can cause painless bleeding. Colon cancer is more likely to cause occult blood loss, rather than hematochezia. Gastritis is unlikely considering the presence of hematochezia rather than melena, and acuity of symptoms. The patient's age and acuity of symptoms make inflammatory bowel disease less likely. His exam did not show signs of acute abdomen (i.e. diffuse tenderness, rebound, and guarding) which would have been expected with a perforated colon.

A 45-year-old previously healthy female presents to the emergency department with right-lower extremity swelling for two days. She has no chest pain or shortness of breath. Her vital signs are normal. Her exam is notable for swelling of the right thigh and calf; the rest of her exam is normal. Duplex ultrasound of the right-lower extremity reveals a popliteal deep-vein thrombosis (DVT) with extension above the popliteal fossa. Upon further discussion with her, you learn that she is visiting from another state and plans to stay in this area for at least a week. She has health insurance through her employer. What is the most-appropriate anticoagulation in this case? A. Discharge from the emergency department (ED), initiate low-molecular-weight heparin (LMWH) and warfarin B. Discharge from the ED, initiate oral apixaban C. Discharge from the ED, initiate warfarin without LMWH D. Hospitalize overnight, initiate heparin infusion and warfarin E. Hospitalize overnight, initiate LMWH and warfarin

The correct answer is B. There is no need to hospitalize the patient, and therefore outpatient treatment options should be considered. A novel oral anticoagulant (NOAC) is the most-appropriate choice based on clinical trials and the most recent guidelines. The first dose should be given in the ED, and a test-claim run to be sure her insurance will pay for this medication.

A 52-year-old bisexual male presents with difficulty swallowing solid foods for two weeks. He has also experienced low-grade fevers, weight loss, and anorexia. He admits to not consistently using condoms. He reports no vomiting, melena, or hematochezia. He takes no medications and does not smoke or drink alcohol. On exam, his oropharynx is normal. He has mild epigastric tenderness and appears cachectic. Laboratory evaluation reveals the following: White blood cell count (WBC): 3.5K (4.5-11); Hemoglobin: 10.2 gm/dl (12-15 gm/dl); Platelet count: 136K (150-420); Human immunodeficiency virus (HIV): PositiveCD4+ T-cell count: 188 cells/ml. An esophagogastroduodenoscopy (EGD) is ordered. What is the most likely finding on the EGD? A. Barrett's esophagus B. Esophageal candidiasis C. Gastritis D. Non-bleeding duodenal ulcer E. Normal

The correct answer is B. This is a male with newly diagnosed HIV and a CD4 count less than 200 cells/mm3 presenting with dysphagia to solid foods. Candidal esophagitis is the most likely diagnosis considering his HIV status, low CD4 count, and presenting symptoms. Candida esophagitis is considered an acquired immune deficiency syndrome (AIDS)-defining illness. It will be unlikely for him to have a normal EGD. The acuity of his presentation, lack of risk factors, and lack of associated symptoms, such as gastrointestinal (GI) bleeding or epigastric pain, make gastritis, Barrett's esophagus, or a duodenal ulcer much less likely.

A 28-year-old fair-skinned male presents with a skin lesion on his left upper arm that he is concerned about. The lesion has been present for as long as he can remember, however, just recently it has been changing in size and color. The lesion does not itch or cause pain. On physical exam, you note an 8 mm round symmetrical patch with areas of black. The lesion is flat without crusting or ulceration. What is the next step in his management? A. Deep-shave biopsy B. Excisional biopsy C. Monitoring of the lesion with follow-up in six months D. Punch biopsy E. Superficial shave biopsy

The correct answer is B. This lesion is very concerning for melanoma based on a diameter >6 mm, color variation, and its evolving character. The biopsy of choice for this lesion is an excisional biopsy in which the entire lesion is removed. Punch biopsies can be used to diagnose very small lesions that are concerning for melanoma (less than 4 mm in diameter), but not larger lesions, as the entire lesion needs to be removed. Shave biopsies are never indicated for suspected melanoma. A deep-shave biopsy can be considered with inclusion of the dermis and epidermis in suspected basal cell and squamous cell carcinomas. It is NOT appropriate to monitor lesions which are concerning for melanoma, as the survival rates decrease as the disease stage increases with time.

You are seeing a female in the emergency department with dyspnea and left-leg swelling. She is 48 years old with a history of deep-vein thrombosis (DVT) and pulmonary embolism (PE) three years ago. At that time, she was anticoagulated on warfarin for six months, and then it was discontinued. She reports having no fever, chills, or productive cough. On exam, her pulse is 110 beats/minute, respiratory rate is 24 breaths/minute, and the other vital signs are normal. Heart and lung exams are normal aside from tachycardia and tachypnea. There is swelling and tenderness of the left calf. What is the next appropriate diagnostic test? A. Chest radiograph B. Computed tomography angiogram (CTA) of chest C. High sensitivity D-dimer D. Lower extremity duplex ultrasound E. Ventilation-perfusion (V/Q) scan

The correct answer is B. This patient has a Wells Score >7 (prior DVT, HR >100, clinical signs of DVT, and alternative diagnoses less likely than PE) resulting in a high probability that she has a PE. CTA is the diagnostic test of choice in this case. Chest radiograph is non-diagnostic for PE, and given the high probability of PE in this case, moving directly to CTA of the chest is the most-appropriate next step. D-dimer is appropriate for patients with a low probability of PE, because it can effectively rule out PE/DVT. A lower extremity (LE) duplex ultrasound doesn't diagnose PE or give information about right strain. V/Q scan is cumbersome, more expensive, and less sensitive than CTA.

A 49-year-old female presents to the emergency department with a one-day history of bright-red blood per rectum. She reports no fevers, abdominal pain, or lightheadedness. Her medical history is significant for alcohol abuse and cirrhosis. She takes no medications. Her pulse is 90 beats/minute and blood pressure is 108/64 mmHg. On physical exam, she has bilateral lower-extremity edema and ascites. After fluid resuscitation and laboratory studies, which of the following is the most appropriate diagnostic study? A. Capsule endoscopy B. Colonoscopy C. Esophagogastroduodenoscopy (EGD) D. Radionuclide imaging E. Upper gastrointestinal (UGI) with small bowel follow-through

The correct answer is B. This patient has cirrhosis, which increases portal pressure and can cause mild to severe bleeding from hemorrhoids. Although patients with cirrhosis are at risk for bleeding from esophageal varicose or portal hypertensive gastropathy, an upper GI bleed would have to be very brisk to cause bright-red blood per rectum, rather than melena. Her hemodynamic stability argues against an upper GI source for her bleeding. Bright-red blood per rectum is usually from a lower GI bleed. An EGD would be indicated immediately if the source of bleeding was more likely from an upper GI source. Once her lower GI bleed is addressed, an EGD should be performed electively to screen for esophageal varices, given her known cirrhosis. Radionuclide imaging (e.g. tagged RBC scan) can be used to localize the source of bleeding, but is not as specific as a colonoscopy. Both an upper GI series with a small bowel follow-through and a capsule endoscopy are useful for evaluating the length of small intestine that is not visualized with EGD or colonoscopy. However, they are generally reserved for patients whose initial endoscopic evaluation is negative.

A 55-year-old male with hypertension and type II diabetes mellitus presents to the office for a routine follow-up. He has a history of bladder cancer that was treated by resection and chemotherapy five years ago. He is currently asymptomatic. He takes lisinopril, aspirin, and metformin. His pulse is 76 beats/minute, blood pressure is 128/78 mmHg, and body mass index (BMI) is 29 kg/m2. The rest of his physical exam is unremarkable. Laboratory studies reveal: Glucose 142 mg/dL Urinalysis: Negative leukocyte esterase, negative nitrite, negative bili, negative blood, trace protein Microalbumin 275 mcg/g Lipid Profile: Total cholesterol - 295, HDL - 35, LDL - 220, Triglycerides - 200 HgbA1c 7.2% Which of the following is the most appropriate medication to initiate in this patient? A. Amlodipine B. Atorvastatin C. Losartan D. Niacin E. Pioglitazone

The correct answer is B. This patient has elevated lipid levels, and the American Diabetes Association (ADA), American Heart Association (AHA), and American College of Cardiology (ACA) guidelines indicate statin therapy for all patients who are 40-75 years old with diabetes mellitus. Additional antihypertensives (e.g. amlodipine or losartan) are not the best choices for two reasons. First, his blood pressure is within the target blood pressure for patients with diabetes (<130/<80), thus additional blood pressure lowering is not needed. In addition, losartan (an angiotensin receptor blocker) and lisinopril (an angiotensin converting enzyme inhibitor) should not be used together, as these medications in combination carry a high risk of adverse effects such as hyperkalemia and hypotension. Niacin is not appropriate, as the AHA/ACC guidelines discourage the use of non-statin medications in the management of dyslipidemia given the lack of evidence that they reduce morbidity or mortality. Pioglitazone is not appropriate, as his blood sugar control is acceptable, and pioglitazone in particular is contraindicated in patients with a history of bladder cancer.

You are taking care of a 63-year-old female on the internal medicine floor who was recently diagnosed with cirrhosis. She was admitted with hepatic encephalopathy which has improved with lactulose. Today when you see her, she notes that she has been feeling more fatigued and weak with some light-headedness upon standing. On exam, she is afebrile. Her pulse is 122 beats/minutes (increased from 80s previously), and her blood pressure is 90/58 mmHg (near baseline since admission). She is breathing comfortably with a normal oxygen saturation. Her cardiac exam is normal with the exception of regular tachycardia. Her abdomen is moderately distended with a positive fluid wave, non-tender, and with normal bowel sounds. She has bilateral 2+ pitting edema to the mid-thigh. What is the most appropriate next step in diagnosis? A. Blood cultures B. Complete blood count (CBC) C. Esophagogastroduodenoscopy (EGD) D. Liver biopsy E. Paracentesis

The correct answer is B. This patient has known cirrhosis with new non-specific symptoms of fatigue and weakness. She has notable baseline hypotension, which is common in cirrhotic patients, but has new tachycardia. Given the new symptoms and sinus tachycardia, she should be evaluated for possible acute blood loss from variceal bleeding which is seen in 25% to 50% of patients with cirrhosis. Bleeding varices often present suddenly with hematemesis and patients can have severe blood loss leading to a high mortality rate. In this case, the presentation is not as severe. If her hemoglobin has decreased acutely, or if she shows signs of gross blood loss (hematemesis, melena, or hematochezia), an EGD should be performed. If the CBC is stable, and there is no concern for active GI bleed, subsequent evaluation could include paracentesis to rule out spontaneous bacterial peritonitis (SBP) in this patient with cirrhosis and any new signs of possible decompensation, which could be due to sepsis. It is reasonable to perform a paracentesis, even if the patient is afebrile. Other indications for paracentesis, in this case, would be acute renal failure or new abdominal pain. A liver biopsy may be indicated for evaluation of the cause of cirrhosis, but in this patient who is acutely ill it is unlikely to help with initial diagnosis or stabilization. Blood cultures can be performed, as there is concern for sepsis given her tachycardia, but a CBC should be the initial step in the evaluation.

An 84-year-old female comes to the clinic with concern for fatigue for the past six months. Otherwise, a review of systems is negative. On physical examination, her vital signs show pulse is 99 beats/minute, blood pressure is 122/76 mmHg, oxygen saturation is 98% on room air. She appears slightly pale, but is otherwise comfortable. Cardiovascular and pulmonary examination is otherwise normal. Her complete blood count (CBC) is as follows: White blood cell (WBC): 3.2 cells x 103/μL Hemoglobin: 7.5 g/dL Hematocrit (Hct): 22% Mean corpuscular volume (MCV): 84 μm3 Red blood cell distribution width (RDW): 20 fLPlatelet (Plt): 128,000 mm3 Given your concern for a chronic normocytic anemia, you obtain a reticulocyte count, which is 0.05%. What is the next-best diagnostic test at this time? A. Esophagogastroduodenoscopy (EGD) B. Iron studies C. Lactate dehydrogenase (LDH) (serum) D. Peripheral smear E. Thyroid stimulating hormone (TSH)

The correct answer is D. This female has normocytic anemia. Given the duration of her symptoms, this is most likely chronic. The reticulocyte count is helpful to differentiate between hypoproliferative and hyperproliferative disorders. Her reticulocyte count is low. Since she is also anemic, it is helpful to calculate the reticulocyte production index (RPI) to assess whether the bone marrow is responding appropriately to the anemia. The following formula helps to calculate the RPI: RPI = (% retic x patient's hematocrit) ÷ (normal hematocrit x reticulocyte maturation time) This patient's RPI= (.05)(22)/(45)(2.5)=.01. The reticulocyte maturation time varies depending on the degree of anemia. An RPI < 2 in the setting of anemia is inappropriately low. This indicates that there is decreased red cell production. The most likely cause of this is a primary bone marrow disorder, such as myelodysplasia, bone marrow infiltration, myeloma, or aplastic anemia. The next step in diagnosis would be to order a peripheral smear. The peripheral smear can give clues to the etiology of the anemia and focus the evaluation. LDH would be ordered if there were concerns for hemolysis on peripheral smear. The anemia is normocytic, which makes iron deficiency less likely. A TSH would be reasonable given her fatigue, but it is unlikely that hypothyroidism is the cause of the anemia of this magnitude in a post-menopausal female.

A 56-year-old female was diagnosed with left-lower-lobe pneumonia. She was hospitalized and has completed four days of ceftriaxone and azithromycin. She has a history of congestive heart failure (CHF), but was euvolemic on admission. You are evaluating her on hospital day five. She continues to complain of cough and dyspnea. Vital signs show temperature is 38.0 C (100.4 F), pulse is 90 beats/minute, respiratory rate is 24 breaths/minute, blood pressure is 122/78 mmHg, and oxygen saturation is 92% on four liters/minute. Her exam is significant for faint crackles and decreased breath sounds in the lower-left-lung fields. Her jugular venous pressure (JVP) is approximately 7 cm, and there is a trace of pedal edema bilaterally. A chest radiograph shows a stable left-lower-lobe infiltrate with a new, moderate left-pleural effusion. Which of the following is the appropriate next step in management? A. Chest computed tomography (CT) B. Diagnostic thoracentesis C. Electrocardiogram D. Repeat blood and urine cultures E. Transthoracic echocardiogram

The correct answer is B. This patient has pneumonia with no improvement (persistent fever, tachypnea) in symptoms after four days of appropriate antibiotic therapy. A repeat chest radiograph shows a new pleural effusion. The differential diagnosis includes a parapneumonic effusion or an effusion related to heart failure, with a parapneumonic effusion being more likely because she is not volume overloaded on exam. A diagnostic thoracentesis should be performed as soon as feasible. Chest CT, electrocardiogram, cultures and an echo are not indicated at least until the pleural effusion is evaluated.

A 65-year-old male presents to the emergency department with dyspnea, cough, and fever for one week. His past medical history is significant for hypertension. On exam, his temperature is 38.5 C (101.3 F), pulse is 100 beats/minute, respiratory rate is 24 breaths/minute, blood pressure is 140/80 mmHg, and oxygen saturation is 88% on room air, increasing to 92% on four liters of oxygen by nasal cannula. The patient appears uncomfortable and is using accessory muscles of respiration. Breath sounds are decreased on the left side with dullness to percussion on the left. Laboratory values include a white blood cell (WBC) count of 15 cells x 103/μL hematocrit (Hct) of 45%, platelet count of 150,000/µL, and an international normalized ratio (INR) of 1.1. Chest radiograph shows a large left-sided pleural effusion. He is given broad-spectrum antibiotics. Which of the following is the most appropriate next step in management? A. Computed tomography (CT) scan of the chest B. Diagnostic and therapeutic thoracentesis C. Echocardiogram D. Intubation and mechanical ventilation E. Placement of a chest tube

The correct answer is B. This patient is experiencing hypoxia, tachypnea, and respiratory distress and has a large left-sided pleural effusion, which is concerning for possible empyema as a source of sepsis. The most appropriate next step is a diagnostic and therapeutic thoracentesis to relieve his symptoms and aid in diagnosis. If the pleural fluid is infected (empyema), then a chest tube would be subsequently indicated. A CT scan of the chest may aid in diagnosis, but the man will tolerate it more comfortably after a thoracentesis. An echocardiogram may be indicated if the pleural fluid is consistent with transudate. Intubation and mechanical ventilation is a clinical decision based on many factors. Nothing in the stem indicates that he must be intubated now.

A 39-year-old male with no significant past medical history presents to the hospital with lower abdominal pain for the past two days. His initial vitals are notable where his temperature is 38.5 C (101.3 F), pulse is 112 beats/minute, and blood pressure is 103/68 mmHg. He reports that his last bowel movement was two days ago, and he recalls passing flatus earlier today. Your physical exam reveals hypoactive bowel sounds, and a diffusely tender abdomen, most tender in the right-lower quadrant. His abdominal muscles are tense, and do not relax with distraction techniques. A computed tomography (CT) scan of the abdomen and pelvis confirms your suspected diagnosis. You order broad-spectrum intravenous (IV) antibiotics. What is the best next step in management? A. Call a gastrointestinal (GI) consult for colonoscopy B. Call a surgical consult C. Order a bowel regimen D. Place a nasogastric tube for decompression E. Start IV steroids

The correct answer is B. This patient is febrile and tachycardic, with right-lower quadrant pain, suggestive of appendicitis. He has rigidity on exam, which suggests peritonitis. Once broad-spectrum antibiotics that cover enteric organisms have been ordered, the best next step from the given options is surgical consultation. Colonoscopy is contraindicated in this case, as the intraluminal pressure from the procedure can cause perforation when there is active infection or inflammation in the colon. A bowel regimen is appropriate for patients with constipation, but this patient has an acute abdomen. Nasogastric decompression is done for small-bowel obstruction. Typically, a patient with a small-bowel obstruction will be nauseated and vomiting, and would not be expected to be febrile. Steroids are indicated in an acute flare of inflammatory bowel disease, which will typically present with loose stool, sometimes bloody. While fever can be present, the symptoms will not usually present so abruptly.

A 45-year-old female with stage 3 chronic kidney disease, secondary to lupus nephritis, presents to the emergency department with confusion. Her husband reports that she has been complaining of nausea and itching for several weeks. Diphenhydramine has not alleviated the itching. Her physical exam is significant for lethargy, orientation to person only, excoriations on all of her extremities, a pericardial friction rub, asterixis, crackles at the lower lung fields, and pitting edema of the lower extremities to the knees. Her labs are significant for sodium 135mEq/L, potassium 5.5mEq/L, creatinine 5mg/dL [creatinine was 2.5mg/dL six months ago], HCO3 20 mEq/L, and an anion gap of 14. What is the pathophysiological mechanism of her acute state? A. Decreased cardiac output from non-ischemic cardiomyopathy B. Inability of the kidney to excrete organic waste products C. Inability of the liver to excrete nitrogenous waste products D. Ingestion of ethanol, leading to accumulation of ketones E. Poor oral intake, leading to accumulation of ketones

The correct answer is B. This patient is presenting with a constellation of symptoms and findings consistent with uremia. Given the chronicity of her nausea and itching, her kidneys have been declining over several months. Uremia is caused by the kidneys' inability to excrete organic waste products. While poor oral intake could result in a starvation ketosis, which would worsen her anion gap metabolic acidosis, it would not explain her other symptoms of itching, friction rub, or asterixis. Hepatic encephalopathy, which occurs with liver failure, is caused by the liver's inability to excrete nitrogenous waste products, however, she has no signs of advanced liver disease (jaundice, ascites, spider angiomata). Ingestion of too much ethanol can cause a ketosis, which would worsen her anion gap metabolic acidosis, does not explain her other symptoms. Non-ischemic cardiomyopathy would explain the volume overload, however, it would not explain her neurological symptoms.

You are working in the emergency department, and your attending asks you to see a 65-year-old male presenting with abdominal pain. In gathering the history, the patient tells you that he has had two to three days of progressive generalized "achy" abdominal pain, along with decreased appetite and decreased urine output. On further clarification, he reports not urinating in the past two days. On exam, vital signs are normal. He is alert, oriented, and in no acute distress. His exam is benign with the exception of mild abdominal distention, a palpable bladder, and an enlarged prostate on digital rectal exam. Which of the following is the next best step? A. Computed tomography (CT) of his abdomen and pelvis B. Foley catheter placement C. Intravenous (IV) diuretics D. IV fluids E. Renal ultrasound

The correct answer is B. This patient is presenting with symptoms of post-renal obstruction. The urgent next step is foley catheter placement to relieve the obstruction, which can be both diagnostic and therapeutic. Alternatively, a bedside bladder scan can be performed first to confirm the enlarged bladder (many bedside nurses can do this technique), before catheter placement. The patient's enlarged prostate increases the likelihood of bladder outlet obstruction. Post-obstructive renal failure is a "can't miss" diagnosis, as longer obstruction time leads to an increased incidence of acute kidney injury, infection, and urosepsis. Prompt relief of the obstruction is important. A urologic CT scan (without contrast) is currently the imaging test of choice due to its availability, however magnetic resonance imaging (MRI) has very high sensitivity and specificity for identifying the obstruction and may also be able to differentiate acute versus chronic renal failure. Its main limitations are cost and accessibility. Imaging is also indicated for patients with presumed obstruction, however imaging would be for diagnostic purposes only. This patient is presenting with signs and symptoms of a urinary obstruction. IV fluids are indicated in pre-renal acute kidney injury, however, would not treat this patient's most-likely underlying pathology. In addition, diuretics (to promote urine output) would be contraindicated in a complete obstruction, and would not be recommended in this setting without any additional information about the underlying etiology of his symptoms. Renal ultrasound is a quick, safe, and relatively inexpensive way to evaluate for hydronephrosis, but is limited in evaluating the entire renal system, as ureters cannot be visualized.

A 58-year-old female with a history of hypertension and hyperlipidemia on chlorthalidone and simvastatin presents to her physician with concerns of fatigue and constipation. Physical exam is significant for dry mucous membranes, her pulse is 105 beats/minute, and there is hard stool in the rectal vault. Her blood pressure is normal. Labs are significant for: glucose 100 mg/dL, sodium 140mEq/L, potassium 4.0mEq/L, calcium 12.0 mg/dL, thyroid-stimulating hormone (TSH) 3.5 mU/L, and parathyroid hormone (PTH) 18 pg/mL. What is the most likely cause of this patient's hypercalcemia? A. Adrenal insufficiency B. Chlorthalidone C. Hyperparathyroidism D. Hyperthyroidism E. Metastasis to the bone

The correct answer is B. This patient is taking chlorthalidone, which can mildly increase calcium levels due to increased absorption of calcium in the distal convoluted tubule of the nephron in exchange for increased sodium excretion. This patient has low parathyroid hormone levels, which argues against hyperparathyroidism. Her blood pressure, sodium, potassium, and glucose are normal, which makes adrenal insufficiency unlikely. If she had cancer so advanced to have metastasized to the bone, she would most likely have bone pain, weight loss, or other sequelae. Her TSH is normal, making hyperthyroidism unlikely.

A 30-year-old female presents with an acute exacerbation of chronic lower back pain that has been present for the past six years. She reports no recent fevers or weight changes. She is also treated for generalized anxiety disorder but has no other chronic conditions. She takes ibuprofen for her pain and escitalopram for anxiety. Temperature is 37 C (98.6 F), pulse is 78 beats/minute, respiratory rate is 14 breaths/minute, blood pressure is 126/78 mmHg. Physical examination reveals a limitation of flexion and extension of the lumbar spine. Straight-leg raise test is negative. Her motor strength and sensory exam in both low extremities are normal. Her Achilles and patellar reflexes are 2+ bilaterally. The remainder of her examination is normal. Into which of the following diagnostic categories is her back pain most likely to fall? A. Infectious etiology B. Inflammatory process C. Metastatic malignancy D. Neurodegenerative disorder E. Psychiatric condition

The correct answer is B. This patient most likely has a seronegative spondyloarthropathy, ankylosing spondylitis, which is the result of an underlying inflammatory process (B). The key findings which support this diagnosis are onset prior to age of 40 years old, the chronicity of her back pain, and the limitation of flexion and extension of the lumbar spine on physical examination. She reports no fevers and is afebrile with normal vital signs on presentation. She is not currently prescribed any medications that would suppress her immune response, so her risk of atypical, more-indolent and chronic infections (A) is low. This patient has no prior history of cancer and no weight changes or other B symptoms suggestive of metastatic malignancy (C). Additionally, her younger age lowers her risk of a primary malignancy. Most neurodegenerative disorders (D) present with focal neurologic deficits which are not present on this patient's examination, so such a process is very unlikely. Psychiatric disorders, particularly somatiform disorders, can present with pain, but generally the exam has no objective abnormalities in such cases. Attributing back pain to a psychiatric condition (E) should only be considered when a thorough clinical evaluation has been completed and excludes all other possible etiologies, which is not the case here.

A 36-year-old female presents to the emergency department with a one-day history of nausea, dysuria, and right-sided flank pain. She was previously healthy. Temperature is 39.3C (102.7F), pulse is 125 beats/minute, respiratory rate is 20 breaths/minute, and blood pressure is 78/50 mmHg. She receives four liters of normal saline and is started on intravenous (IV) vasopressors and antibiotics. A repeat blood pressure is 88/52 mmHg. What type of shock does this patient most likely have? A. Cardiogenic B. Distributive C. Hypovolemic D. Multifactorial E. Obstructive

The correct answer is B. This patient most likely has sepsis from pyelonephritis (fever, dysuria, flank pain), and septic (distributive) shock is the most likely-diagnosis. Hypovolemic shock is less likely, considering her lack of adequate response to IV fluids, and the need for the addition of vasopressors. In hypovolemic shock, patients have low cardiac output, high systemic vascular resistance, and low cardiac filling pressures. Patients may present with a history of bleeding or volume depletion from other causes such as diarrhea. There are no symptoms of pulmonary embolism, tension pneumothorax, or cardiac tamponade to support a diagnosis of obstructive shock. Patients with obstructive shock usually present with hypotension, tachycardia, and low cardiac output.

A 32-year-old male with type 1 diabetes mellitus presents to the emergency department. He has had recent suicidal ideation and recent admissions for diabetic ketoacidosis. He has a suspected, but not proven, ethylene glycol overdose, as his friend found an empty bottle of antifreeze in his house. His arterial pH is 7.25, his serum bicarbonate level is 14 mEq/L, and his osmolar gap is 12 mOsm/kg H2O. The ethylene glycol level is pending. What should be the immediate next step? A. Consult nephrology to initiate hemodialysis B. Order fomepizole C. Send urine to evaluate for calcium oxalate crystals D. Start an intravenous bicarbonate continuous infusion E. Start an intravenous insulin drip

The correct answer is B. Fomepizole (B) is a competitive inhibitor of alcohol dehydrogenase that helps prevent the metabolism of methanol and ethylene glycol to their toxic metabolites. It is a generally well tolerated antidote for both methanol and ethylene glycol poisoning. The American Academy of Clinical Toxicology has defined criteria for starting fomepizole as: Documented plasma ethylene glycol concentration >20 mg/dL (3 mmol/L) or documented recent (hours) history of ingesting toxic amounts of ethylene glycol and osmolar gap >10 mOsm/kg H2O or history or strong clinical suspicion of ethylene glycol poisoning and at least three of the following criteria: i) Arterial pH less than 7.3 ii) Serum bicarbonate level <20 mEq/L (20 mmol/L) iii) Osmolar gap >10 mOsm/kg H2O iv) Urinary oxalate crystals present. Sending urine to evaluate for oxalate crystals (C) may be useful to help support the diagnosis of ethylene glycol ingestion. Since there is already suspicion of ingestion and the patient meets three of the four criteria for fomepizole treatment, it is not appropriate to withhold treatment while waiting for this additional lab result. Intravenous sodium bicarbonate (D) may temporarily correct his metabolic acidosis, but it does not address the primary mechanism for it as fomepizole will. Hemodialysis (A) is a potential treatment option for ethylene glycol ingestion, but is typically reserved as first-line therapy for patients with a very severe metabolic acidosis with serum bicarbonate level <10 meq/L, osmolar gap >20 mOsm/kg H2O, or evidence of end organ damage. An intravenous insulin drip (E) is appropriate therapy for diabetic ketoacidosis (DKA), but the finding of an osmolar gap of greater than 10 mOsm/kg H2O is not consistent with a diagnosis of DKA.

A 24-year-old female presents to the emergency department with 18 hours of lower abdominal pain, nausea, vomiting, and diarrhea. She is afebrile and has normal vital signs except for tachycardia with a pulse at 105 beats/minute. Her abdomen is soft and non-distended with hypoactive bowel sounds. There is rebound tenderness in the left-lower quadrant. The abdominal exam finding of rebound tenderness has +LR = 2 and -LR = 0.4 for peritonitis. What is the best interpretation of this finding? A. Absence of rebound tenderness strongly argues against peritonitis B. Absence of rebound tenderness strongly argues against peritonitis, but presence of it does not affect the probability of peritonitis C. Neither the presence nor the absence of rebound tenderness is very helpful in diagnosing peritonitis D. Presence of rebound tenderness strongly suggests peritonitis E. Presence of rebound tenderness strongly suggests peritonitis, but absence of it does not affect the probability of peritonitis

The correct answer is C. +LR of 2-5 is weak evidence, 5.1-10 is moderate evidence, and > 10 is strong evidence in favor of a disease. -LR of 0.51-1.0 is weak evidence, 0.11- 0.5 is moderate evidence, and < 0.1 is strong evidence against a disease. As rebound tenderness has a +LR = 2 and a -LR = 0.04, both the presence of, and the absence of rebound tenderness provide weak evidence in diagnosing peritonitis.

A 51-year-old male is admitted to the hospital for acute, uncomplicated diverticulitis. Medical history is significant for asthma, gastroesophageal reflux disease (GERD), and hypertension. He is treated with intravenous ciprofloxacin and metronidazole, intravenous fluids, and pain medication. After two days, his pain is improved and he is tolerating food. Which of the following discharge plans is most appropriate for him? A. Continue oral antibiotics for 21 days B. Prescribe subcutaneous enoxaparin for 28 days C. Refer to gastroenterology for a colonoscopy D. Refer to a nutritionist to initiate a gluten-free diet E. Refer to surgery for resection of the affected bowel

The correct answer is C. A colonoscopy should be performed four to six weeks after recovery to explore the full extent of diverticulosis and rule out polyps or cancer. If he has recurrent episodes of diverticulitis, he may need to see a surgeon to remove a section of the diseased bowel, but following a first episode of uncomplicated diverticulitis, surgery referral would not be warranted. A gluten-free diet is indicated for celiac disease, not for diverticulosis. A high-fiber diet, however, can decrease constipation and intraluminal pressure, and may decrease the chance of recurrent diverticulitis. A seven- to 14-day course of antibiotics is recommended for uncomplicated diverticulitis. Extended exposure to antibiotics is not recommended as it can cause antibiotic-associated diarrhea or Clostridium difficile ( C. difficile ) colitis. Enoxaparin for deep vein thrombosis (DVT) prophylaxis is indicated while a patient is in the hospital and acutely ill, but not upon discharge.

A 32-year-old female with no known medical history presents with two weeks of pain in her right knee, fever, and an associated effusion. You evaluate her and plan to perform an arthrocentesis to further elucidate the cause of the effusion. You review the indications for the arthrocentesis, but the patient asks you if there are any risks. Which of the following, if any, is a risk of an arthrocentesis? A. Dry tap B. Introduction of calcium pyrophosphate crystals into the joint C. Introduction of infection into the joint D. None E. Radiation exposure from ultrasound guidance

The correct answer is C. Although a sterile technique should be used when performing an arthrocentesis, there is still a risk of introducing bacterial infection into the joint. Other potential risks of arthrocentesis include tendon or ligament injury, bleeding into the joint (hemarthrosis), and allergic reaction to the local anesthetic. A dry tap is when no fluid is yielded upon arthrocentesis, despite appearance of fluid on physical examination. A dry tap can occur with arthrocentesis, but is not a risk per se; it may be caused by fluid that is too viscous; blockage of the needle by fat, debris, or other; or an inaccurate physical examination. Calcium pyrophosphate crystals are formed endogenously in the joint in pseudo gout, but they cannot be introduced by the arthrocentesis. Ultrasound can be used to guide the arthrocentesis needle, but ultrasound does not produce radiation. There are always risks with any invasive procedure.

A 63-year-old female with a past history of hypertension, chronic obstructive pulmonary disease (COPD), type 2 diabetes mellitus, and hyperlipidemia is admitted to the hospital with Clostridium difficile (C. difficile) colitis. Of the following options, which is the best choice for deep vein thrombosis (DVT) prophylaxis for her? A. Ambulation B. Aspirin, 81 mg by mouth (PO) daily C. Enoxaparin, 40 mg subcutaneous daily D. Heparin intravenous (IV) continuous infusion, titrated for a goal-activated partial thromboplastin time (aPTT) of 50 to 70 seconds E. Intermittent pneumatic compression (IPC) device

The correct answer is C. Anticoagulation with a low-molecular-weight heparin, unfractionated heparin, or fondaparinux, is effective in reducing the risk of DVT. The doses are smaller for prophylaxis than for treatment of a known venous thromboembolism (VTE). Enoxaparin of 40 mg subcutaneous daily or unfractionated heparin of 5000 units subcutaneous every eight hours (Q8H) would be appropriate choices for DVT prophylaxis. Ambulation is not as effective as pharmacologic anticoagulation in preventing DVT; in addition, it cannot be relied upon for an acutely ill patient who may not be feeling well enough to frequently ambulate. Aspirin is not effective for DVT prevention. A heparin infusion is used for full anticoagulation (treatment), not for prophylaxis. IPC devices often have limited effect, as they are often removed or disconnected when patients get out of bed. These should only be ordered for patients with a contraindication to pharmacologic prophylaxis (e.g. active bleeding).

A 26-year-old male presents to the emergency department with a three-day history of chest pain. The pain is described as sharp, left-sided, and exacerbated by breathing. He reports having no exertional dyspnea. He is previously healthy. Vital signs are normal. He has no tenderness with palpation of the chest wall over the area where he has pain. There is 1+ pitting edema in the right leg. The remainder of the examination is normal. What is the next most-appropriate test to determine the probability of thromboembolism in this patient? A. Chest radiograph B. Computed tomography angiogram (CTA) of the chest C. High sensitivity D-dimer D. Lower-extremity (LE) duplex ultrasound E. Ventilation-perfusion (V/Q) scan

The correct answer is C. D-dimer is the appropriate study because, given the current history and exam findings, he has a low probability of a pulmonary embolism (PE). D-dimer is a sensitive test that can effectively rule out deep-vein thrombosis (DVT) or PE in this case. If he did not have the unilateral leg edema, he would have scored zero on a Pulmonary Embolism Rule-out Criteria (PERC) score and could be ruled out for DVT/PE without a d-dimer. Using PERC and Wells' scores, he has a low probability of DVT, so a d-dimer is the best option. This test is the least-expensive option and doesn't involve radiation exposure which both the chest x-ray (CXR) and especially CTA, involve.

A patient is admitted to the hospital for suspected bacterial meningitis. A lumbar puncture is performed, and the results are pending. While awaiting the cerebrospinal fluid (CSF) results, which of the following types of isolation would be the most appropriate for this patient? A. Airborne isolation B. Contact isolation C. Droplet precautions D. No isolation E. Standard precautions

The correct answer is C. For a patient with suspected bacterial meningitis, droplet precautions are the most appropriate type of isolation while awaiting CSF results. Because Neisseria meningitidis is transmitted by respiratory droplets, the patient should be maintained in droplet precautions until infection with this organism is ruled out. Droplet precautions, in general, are used for any infection that is potentially transmissible through close contact with respiratory or mucous membrane secretions. Examples include respiratory viruses such as influenza and adenovirus; bacterial infections including pertussis and Mycoplasma pneumoniae infection. These pathogens do not remain infectious over long distances, and thus special air handling and ventilation systems are not required.

A 55-year-old male is seen in the emergency department for acute abdominal pain. He is lying motionless when you interview him. On exam, there are minimal bowel sounds. He has diffuse abdominal tenderness. His abdomen is hard when you palpate it, and he is unable to relax his abdominal muscles after you warm up your hands, distract him, and palpate gently. When you accidentally bump the stretcher, he cries out in pain. What type of pain is this patient experiencing? A. Ischemic pain from celiac artery occlusion B. Obstruction of a hollow viscera C. Parietal pain from inflammation of the parietal peritoneum D. Referred abdominal pain from an inferiorly located pneumonia E. Somatic body-wall pain from an abdominal wall muscle injury

The correct answer is C. Hallmarks of parietal peritoneal pain include rigidity and lying still, as this is the position of maximal comfort. Rigidity refers to a tensing of the abdominal muscles overlying the inflamed peritoneum. Rigidity is sometimes referred to as "involuntary guarding." Often rebound tenderness is present with peritonitis, but it adds little diagnostic value to an exam (due to its non-specificity) and can be unnecessarily painful for the patient. Patients with peritonitis may complain of pain with coughing, sneezing, or with movement of the bed. Ischemic pain presents as pain out of proportion to exam findings (in other words, on exam, the patient does not seem as tender as you expect). Obstruction from a hollow organ usually causes patients to have colicky pain, and patients will move around trying to find a comfortable position. Referred pain from pneumonia would not cause a rigid abdomen. Somatic body-wall pain is usually well localized, and while it can be very painful, the abdomen should not be rigid.

A female with a severe headache undergoes a lumbar puncture, and red blood cells (RBCs) are found in the cerebrospinal fluid (CSF) specimen. Which of the following additional findings would best support the hypothesis that a traumatic tap caused the presence of RBCs in the CSF specimen? A. CSF glucose lower than serum glucose B. Elevated protein in the CSF C. Fewer RBCs in each subsequent tube of CSF fluid analyzed D. Positive Gram stain of the CSF E. Presence of xanthochromia in the CSF fluid

The correct answer is C. In distinguishing a "traumatic tap" from a true intracerebral hemorrhage, you should look for progressive clearing of RBCs with each tube of CSF sent for analysis. The absence of xanthochromia in the CSF fluid is also a helpful finding for excluding intracerebral bleeding. While a low CSF glucose (hypoglycorrhachia) and positive Gram stain argue for infectious meningitis, these two findings are not adequate to exclude an intracranial hemorrhage as the cause of blood in the CSF. Elevated protein may be present with a traumatic tap, but it is non-specific and associated with many other conditions (e.g., bacterial and viral meningitis, and inflammatory conditions).

Patients with chronic hepatitis C virus (HCV) infection are at risk of fulminant hepatic failure, if co-infection with hepatitis A virus (HAV) occurs. Which if the following factors poses the greatest risk for infection with HAV? A. Blood transfusion B. Consumption of undercooked shellfish C. Foreign travel D. Illicit drug use E. Working at a daycare

The correct answer is C. In the United States, foreign travel was responsible for 47% of HAV cases from 2004 -2007 and remains the leading cause of HAV infection in the nation followed by consumption of contaminated food. While illicit drug use, blood transfusion, and employment at a daycare are also risk factors, they account for far fewer cases.

A 50-year-old male presents with progressive bilateral knee and ankle pain. You are concerned about a possible inflammatory arthritis. Which of the following key findings on history is most consistent with an inflammatory etiology of this patient's pain? A. Monoarthritis with weakness B. Morning stiffness lasting less than one hour C. Polyarthritis with joint warmth D. Previous trauma to affected joint(s) E. Recent overuse of affected joint(s)

The correct answer is C. Inflammatory arthritis often affects multiple joints, and most patients will present with joint warmth, as described in answer (C). Answer (B) is incorrect because the morning stiffness associated with inflammatory arthritis should be more, not less, than one hour. A history of previous trauma (D) is more associated with osteoarthritis (OA). A history of recent overuse (E) is commonly associated with tendonitis and osteoarthritis.

A 66-year-old male is admitted to the hospital for treatment of new-onset atrial fibrillation with a rapid ventricular rate. The troponin, complete blood count (CBC), and basic metabolic profile are within normal limits. Which additional laboratory test is indicated as part of the initial workup for new-onset atrial fibrillation? A. Creatine kinase-MB (CK-MB) B. Prothrombin time and international normalized ratio (ProTime INR) C. Thyroid stimulating hormone (TSH) D. Urinalysis E. Vitamin B12

The correct answer is C. Initial laboratory evaluation of new-onset atrial fibrillation should focus on identifying the underlying etiology. Laboratory evaluation includes: thyroid function test to evaluate for thyrotoxicosis troponin to evaluate for inciting ischemic event basic metabolic profile to evaluate for electrolyte abnormality (particularly K and Mg) CBC to evaluate for infection and/or anemia Although CK-MB was used in the past to diagnose acute myocardial infarction, it is not necessary if a troponin is available and negative. A ProTime may be obtained as a baseline if anticoagulation will be initiated, but it is not helpful in diagnosing the underlying etiology of new atrial fibrillation. If recreation stimulant use is suspected (e.g. cocaine), then a drug screen would also be indicated. However, a urinalysis alone would not provide useful information. Vitamin B12 is not known to directly contribute to atrial fibrillation.

An 80-year old male with non-obstructive coronary artery disease, diabetes mellitus type 2, and hypertension presents after a fall he had at home about five hours ago. He felt well this morning. After a light breakfast, he read the newspaper at the kitchen table. When he stood up, he felt lightheaded and fell to the floor. His daughter, who lives with him, heard the thud and found him on the floor. He was orientated to person, place, and time when found by his daughter. He denied having any urinary or stool incontinence. He also has not had fevers, palpitations, abdominal discomfort, hematuria, or dysuria. As he was complaining of left hip pain, she brought him into the emergency room for evaluation. He has been prescribed aspirin, metoprolol, enalapril, amlodipine, and metformin, all of which he took this morning. The enalapril was recently added to his regimen. On exam, vitals were as follows: temperature is 37.6 C (99.7 F), pulse is 60 beats/minute, and blood pressure is 110/70 mmHg. You find a pleasant male who appears his stated age and is in no distress. His exam, including the neurological exam, is unremarkable except for ecchymoses over the left hip. He scores a 29 on the mini-mental state examination (MMSE). He is unable to complete a "Get Up and Go" test due to pain in his left hip. Initial tests show a finger stick glucose of 220 mg/dL, sodium 135 mmol/L, creatinine 1.0 mg/dL, white blood cell count 9,000 cells/mL, hemoglobin 14 g/dL, and troponin I 0.01 ng/mL. A urinalysis showed 4+ bacteria with no leukocyte esterase or nitrites. An electrocardiogram (ECG) showed a normal sinus rhythm with significant Q waves in leads II, III, and avF. A non-contrast computed tomography (CT) of the head shows no structural lesions. A pelvic radiograph and magnetic resonance imaging (MRI) of the pelvis both show no evidence of fracture. What is the most likely contributor to his fall? A. Acute coronary syndrome B. Hypoglycemia C. Medications D. Seizure disorder E. Urinary tract infection

The correct answer is C. Medications are an important risk factor contributing to falls in older people. In this case, orthostatic hypotension related to the recent initiation of enalapril was likely causative. Adverse reactions due to specific types of medications, greater number of medications (polypharmacy), and recent changes in medication dose have been associated with an increased risk of falls. Medication use is one of the most-modifiable risk factors for falls. Other risk factors for falls include visual impairment; cognitive impairment; a history of falls; chronic diseases such as osteoarthritis, diabetes mellitus, and cardiovascular disease; and environmental factors such as loose rugs and lack of handrails. Hypoglycemia is unlikely to have contributed to this fall. His only medication for diabetes is metformin, which does not cause hypoglycemia; in addition, he would not have normal mentation immediately after the fall if it was secondary to hypoglycemia. Likewise, the lack of a postictal state makes a seizure disorder unlikely. He is lacking signs and symptoms of a urinary tract infection, suggesting an asymptomatic bacteriuria that does not require intervention and would not result in a fall. While Q waves are seen on the ECG, these reflect an old myocardial infarction. The normal troponin I result also speaks against a myocardial infarction as an etiology of his fall.

A 65-year-old female is brought to the emergency department by the paramedics after a syncopal event. She reports a prodrome of lightheadedness and diaphoresis while straining to have a bowel movement. Her husband says that she lost consciousness for fewer than five minutes in the bathroom. Her medical history is significant for diabetes mellitus type 2 for the past 10 years. She takes metformin, aspirin, and furosemide daily. She is currently awake and oriented to person, place, and time. Her vital signs are normal, including orthostatic vitals, and she was able to stand for the orthostatic vitals without symptoms. Physical examination and routine blood tests are normal. Her electrocardiogram (ECG) is normal. Which of the following is the most likely etiology of syncope in this patient? A. Hypoglycemia B. Medication-induced bradycardia C. Neurocardiogenic syncope D. Orthostatic syncope E. Tachyarrhythmia

The correct answer is C. Neurocardiogenic (often called vasovagal) syncope is commonly associated with prodrome of lightheadedness, diaphoresis, dizziness, vertigo, etc., and is often situational. Triggers can include coughing, sneezing, gastrointestinal (GI) stimulation, or micturition, especially straining to urinate with an enlarged prostate (in men) or constipation. This patient had both a prodrome and was straining for a bowel movement. Hypoglycemia can cause loss of consciousness, however, this would not result in spontaneous reversal. In addition, her blood tests were normal. The patient's medications are not commonly associated with bradycardia. Positional change is consistent with an orthostatic cause of syncope, however, the patient's orthostatic vital signs were negative (importantly, including her absence of symptoms) and there was not a history of standing prior to loss of consciousness, making this less likely. Women (and men) with a tachyarrhythmia often do not have a prodrome prior to loss of consciousness. In addition, her lab tests (electrolytes) and ECG were normal, making this less likely. Please note, however, that a paroxysmal tachyarrhythmia cannot be completely ruled out at this time.

A 50-year-old female comes to the office with joint pain. She has pain, swelling, and warmth in her bilateral metacarpophalangeal (MCP) and proximal interphalangeal (PIP) joints, wrists, and ankles consistent with chronic inflammatory polyarthritis. She has no skin or oral concerns. Which of the following is the most common cause of this patient's clinical presentation? A. Dermatomyositis B. Gonococcal arthritis C. Rheumatoid arthritis D. Systemic lupus erythematosus E. Systemic vasculitis

The correct answer is C. Rheumatoid arthritis (RA) (C) is the most common​ cause of chronic inflammatory polyarthritis, affecting 0.5-1% of the world's population, and is more common in females with the peak onset between the ages of 50 and 75. Dermatomyositis (A) is a rare disorder, with an incidence of 9.63 cases per million people. Patients typically present with proximal muscle weakness and classic skin findings, not consistent with this case. Gonococcal arthritis (B) typically presents as a migratory polyarthritis, rather than a chronic polyarthritis. Arthritis results from hematogenous spread of the bacteria, which occurs in only 0.5-3% of cases of Neisseria gonorrhoeae infections. The diagnosis of systemic lupus erythematosus (D) is based on a constellation of clinical and laboratory findings. The patient's lack of skin or oral concerns makes the diagnosis of SLE less likely than RA. Systemic vasculitis (E) is not a cause of chronic inflammatory polyarthritis.

An 85-year-old female with widely metastatic breast cancer presents to the emergency department with confusion, fatigue, constipation, and global weakness. Her daughter reports that she has been drinking over three liters a day of water and juice. She has not had a bowel movement in five days. Her labs are significant for a calcium of 15mg/dL. An electrocardiogram (ECG) is ordered. What finding is the hypercalcemia most likely to cause on her ECG? A. Atrial fibrillation B. PR segment depression C. Shortened QT interval D. Sinus tachycardia E. T-wave inversion

The correct answer is C. Severe (>14 mg/dL) hypercalcemia can cause cardiac disturbances, including a shortened QT interval, and rarely first-degree arteriovenous (AV) block and/or an ST-segment elevation mimicking a myocardial infarction. The QT interval is shortened because hypercalcemia shortens phase two of the myocardial action potential. The patient may be tachycardic from her volume depletion, but tachyarrhythmias such as sinus tachycardia, atrial fibrillation, and supraventricular tachycardia have not been associated with hypercalcemia. PR segment depression can be seen in pericarditis, but is not commonly associated with hypercalcemia.

An 80-year-old Spanish-speaking female who lives alone and has no medical information available was found down after an unknown period of time. She is confused and unable to provide a reliable history, even with a Spanish interpreter present throughout your interview. Her exam is notable for normal vital signs and tenderness over the right pubic rami. Hip and pelvis images reveal two pubic rami fractures, but no significant hemorrhage. Laboratory workup is significant for blood urea nitrogen (BUN) of 30 mg/dL and creatinine (Cr) of 1.5 mg/dL. While the Spanish interpreter is still available, your resident asks you to use the Confusion Assessment Method to determine whether delirium is the cause of the patient's confusion. Which of the following is the best method to assess for inattention? A. Count backwards from 100 by increments of seven B. Describe name, location, and date C. Recite the days of the week forward and backward D. Remember three objects after two minutes E. Spell "world" forwards and backwards

The correct answer is C. The Confusion Assessment Method is a validated tool to assess whether confusion is caused by delirium, as opposed to dementia or depression, in geriatric patients. To identify delirium, patients must meet the first two criteria (acute onset and fluctuating course, and inattention) and one of the last two criteria (either disorganized thinking or altered level of consciousness). For patients who have a minimal education, are not literate, or do not speak English, reciting the days of the week forward and backward is a more reliable method to assess inattention. Counting backwards by increments of seven may be falsely positive in a patient who is not educated. Similarly, spelling "world" forwards and backwards may be falsely positive in a patient with low literacy or who does not speak English. Orientation questions (name, location, date) are not part of the confusion assessment method. Remembering three objects after two minutes assesses short-term memory.

A 55-year-old male with a 40-pack-per-year smoking history presents with a several month history of a 15-pound unintentional weight loss, and a worsening cough. He reports polyuria and polydipsia. Physical exam is significant for a wheezing in the lower lung fields bilaterally. His calcium level is 12.5 mg/dL. A chest x-ray is significant for a 2 cm lung nodule next to the left bronchus and prominent hilar lymphadenopathy. What is the next-best test that should be ordered to work up this patient's hypercalcemia? A. 1,25(OH)2 Vitamin D B. Cortisol C. Parathyroid hormone (PTH) D. Parathyroid hormone-related peptide (PTH-rp) E. Thyroid stimulating hormone (TSH)

The correct answer is C. The first step in the diagnostic evaluation of this patient with hypercalcemia is a parathyroid hormone (PTH) level. If the PTH is low, then conducting a malignancy workup with or without a parathyroid hormone-related peptide (PTHrP) would be a reasonable next step. If the PTH levels are normal or high, then he most likely has hyperparathyroidism, because the PTH levels are not being appropriately suppressed by his hypercalcemia.

A 46-year-old female with diabetes and obesity presents to the office for a routine follow-up. She is currently asymptomatic and takes aspirin and metformin. She would like to lose weight. On physical examination, her pulse is 76 beats/minute, blood pressure is 138/72 mmHg, and body mass index (BMI) is 42 kg/m2. She asks if there is anything she could be doing to improve her health. Which of the following is the most appropriate counseling statement for this patient? A. "I think you should try orlistat to lose weight." B. "We need to refer you to the bariatric surgeon given your diabetes and high BMI." C. "We will work together on trying to achieve a 5-10% loss in weight over the next six months." D. "You should exercise more in order to lose weight." E. "You should attempt to lose enough weight to achieve a normal BMI."

The correct answer is C. The most appropriate lifestyle counseling statement indicates support and partnership in the weight loss process. Maintenance of a healthy body weight is essential in the management of patients with diabetes. A 5-10% weight loss over six months is the most appropriate and realistic goal to state prior to discussing more specific recommendations on dietary, exercise, and behavior modifications. Attainment of ideal body weight is too large a goal, especially in morbidly obese patients such as this patient with a BMI > 40. Studies have shown that a modest weight loss of approximately 5-10% of the current weight can lead to significant improvement in glycemic control, blood pressure control, and lipid parameters. Medications such as orlistat and bariatric surgery are inappropriate prior to attempts at lifestyle modification. Non-specific recommendations for exercise are not helpful and do not address dietary changes.

A 42-year-old female comes to the clinic for a follow-up of newly diagnosed type 2 diabetes mellitus. She is asymptomatic and has a normal physical exam. Her blood pressure is 142/76 mmHg, which is consistent with previous blood pressure measurements at home. Results of lab work show 1+ protein on her urinalysis and a normal-basic metabolic panel. What is the most appropriate treatment for the patient? A. Amlodipine B. Hydrochlorothiazide C. Lisinopril D. Metoprolol E. No further treatment

The correct answer is C. The patient in the vignette has hypertension in the setting of type 2 diabetes mellitus, and treatment should be started to lower her risk of cardiovascular disease. She also has moderate macroalbuminuria evidenced by her urinalysis result with 1+ protein. She would benefit from an angiotensin-converting-enzyme (ACE) inhibitor or an angiotensin receptor blocker (ARB) to both control her blood pressure and lower her risk of diabetic nephropathy progression. While amlodipine (a dihydropyridine calcium channel blocker) is an appropriate first-line blood pressure medication in most patients, patients with diabetic nephropathy would benefit more from an ACE inhibitor or an ARB. Neither metoprolol nor hydrochlorothiazide offer the renal protection that an ACE inhibitor or an ARB would.

A 38-year-old male with alcohol use disorder is admitted to the hospital medicine service. He presented to the emergency department with shaking that began earlier today. His last drink was 16 hours ago. He has been drinking a pint of vodka daily for the past 10 years. On physical exam, he is disheveled and unkempt. His vitals reveal temperature is 37.2 C (99 F), pulse is 105 beats/minute, respiratory rate is 16 breaths/minute, and blood pressure is 165/95 mmHg. He has a fine tremor with his arms outstretched without asterixis. Except for the tachycardia, his cardiac exam is unremarkable. He has an unsteady gait and some visual hallucinations, but is fully oriented. He denies any other ingestions or any daily medications. Which of the following should your treatment plan include? A. A beer with each meal B. Cobalamin C. Diazepam D. Folate E. Phenobarbital

The correct answer is C. The patient is having acute alcohol withdrawal with delirium. Long-acting benzodiazepines, such as diazepam, continue to be the cornerstone of alcohol withdrawal treatment, to prevent withdrawal seizures. Beer, or other alcohol, is not recommended as treatment because it is an obvious toxin, it may be associated with more respiratory depression and intoxication-related aggression than benzodiazepines, and it reinforces pathologic behavior and addiction. Cobalamin and folate deficiencies should be diagnosed before replenishing them. Phenobarbital and valproic acid are being tried at some institutions to limit benzodiazepine use. Because of a lack of sufficient data on medical inpatients (outside the intensive care unit (ICU)), they are not considered the standard of care.

A 37-year-old male with a history of alcoholic cirrhosis is brought in to the emergency department by his husband with confusion and lethargy. He was otherwise doing well until yesterday when he slept most of the day and refused his medications. This morning, he was difficult to wake up, and when he did wake up, he was very confused and somewhat combative. His husband reports that up until yesterday, he was adherent to his medications and has not drunk alcohol in four months. The husband says this has never happened to the patient before. On exam, the patient's temperature is 38.5 C (101.3 F), pulse is 98 beats/minute, respiratory rate is 12 breaths/minute, and his blood pressure is 94/64 mmHg. He is somnolent and only arousable to sternal rub. When he does wake up, he can only tell you his name. He is notably jaundiced. His heart and lung exams are normal. His abdomen is distended with a positive fluid wave and flank dullness. He winces as you palpate his abdomen without a clear localization. What is the most likely cause of the patient's altered mental status? A. Alcohol intoxication B. Hepatic encephalopathy C. Spontaneous bacterial peritonitis D. Urinary tract infection E. Variceal bleeding

The correct answer is C. The patient presents with altered mental status and an elevated temperature. He has ascites on his abdominal exam and is diffusely tender. Spontaneous bacterial peritonitis (SBP) is a can't-miss diagnosis, as it can quickly progress to shock, renal failure, and other organ failures. Common presentations in a patient with cirrhosis and ascites include fever, altered mental status, and abdominal pain or tenderness. Any patients with cirrhosis and any of these signs or symptoms should have an urgent paracentesis and should be started on empiric antibiotics to control SBP. Alcohol intoxication is a concern, but in this patient with fever, SBP would be more likely. Hepatic encephalopathy is common, but is not typically associated with fever and abdominal tenderness. Urinary tract infections are common in the general population, but with the presentation of altered mental status, abdominal tenderness, and fever in a patient with ascites, SBP is much more likely. One must have a low index of suspicion for variceal bleeding, but typically this will cause grossly bloody emesis and melena or occasionally hematochezia.

A 32-year-old female comes to the emergency department with increasing abdominal distension and shortness of breath over the past week. She reports no fever or chills but has had decreased intake due to early satiety. Medical history is positive for autoimmune hepatitis. On exam, she is mildly tachypneic but her lungs are clear to auscultation bilaterally. Cardiac exam is normal. Her abdominal exam is significant for marked distension and a positive fluid wave, without tenderness on palpation. You perform a paracentesis and remove five liters of fluid. The fluid is clear with 15 white blood cells (WBCs), 90% neutrophils, 20% leukocytes. The paracentesis fluid albumin is 0.2 g/dL, lactate dehydrogenase (LDH) is 50 U/L, and protein is 1.2 g/dL. The serum albumin is 1.9 g/dL, LDH is 170 U/L, and total protein 5.9 g/dL. Which of the following is the most likely cause of the patient's ascites? A. Congestive heart failure B. Metastatic lung cancer C. Portal hypertension D. Secondary peritonitis E. Spontaneous bacterial peritonitis

The correct answer is C. The patient presents with new ascites. All patients with new onset ascites should have a diagnostic paracentesis to evaluate the cause. In this case, the patient also needed a therapeutic paracentesis to relieve her symptoms of abdominal distension causing shortness of breath. The first test to evaluate ascites is the serum-to-ascites albumin gradient (SAAG), which helps to differentiate ascites due to portal hypertension (SAAG greater than or equal to 1.1 g/dL) from other causes. While high SAAG is highly predictive of portal hypertension, it is possible that the portal hypertension could be due to congestive heart failure (CHF) causing "cardiac ascites." This patient has no other signs or symptoms of CHF and also ascitic fluid protein less than 2.5 g/dL, which also helps to differentiate cirrhosis from heart failure (ascitic fluid protein greater than or equal to 2.5 g/dL). Metastatic lung cancer would have a low SAAG, while spontaneous bacterial peritonitis would be suggested by a neutrophil count greater than 250 (while the patient's neutrophil percentage is high, her total WBC count is only 15 which is a total neutrophil count of 13.5). Secondary peritonitis would be suggested if she had abdominal pain and the peritoneal fluid showed leukocytosis and if more than one organism was seen on peritoneal fluid gram stain or culture. Secondary peritonitis should also be considered if two of the following three criteria are met in the peritoneal fluid: total protein >1g/dL, glucose <50 mg/dL, and LDH>upper limit of normal for serum. This patient does not have a leukocytosis in her peritoneal fluid; therefore, infection (either SBP or secondary peritonitis) is unlikely.

A 54-year-old male is being evaluated for two months of left knee pain. Over the same time period, he has also noticed a "creaking" sensation when walking down stairs and stiffness in the morning that lasts about 20 minutes. He is otherwise feeling well. On exam, there is visible bony enlargement and some mild bony tenderness, but no warmth or effusion. Which of the following is the most-likely diagnosis? A. Anterior cruciate ligament (ACL) tear B. Gout C. Osteoarthritis D. Rheumatoid arthritis E. Septic arthritis

The correct answer is C. The patient's history and exam are more consistent with osteoarthritis (C). The American College of Rheumatology's clinical criteria for diagnosing osteoarthritis of the knee includes any three of the following: age greater than 50 years old, morning stiffness lasting less than 30 minutes, crepitus, bone tenderness, bone enlargement, and no palpable warmth. These criteria are 95% sensitive and 69% specific for a diagnosis of osteoarthritis. Anterior cruciate ligament (ACL) tear (A) usually results from trauma, and presents more acutely. People often report hearing a "popping" sound, with swelling and inability to continue with activity following the injury. Rheumatoid arthritis (D) can present in the knee, but is more common in smaller joints and usually affects people in multiple joints, and typically in a symmetrical pattern. In addition, it is an inflammatory arthritis with prolonged (>1 hour) joint stiffness. Septic arthritis (E) typically presents more acutely with fever, significant joint pain, erythema, warmth, and effusion.

A 48-year-old male with known alcohol abuse history comes to the hospital with abdominal discomfort and early satiety. He has no fevers or chills. He describes the discomfort as generalized. Eating makes it worse, while belching can make it better. The pain does not radiate, and he rates it typically as 3/10. He says his pants no longer fit him well. His abdomen has been getting bigger and his legs are more swollen--both of which he attributes to drinking more alcohol recently. His only medication is an antihypertensive. His last drink was earlier this morning. What physical examination finding would be most specific to identify the cause of this abdominal swelling? A. Caput medusae B. Flank dullness C. Fluid wave D. Lower extremity edema E. Shifting dullness

The correct answer is C. The patient's history is consistent with possible ascites due to portal hypertension from chronic alcohol abuse. His ascites is causing pressure on his intra-abdominal organs, contributing to discomfort and early satiety. While all of the physical exam findings are consistent with ascites, a positive fluid wave is 90% specific for the presence of ascites. Lower extremity edema is frequently present, but can be caused by various etiologies.

You are seeing a delightful 62-year-old female in the primary care clinic for follow-up of pulmonary function tests (PFTs). She has long been your patient, although you have never been able to get her to quit smoking. She has had no hospitalizations since her children were born. Over the past three months she has felt dyspneic with exertion. Her complete blood count (CBC), basic metabolic panel, and stress echo were normal. Her PFTs showed a post-bronchodilator FEV1/FVC of 65% and FEV1 of 41% predicted. Based on this data, what additional information do you need to guide pharmacologic therapy? A. Computed tomography (CT) of her chest B. Folate level C. History of breathlessness with getting dressed D. History of headaches E. Troponin T measurement

The correct answer is C. The post-bronchodilator FEV1/FVC of 65% confirms the diagnosis of chronic obstructive pulmonary disease (COPD) by PFTs, and her FEV1 of 41% predicted places her in the GOLD 3 class of 'severe' airflow limitation. This continues to have a certain prognostic significance, but as of the latest 2017 iteration of the Global Initiative for Obstructive Lung Disease (GOLD) guidelines, spirometry is no longer used to guide therapy. Instead, functional assessment is used, either the Modified MRC (mMRC) dyspnea scale, or the COPD Assessment Test (CAT). Breathlessness with dressing (C) or undressing or with leaving the house is considered grade 4 on the mMRC. Although a low-dose chest CT (A) may be warranted to screen for lung cancer, it is not necessary to guide therapy of her COPD. A history of headaches (D) is not part of either standardized functional assessment tool for COPD. A troponin T (E) measurement is not useful in the absence of symptoms of acute coronary syndrome, and she recently had a negative stress test. She is not anemic; a folate level (B) is not indicated.

You are called to the bedside to evaluate a 45-year-old male with acute dyspnea. He has been hospitalized for treatment of an exacerbation of inflammatory bowel disease for five days. He had no hypoxia or tachypnea on admission. He has no cough. Current medications include methylprednisolone and morphine. Vital signs are significant where temperature is 37.5 C (99.5 F), pulse is 110 beats/minute, respiratory rate is 30 breaths/minute, blood pressure is 120/80 mmHg, and oxygen saturation is 92% on 2 liters. He appears uncomfortable with mild use of accessory muscles of respiration. Lungs are clear to auscultation bilaterally. A chest x-ray shows no infiltrate, effusion, or pulmonary edema. His basic metabolic profile is normal. What is the most-appropriate next step in management? A. Albuterol inhaler B. Blood and urine cultures C. Computed tomography pulmonary angiogram (CTPA) D. Pulmonary function testing E. Vancomycin and cefepime

The correct answer is C. This is a hospitalized patient with acute onset of dyspnea, tachypnea, and hypoxia with no evidence of infection or pulmonary edema. He is at increased risk of pulmonary embolism due to his inflammatory illness and hospitalization. His Wells' score is 4.5, which translates to a moderate pretest probability for a pulmonary embolism (PE). The best next step is a CTPA to evaluate for pulmonary embolus. Although his corticosteroids may blunt a febrile response, he has no fever, abnormal lung sounds, or chest x-ray (CXR) findings to support a diagnosis of pneumonia. If pneumonia was suspected, then blood cultures would be appropriate. Albuterol is unlikely to provide relief, as there is no evidence of bronchospasm on exam (wheezing). Pulmonary function testing is difficult to accomplish inpatient and would not help you assess his acute respiratory issue. Vancomycin and cefepime would be indicated if you thought he had pneumonia but this is not as likely as PE with this presentation and risks.

An 80-year-old female presents to the office with urinary incontinence. She has a history of congestive heart failure (CHF) and severe osteoarthritis of her knees. She takes furosemide, but states that she often misses doses because it makes her urinate more frequently. She is embarrassed when she has urinary incontinence while out of the house. She has no dysuria or difficulty controlling her bowel movements. She can feel the urge to urinate, and had rare difficulty with incontinence prior to starting diuretics. Her exam is significant for clear lungs, severe crepitus of both knees, and a normal neurologic exam. Her "Timed Up and Go" test is 30 seconds. What is the next step in the management of her urinary incontinence? A. Discontinue diuretics B. Initiate tamsulosin C. Modify the timing of her diuretic therapy D. Order a computed tomography (CT) of her abdomen and pelvis E. Place an indwelling urinary catheter

The correct answer is C. This patient has functional incontinence. She has bladder control, but because of her severe osteoarthritis, she is unable to get to the bathroom on time. This is an indication for modifications to her medications or physical environment. Discontinuing diuretics may decrease the frequency of urination, but is likely to trigger a CHF exacerbation. Therefore, changing the timing of her diuretics is likely the safest option. Consider recommending that she take her diuretic first thing in the morning, plan to have easy access to a bathroom for the following six hours, and then schedule social engagements later in the day once the potent diuretic effect has passed. Tamsulosin is helpful in males with benign prostatic hyperplasia (BPH) and overflow incontinence, but not in women with functional incontinence. A CT scan would provide unnecessary radiation, and not manage the underlying problem. An indwelling catheter increases the risk of a urinary tract infection (UTI), delirium, and falls in older adults and is not indicated.

An 82-year-old female with diabetes, hypertension, hyperlipidemia, and a stroke three years ago with residual dysarthria, is admitted to the intensive care unit (ICU) after her daughter was unable to wake her up this morning. On exam, she is afebrile, and her neurological exam is not focal. A head computed tomography (CT) scan did not reveal any acute pathology. Labs are as follows: white blood count (WBC) 12.4 cells x 10 3 /μL sodium 123mEq/L, potassium 3.4mEq/L, chloride 107 mEq/L, bicarbonate 22 mEq/L, blood urea nitrogen (BUN) 42 mg/dL, creatinine (Cr) 2.3 mg/dL, glucose 840 mg/dL. A serum osmolality is 410 mosm/kg. In addition to insulin, which of the following interventions is most likely to improve her outcome? A. Antibiotics B. Dialysis C. Intravenous (IV) fluids D. Potassium supplementation E. Sodium bicarbonate

The correct answer is C. This patient has hyperosmolar hyperglycemic state. Her glucose is significantly elevated in the absence of ketoacidosis; her anion gap is only 12 (141-107-22). Her serum osmolality is increased, and she has mental status changes. Due to osmotic diuresis in the setting of hyperglycemia, patients are profoundly volume depleted and may develop acute kidney injury. Treatment includes IV insulin and aggressive hydration with IV normal saline fluid. Her sodium, corrected for hyperglycemia, is 141 mg/dL (using a correction factor of 2.4mEq/L for every 100 mg/dL increase in serum glucose above 100 mg/dL). The patient does not have any localizing signs or symptoms of infection, so antibiotics are not indicated. Her potassium is slightly low, but she is not acidemic; therefore potassium and bicarbonate are not indicated. While her creatinine is 2.3 mg/dL and she is older, she is significantly volume depleted with her hyperosmolar state, and her creatinine and urine output should be monitored for improvement with fluid administration.

A 68-year-old male presents with concern for substernal chest pressure with exertion lasting five minutes and alleviated with rest. He has a remote history of a myocardial infarction (MI). He recently presented with angina symptoms and had a cardiac catheterization which did not reveal any concerning areas of stenosis. He has not been taking his medications. Physical examination shows his pulse is 88 beats/minute, respiration rate is 16 breaths/minute, and blood pressure is 130/80 mmHg. His lungs are clear, heart sounds are normal, and there is no lower-extremity edema. Which of the following is the best treatment to manage the chest pain? A. Angiotensin-converting-enzyme (ACE) inhibitor B. Baby aspirin C. Beta blocker D. Calcium channel blocker E. Statin

The correct answer is C. This patient has stable angina and should be treated with a beta blocker. The beta blocker will reduce angina by slowing his heart rate and decreasing his blood pressure, thus decreasing myocardial oxygen consumption. Although studies have shown that calcium channel blockers may reduce angina, beta blockers are the first line since data suggests an improved survival rate. Aspirin and statins are indicated for secondary prevention of cardiovascular disease (CVD), but will not treat angina. There is conflicting data as to whether or not ACE inhibitors reduce angina symptoms and these are not recommended by the American College of Cardiology/American Heart Association guidelines for specific management of angina.

A 65-year-old female presents to the emergency department with one month of worsening dyspnea. She reports no fever or cough. Her past medical history is significant for diabetes. She has a 20-pack-per-year smoking history. Vitals show her temperature is 37 C (98.6 F), pulse is 100 beats/minute, respiratory rate is 24 breaths/minute, blood pressure is 120/80 mmHg, and oxygen saturation of 96% on two liters of oxygen by nasal cannula. A chest x-ray shows a large left-sided pleural effusion. A thoracentesis is performed. Serum and pleural fluid results are shown below: Serum protein: 6.0 g/dL Serum lactate dehydrogenase (LDH): 400 U/L Pleural fluid studies: Pleural fluid LDH: 1500 U/L Pleural fluid total protein: 5.5 g/l Pleural fluid glucose: 66 mg/DL Pleural fluid red blood cells (RBCs): 6.5 cells x 106 /µL Pleural fluid white blood cell count (WBC): 4.0 cells x 103/L; 80% lymphocytes. Gram stain shows no organisms. Which of the following is the most appropriate next step in management? A. Consult a thoracic surgeon to place a chest tube B. Refer for pleural biopsy C. Send pleural fluid for cytology D. Treat with ceftriaxone and azithromycin E. Treat with vancomycin and piperacillin-tazobactam

The correct answer is C. This patient is at risk for lung cancer due to her age and smoking history and has an exudative pleural effusion. The next step in determining whether this is a malignant effusion would be to send the pleural fluid for cytology. She has no evidence of infection, and the presentation was subacute, arguing against a parapneumonic effusion. Therefore, treating with antibiotics for community acquired organisms (with ceftriaxone and azithromycin) or hospital-acquired organisms (with vancomycin and piperacillin-tazobactam) would not be appropriate. If the pleural fluid cytology is negative, and a mass is seen after the fluid is drained, then a tissue biopsy may be considered, e.g. pleural biopsy, percutaneous biopsy, or endoscopic biopsy. A chest tube or pigtail catheter is necessary to drain an empyema, but it is not immediately necessary for a sterile effusion. A catheter may be useful therapeutically later on, if a malignant effusion rapidly accumulates and is symptomatic.

A 32-year-old postpartum female comes to see you in the clinic. She has concerns about "splotchy" dark spots above her lip that do not bother her aside from their "ugly" appearance. She reports no acne or other skin conditions such as rosacea. On exam, you note hyperpigmented macules on the upper lip. What is her diagnosis? A. Café au lait spots B. Fixed-drug eruption C. Melasma D. Postinflammatory hyperpigmentation E. Solar lentigo

The correct answer is C. This patient with splotchy hyperpigmentation on the upper lip and recent pregnancy most likely has melasma. This is also common in conjunction with exogenous hormones and can occur in males. Café au lait spots are hyperpigmented macules, however, they appear during early childhood. Fixed-drug eruptions are characterized by edematous plaques with dark postinflammatory pigmentation. Postinflammatory hyperpigmentation occurs following an injury or skin irritation such as severe acne or eczema; in the absence of this history, this is an unlikely diagnosis. Solar lentigo are otherwise known as "age spots." This patient is unlikely to have such lesions at her young age.

A 33-year-old male presents to the emergency department for persistent fever 38.3 C to 38.9 C (101 F to 102 F), night sweats, and fatigue for the past four weeks. This is the second time he has come to the emergency department for these concerns. He does not have insurance and does not have a primary care provider. During his first emergency department visit one week ago, his evaluation included a detailed but non-revealing history and physical examination, normal labs (including complete blood count, complete metabolic panel, erythrocyte sedimentation rate, c-reactive protein, human immunodeficiency virus (HIV) and RNA polymerase chain reaction (PCR), urinalysis, urine culture, blood cultures, lactate dehydrogenase, antinuclear antibodies), negative chest x-ray, and a negative computed tomography (CT) of the chest and abdomen. Which of the following would be the most-appropriate next step? A. Perform a bone marrow biopsy B. Perform a tagged white cell scan C. Perform interferon gamma release assay for tuberculosis D. Repeat blood culture and start empiric antibiotics E. Start therapeutic trial of corticosteroids

The correct answer is C. Tuberculosis is a common cause of infection in a fever of unknown origin (FUO), and an interferon gamma release assay or tuberculin skin test should be performed. Drawing repeat blood cultures may be prudent, but the first set was already negative. In addition, empirically treating with antibiotics or corticosteroids may delay the diagnosis of the underlying etiology of fever and is not recommended. Bone marrow biopsy is not recommended as a screening test for FUO but rather used for directed evaluation. Tagged white cell scans can be helpful when initial evaluation, to include CT of the abdomen and pelvis, are negative and whole body screening is desired. At this time, the next step would be to evaluate for tuberculosis infection before considering bone marrow biopsy or tagged white cell scan.

A 50-year-old male with a history of hypertension presents with urgency, frequency, and pain when he urinates. He reports no fever, but has chills, lower abdominal pain, cloudy urine, and nausea. He notes he had a similar presentation about a year ago that resolved with antibiotics. He states that he has a weak urinary stream and a sensation that his bladder is full even after urinating. His current medications are acetaminophen and amlodipine. On physical examination his vital signs are normal, but his examination is limited due to severe suprapubic tenderness. Laboratory studies show an elevated white blood cell count, normal basic metabolic panel, and negative gonorrhea and chlamydia testing. His urinalysis is consistent with a urinary tract infection (UTI). In addition to antibiotic treatment, which of the following is the most appropriate next step? A. Cystoscopy B. No further management C. Post-void residual bladder volume D. Repeat the urinalysis and urine culture E. Transurethral resection of the prostate

The correct answer is C. Urinary tract infections are more common in females than males, because their urethra is shorter, which allows bacteria to travel a shorter distance to get to the bladder. It is also hypothesized that the antibacterial substances in prostatic fluid decreases urinary tract infection in males. In this patient with recurrent UTI, it is imperative that further workup is done to elucidate the source of his recurrent UTI. A post void residual (PVR) measures the volume of urine that remains in the bladder after urination. Usually values greater than 100-200 ml are considered abnormal. A significant amount of urine remaining in the bladder (urinary retention) serves as a nidus for infection. The most common cause of a high PVR is benign prostatic hyperplasia. Cystoscopy might be indicated if preliminary studies are unremarkable in a male with recurrent urinary tract infection. However, if an alternative etiology such as urinary retention secondary to benign prostatic hyperplasia (BPH) is identified, this would not be needed. A source for recurrent UTI should always be considered in a male patient. Repeating urinalysis (UA) and urine culture (UC) is not required after antibiotic therapy. The decision to repeat UA/UC should be dictated by ongoing symptoms. Transurethral resection of the prostate is not indicated now, since he does not carry a diagnosis of BPH and he has not tried any pharmacologic therapies.

A 25-year-old female presents for advice on contraception methods. She has a history of migraine with aura, but states she has not had a migraine in six months. She takes non-steroidal antiinflammatory drugs as needed for the migraines. She is working at her "dream job", although she often has to work 12- to 14-hour days. She lives with her boyfriend, and they will be getting married in two months. She would like to start having children in about two to three years. She has normal monthly menstrual cycles that last about four to five days. Her last menstrual period was two days ago. She states no alcohol or illicit drug use, but admits that she often smokes 5-10 cigarettes a week and is trying to quit. Physical examination and vital signs are unremarkable. She is requesting the most-effective contraception method against pregnancy that can accommodate her busy work schedule. She is concerned about weight gain. What would be the most appropriate contraception for this patient? A. Condoms B. Depot medroxyprogesterone acetate C. Estrogen/progestin contraceptive pills D. Levonorgestrel intrauterine device E. Post-coital contraceptives

The correct answer is D. A levonorgestrel intrauterine device (IUD) would be the most appropriate contraceptive method for this patient. When deciding about contraception methods, it is imperative that the clinician takes into account cost, the patient's wishes, her desire for pregnancy, the number of sexual partners she has had, and side effects. In this patient, some of the key considerations are her (1) smoking history, (2) desire for pregnancy in two to three years, (3) desire for minimal side effects, and (4) history of migraine headaches with aura. The most-effective methods are the long-acting reversible contraception - intrauterine devices and contraceptive implants. They are associated with the lowest pregnancy rates, because they are not influenced by the patient's action or compliance. Condoms are not as effective as an IUD. Typical use shows an 18% pregnancy rate compared to an IUD at <1%. Depot medroxyprogesterone acetate is an effective contraceptive method, but not the most effective of the choices given. In the first year of typical use, there is a 6% pregnancy rate compared to the IUD at <1%. Also, it requires a visit to the clinician every three months, which might be difficult for a patient who works 12- to 14-hour days. Oral contraceptive pills (OCPs) are not the best choice for this patient with migraine with auras. OCPs with estrogen have been known to increase migraine and also increase the female's risk of stroke. OCPs are contraindicated in patients who are 35 years of age or older and smoke, as there is an increased risk of thromboembolism. This patient does not fall into the age cutoff, but still needs to be cautious using OCPs. Furthermore, her busy work schedule might decrease her ability to take a medication regularly. She is also concerned about weight gain, and OCPs have been associated with bloating. Post-coital contraceptives, also known as the "morning after pill," should not be considered as a form of contraception and should only be used if other methods fail.

A 40-year-old female comes to your office after an emergency department visit for abdominal pain, which has subsequently resolved. However, an ultrasound at that time suggested non-alcoholic fatty liver disease. She is worried about developing cirrhosis. Which of the risk factors is most likely associated with the progression of non-alcoholic fatty liver disease (NAFLD) to non-alcoholic steatohepatitis (NASH), and eventually liver fibrosis (cirrhosis)? A. High amounts of coffee consumption B. Low body mass index (BMI) C. Low low-density lipoprotein (LDL) D. Type 2 diabetes mellitus E. Younger age

The correct answer is D. A number of factors have been associated with the progression of NAFLD to fibrosis and eventually liver failure requiring transplant. Liver biopsy is often required to evaluate the stage of disease and monitor for progression. In a systematic review, diabetes, obesity, older age, and high LDL were shown to be risk factors for fibrosis progression in NAFLD. Weight loss and coffee consumption have been shown in randomized controlled trials to reduce the progression of fibrosis in patients with NASH.

A 28-year-old Caucasian female presents to the clinic for her annual exam. She has no complaints. On exam, her pulse is 84 beats/minute, her blood pressure is 116/70 mmHg, and her body mass index (BMI) is 28 kg/m2. Which of the following would prompt screening for diabetes? A. A hemoglobin A1c (HbA1c) of 5 at last year's exam B. A mother recently diagnosed with hypothyroidism C. A paternal aunt with diabetes D. A history of gestational diabetes with her last pregnancy, seven years ago. E. Frequent consumption of refined carbohydrates

The correct answer is D. According to the American Diabetes Association, all individuals should be screened for diabetes by age 45. In addition, screening should be pursued in any adult who is overweight (BMI > 25) with at least one of the following risk factors: physical inactivity first-degree relative with diabetes high-risk race/ethnicity (ex, African American, Latin, Native American, Asian American, Pacific Islander) females who delivered a baby weighing more than nine pounds or who was diagnosed with gestational diabetes blood pressure greater than or equal to 140/90 mmHg or on therapy for hypertension females with polycystic ovary syndrome HbA1c greater than or equal to 5.7% other clinical conditions associated with insulin resistance history of cardiovascular disease Diet is not included as a risk factor for diabetes screening. First-degree relatives with medical conditions other than diabetes are also not considered a risk factor.

Your new admission is a 60-year-old male with a massive retroperitoneal hemorrhage which required embolization in interventional radiology. His only past medical history is atrial fibrillation, for which he takes rivaroxaban. His blood pressure was 70/30 mmHg upon arrival in the emergency department (ED) and has since stabilized. His admission labs show his blood urea nitrogen (BUN) is 45 mg/dL and creatinine (Cr) is 3.2 mg/dL. His urine microscopy shows muddy brown casts (see image). What is the most-likely diagnosis? A. Acute glomerulonephritis (Acute GN) B. Acute interstitial nephritis (AIN) C. Acute pyelonephritis D. Acute tubular necrosis (ATN) E. Chronic kidney disease (CKD)

The correct answer is D. Acute tubular necrosis (ATN) is the most-common cause of intrinsic renal injury, and results from ischemic or toxic insult to the tubules. For this patient, it was his hypovolemia from massive acute blood loss that led to tubular ischemia. Muddy brown casts, as depicted in the image, can be seen in ATN, but are absent in 20-30% of cases. Management of ATN includes correcting the underlying condition (i.e. improving renal perfusion, stopping offending medication) and close monitoring and management of fluid and electrolytes. Acute GN results from an autoimmune response that leads to inflammation and damage to the glomeruli. The urinalysis (UA) in Acute GN shows hematuria, proteinuria, and red blood cell (RBC) casts AIN is most-commonly caused by infections and medications (i.e. antibiotics, proton pump inhibitors, non-steroidal anti-inflammatory drugs (NSAIDs), etc.) and the UA often shows sterile pyuria. Urine eosinophils can be seen, but this test lacks both sensitivity and specificity. Acute pyelonephritis is an infection of the kidney, and patients generally present with signs and symptoms of an active infection. In addition, the UA should support the diagnosis of an active urinary tract infection (UTI), such as the presence of white blood cells (WBCs) CKD is defined as either kidney damage or a decreased glomerular filtration rate (GFR) of less than 60 mL/min/1.73 m2 for at least three months. This patient's kidney damage is likely acute.

A 59-year-old male with a history of end-stage kidney disease, for which he is on hemodialysis, presents to the emergency department with 12 hours of fever and severe headache. During a hospitalization 10 days ago, he was diagnosed with Staph aureus bacteremia. Since he was discharged five days ago, he has been receiving vancomycin at his dialysis sessions. His exam is notable for a temperature of 39.1C (102.4F), a stiff neck with flexion, and blurred edges of the optic disc on retinal exam. He is fatigued, but his neurological exam is otherwise normal. He was given intravenous (IV) ceftriaxone by the emergency medicine team. Which of the following is a contraindication for performing a lumbar puncture? A. Aspirin, 81 mg daily B. Bacteremia C. Fever D. Papilledema E. Previous administration of antibiotics

The correct answer is D. Although a lumbar puncture should be performed as quickly as possible if meningitis is suspected, papilledema is worrisome for increased intracranial pressure, and a lumbar puncture should be avoided until detailed neuroanatomical imaging studies (e.g. head computed tomography (CT)) have been completed. A history of fever, previous administration of antibiotics, or bacteremia are not contraindications for a lumbar puncture. A history of uncorrected coagulopathy and use of antiplatelet agents or anticoagulants are considered relative contraindications to lumbar puncture. There is no consensus on absolute cut-offs for platelet counts, doses of anticoagulants, or values of clotting parameters when deciding whether to perform a lumbar puncture. However, most invasive procedures, including lumbar puncture, can be safely performed on low-dose aspirin.

A 25-year-old female is brought to the emergency department for a one-hour history of decreased mental status. She was found by her roommate barely arousable on the couch with an empty unlabeled pill bottle on the floor next to her. The woman mumbles inaudible responses to questions. Physical examination reveals pinpoint pupils; decreased muscle tone; and slow, shallow respirations. Which of the following medication classes is the most likely cause of this patient's findings? A. Antihistamines B. Barbiturates C. Benzodiazepines D. Opioids E. Tricyclic antidepressants

The correct answer is D. Although benzodiazepines, barbiturates, antihistamines, and tricyclic antidepressants can all cause drowsiness, the combination of slow, shallow respirations and pinpoint pupils, is typical of opioid overdose (D). Tricyclic antidepressant (E) or antihistamine (A) overdose would cause anticholinergic symptoms, including dilated pupils, flushed skin, dry mouth, tachycardia, hypoactive bowel sounds, and urinary retention. Barbiturates (B) and benzodiazepines (C) can cause hypotonia and respiratory depression as well, but not pinpoint pupils.

A 67-year-old male is admitted to the hospital with a non-ST elevation myocardial infarction. He took no medications prior to admission. During his hospital stay, he is noted to have hypertension. Echocardiogram shows left ventricular hypertrophy, an ejection fraction of 60%, and normal wall motion and valvular function. Which of the following is the best antihypertensive agent to start in this patient? A. Hydralazine B. Isosorbide mononitrate C. Lisinopril D. Metoprolol E. Spironolactone

The correct answer is D. Beta-blockers, like metoprolol, are first-line agents for hypertension in the setting of coronary artery disease. Angiotensin-converting enzyme (ACE) inhibitors are recommended in the setting of new left-ventricular systolic dysfunction. Spironolactone can be used for hypertension, but is typically not a first-line agent; it has been shown to reduce mortality in patients with reduced ejection fraction. Similarly, hydralazine and nitrates may be added to a multidrug, antihypertensive regimen. The combination of both has been shown to decrease morbidity and mortality in select patients with heart failure with reduced ejection fraction.

A 78-year-old male has been admitted to the hospital after a severe stroke. Past medical history is significant for congestive heart failure, hypertension, and chronic obstructive pulmonary disease (COPD). On day five of his hospitalization, he develops a fever, productive cough, pleuritic chest pain, and shortness of breath. Pulmonary exam reveals bronchial breath sounds, dullness to percussion, increased tactile vocal fremitus, and whispered pectoriloquy in the right-lower lung fields. There are no adventitious sounds. What is the most likely diagnosis? A. Chronic obstructive pulmonary disease exacerbation B. Congestive heart failure exacerbation C. Pleural effusion D. Pneumonia E. Pulmonary embolism

The correct answer is D. Cerebrovascular accidents frequently lead to dysphagia and predispose patients to aspiration. This patient's pulmonary exam findings are consistent with a right-lower lobe consolidation from an aspiration pneumonia. Lung exam findings that should raise the suspicion for chronic obstructive pulmonary disease exacerbation include wheezing and coarse crackles. Congestive heart failure exacerbations are typically associated with fine bibasilar crackles. Lung exam is usually unremarkable in pulmonary embolism. Presence of fluid in the pleural space is associated with dullness to percussion. However, breath sounds and tactile vocal fremitus are typically decreased in patients with a pleural effusion.

Mrs. Grant is a 72-year-old female with New York Heart Association (NYHA) grade 3 heart failure with a left ventricular ejection fraction of 30%, severe chronic obstructive pulmonary disease (COPD) with ongoing tobacco use, and a recent diagnosis of dementia. During her office visit, her daughter asks if Mrs. Grant should get a colonoscopy. Her last colonoscopy, which was normal, was done when she was 62 years old. She does not have any gastrointestinal symptoms. On examination, pulse is 92 beats/minute, blood pressure is 106/72 mmHg, and body mass index (BMI) is 20.4 kg/m2. She is barrel-chested, has a prolonged expiratory phase, scattered wheezes, and bibasilar crackles. Cardiac exam is regular with a grade 2/6 systolic murmur at the apex. She has 1+ bilateral pedal edema. What approach would you recommend regarding a screening colonoscopy? A. Discourage her, because the test would not be covered by Mrs. Grant's insurance B. Insist that Mrs. Grant have the colonoscopy as soon as possible C. Perform an alternative screening test for colon cancer, such as a fecal immunochemical test (FIT) D. Recommend against colonoscopy due to comorbid conditions and life expectancy E. Schedule a colonoscopy once Mrs. Grant's ejection fraction improves and she quits smoking

The correct answer is D. Colonoscopy meets the criteria for a screening method that will detect early, asymptomatic disease for which there is a treatment. With colon cancer, the lead time from earliest polyp to invasive cancer is about 10 years. Since Mrs. Grant had a normal colonoscopy 10 years ago, even if she developed a polyp in the intervening period, it would be several years before she would develop invasive cancer. With her underlying cardiac and pulmonary disease, in addition to her dementia, her life expectancy is less than that. In addition, it is likely that she would not tolerate aggressive therapies if colon cancer was identified. Thus, there would be no benefit for her undergoing screening testing of any type, including delayed colonoscopy, immediate colonoscopy, or fecal immunochemical testing (FIT). Insurance coverage for appropriately timed screening colonoscopy is not a common issue. Even if she were to quit smoking or her ejection fraction were to improve, her life expectancy is likely still foreshortened, and a colonoscopy would not be recommended. U.S. Preventive Services Task Force (USPSTF) cancer screening recommendations are generally covered by all insurers.

A 75-year-old female is brought to the emergency department with a one-day history of somnolence and confusion. She resides in a skilled nursing facility. Medical history is significant for mild dementia. Two weeks ago she received ciprofloxacin for white blood cells (WBCs) on urinalysis. She is normally alert and able to converse, but for the past day she has been sleeping most of the time with only brief intervals of alertness. She has urge incontinence at baseline, but has previously denied any dysuria, increased frequency, or hematuria. Nursing records show six liquid stools over the past two days, but no fever or chills, and no abdominal or flank pain. On exam, her vital signs are normal. She states the days of the week forward correctly, but cannot recite the days backwards. She falls asleep during your interview. The remainder of her exam, including her abdominal and neurologic exam, are unremarkable. Basic metabolic profile shows a mildly elevated blood urea nitrogen (BUN). A complete blood count (CBC) reveals a mild chronic anemia and leukocytosis. Which of the following would be the most appropriate diagnostic study to determine the cause of her delirium? A. Computed tomography (CT) of her head B. Electroencephalogram (EEG) C. Ferritin D. Stool C. difficile toxin test E. Urine culture

The correct answer is D. Delirium can be precipitated by any infection in a vulnerable older adult. However, delirium is commonly misattributed to a urinary tract infection (UTI) in patients with bacterial colonization or pyuria, which is common among older adult female residents of skilled nursing facilities (SNFs). An unfortunate consequence of incorrectly diagnosing UTIs in this population is the overuse of antibiotics and increased risk of C. difficile infection. Ciprofloxacin is commonly associated with C. difficile infection. A CT of her head is not part of an initial workup for delirium unless there is a history of fall with head trauma, headache, or focal neurologic deficit on exam. An EEG is not part of the initial workup of delirium unless there is a history of seizure-like activity or elevated risk factors for subclinical seizures. A ferritin level can help elucidate the cause of her anemia, but her anemia is neither severe nor acute, so very unlikely to be the cause of this patient's delirium. A urine culture should not be checked unless she has specific signs or symptoms of a UTI, e.g. dysuria, pain, increased frequency, fever, or leukocytosis.

A 37-year-old female comes to the clinic with a two-week history of intermittent fevers to 39.4 C (103 F). She also complains of chills, myalgias, and headaches associated with the fevers which occur about twice a day. She was previously healthy. She lives in the northeast United States, but traveled to rural Pakistan one month ago to visit family. Laboratory studies are shown as: White blood count (WBC): 8500 /mm3 Hemoglobin: 9.2 g/dl Hematocrit (Hct): 28% Platelets: 75000 /mm3. Which diagnostic study will most-likely confirm the etiology of her symptoms? A. Chest x-ray B. Computed tomography (CT) of the head C. Lyme enzyme immunoassay (EIA) D. Thick and thin blood smears E. Tuberculosis skin testing

The correct answer is D. Given this patient's recent travel to Pakistan, history of periodic fevers, headache, and anemia and thrombocytopenia on complete blood count (CBC), there is a concern for malaria. Thick and thin smears are the appropriate test of choice for diagnosis. Without symptoms of respiratory illness, a chest x-ray may not be helpful. A CT of the head is warranted if there are signs of cerebral edema (such as nausea, vomiting, dizziness, vision changes, ataxia) in cases of malaria, but a headache alone would not necessarily warrant a CT of the head and would not lead to a definitive diagnosis in this case. This patient lives in an area of high Lyme prevalence, but the clinical presentation with anemia and thrombocytopenia is not consistent with Lyme disease. Tuberculosis testing may be warranted if her evaluation remains inconclusive, but given diagnoses that are more probable in this case, tuberculosis skin testing would not be the next-best test of choice.

Which of the following factors carries the highest risk for contracting hepatitis C? A. Birth between 1945 and 1965 B. Blood transfusion C. Consumption of more than three alcohol-containing drinks per day D. Injection drug use E. Sexual contact with an injection drug user

The correct answer is D. Globally, it is estimated that 185 million people have chronic infection with hepatitis C virus (HCV). Injection drug use is associated with acquisition of almost 50% of hepatitis C in the United States. The Centers for Disease Control and Prevention (CDC) recommends testing for chronic infection for individuals in high-risk cohorts. The baby boomer generation (birth cohort 1945-1965) is considered at risk due to large proportions of the population with histories of injection drug use, multiple sexual partners, and tattoos prior to sterile techniques being used. Blood transfusion accounts for less than 1% as does organ/tissue donation. Alcohol consumption alone was not associated with increased HCV infection rates and was not a risk factor. Sexual contact as the only identifiable risk accounts for 15% to 20% of incidence.

Ms. Tsvetanova is a 42-year-old female with no significant past medical history presenting to establish care with her primary care physician. On review, she notes a weight gain of 14 kg (30 lbs) over the last three years. She attributes this mostly to her sedentary lifestyle, snacking, and difficulty with portion control. She works as a receptionist for a local physician's office and spends most of her day sitting. She denies constipation, low energy, cold intolerance, muscle weakness, depressed mood, easy bruisability, or other skin changes. On physical exam, vital signs reveal temperature is 36.8 C (98.2 F), pulse is 82 beats/minute, respiratory rate is 12 breaths/minute, blood pressure is 130/82 mmHg, weight is 81.6 kg (180 lbs), and height is 163 cm (64 in). The remainder of her physical exam is normal. Which of the following laboratory tests is most appropriate for the evaluation of this patient? A. 24-hour urine catecholamine levels B. 24-hour urine cortisol level C. Basic metabolic profile D. Lipid profile E. Thyroid stimulating hormone (TSH)

The correct answer is D. In the absence of symptoms and signs associated with secondary causes of obesity such as hypothyroidism and Cushing disease, the initial laboratory evaluation in patients with obesity should be limited to assessment of coexisting risk factors for atherosclerotic cardiovascular disease (ASCVD), including dyslipidemia and diabetes. Guidelines regarding lipid screening in adults from the American Heart Association (AHA) recommends screening all adults aged 20 to 79 years old every four to six years in those without ASCVD.

A 49-year-old male comes into the clinic with new abdominal swelling. He has no past medical history and had otherwise been well prior to presentation. On exam, he is well-appearing and in no apparent distress. His vital signs, cardiac, and pulmonary exams are normal. His abdomen is distended and tense with a positive fluid wave. He is not tender on exam. The clinic is equipped with an ultrasound, so you prepare to do a diagnostic paracentesis. Before you start, you obtain the patient's consent. Which of the following is NOT a component of informed consent? A. Ability to find the cause of the ascites B. Bleeding risks of a paracentesis C. Infection risks of a paracentesis D. Size of the needle used in the procedure E. Watchful waiting as an alternative to the procedure

The correct answer is D. Informed consent should include: the diagnosis and reason for the procedure, description of the proposed procedure, alternatives to the procedure, risks of the procedure, and risks of not doing the procedure. While size of the needle can be discussed when explaining the procedure, it is not a vital part of the informed consent.

A 52-year-old female with hypertension, diabetes, and obesity presents to the office for a routine follow-up. She is currently asymptomatic and on lisinopril, aspirin, simvastatin, and metformin. On physical examination, her pulse is 76 beats/minute, blood pressure is 128/72 mmHg, and body mass index (BMI) is 32 kg/m2. She is due for an annual examination of her feet. Which of the following is the most appropriate component of the foot exam for this patient? A. Assessment of Achilles tendon thickness B. Assessment of femoral pulses C. Assessment of patellar reflex D. Inspection between toes for evidence of skin breakdown E. Use of monofilament to assess temperature sensation

The correct answer is D. It is very important to assess skin integrity in areas that may not be easily seen or visualized by patients, including between the toes. Areas that have skin breakdown could be sources of systemic or local infection, including fungal infections. Assessment of Achilles tendon thickness is useful to assess for tendinopathy or injury, but is not likely to be of benefit in this asymptomatic patient. Increased tendon thickness is associated with lipid abnormalities, but is not a routine part of the diabetic foot exam. Assessment of dorsalis pedis and posterior tibial pulses, not femoral pulse, would be appropriate for evaluation of peripheral arterial disease. Patellar reflex is a part of the neurologic exam, but of greater importance in the foot exam for this patient is assessment of the Achilles reflex. Use of a monofilament would be appropriate to assess pressure sensation, not temperature sensation.

A 51-year-old female with a history of rheumatoid arthritis and coronary artery disease is admitted to the hospital with acute liver failure. During the admission process, the team calls her pharmacy and primary care physician and confirms the doses and route of administration and her adherence to her prescription medications. What additional information should be obtained in the medication reconciliation process? A. Allergies to medications B. Cost of prescriptions C. List of all medications taken in the last five years D. List of over-the-counter medications taken E. Number of refills remaining on prescriptions

The correct answer is D. Medication reconciliation is the process of obtaining an accurate list of medications, both prescription and over-the-counter, along with dosages, route of administration, and an assessment of adherence to the medications. Obtaining an accurate medication list on admission increases the likelihood of an accurate medication list on discharge. Over-the-counter medications and traditional remedies are easily overlooked, but should be considered when new findings, such as liver toxicity, are diagnosed. While medication allergies should be assessed with every admission, this is not considered part of medication reconciliation. Cost of medications should be considered, especially if there is an issue with adherence, but this is not part of reconciliation. Prior history of medications and number of refills are also not part of reconciliation.

A 78-year-old male who recently entered an assisted living facility is brought to the office because of increasing confusion and somnolence. He has a history of mild dementia, hypertension, hypercholesterolemia, benign prostatic hypertrophy, and insomnia. He has had no fevers, pain, vomiting, diarrhea, constipation, or dysuria. He takes amlodipine, atorvastatin, donepezil, lorazepam, and tamsulosin. His vital signs are normal, and aside from short-term memory loss, his physical exam is unremarkable for abnormal findings. Which of the following medications should be discontinued? A. Amlodipine B. Atorvastatin C. Donepezil D. Lorazepam E. Tamsulosin

The correct answer is D. Medications that act on the central nervous system as a primary target or as a side effect can precipitate delirium in older adults, especially adults with underlying cognitive impairment or dementia. Common precipitants include prescription medications such as benzodiazepines (like lorazepam), opioids, central-acting antihypertensives (e.g. clonidine), as well as non-prescription medications, such as diphenhydramine, a medication with anticholinergic properties. Donepezil is centrally acting, but is used to treat dementia; lorazepam is a more likely trigger of delirium. Amlodipine, atorvastatin, and tamsulosin do not act centrally. The only time a benzodiazepine, such as diazepam or lorazepam, would be used in delirium would be if the delirium were triggered by alcohol withdrawal. In this situation, long-acting benzodiazepines are the preferred therapy. Before complete discontinuation of the lorazepam, it is important to clarify the dose and frequency with which the patient uses it. If he is taking it on a regular basis, it would be best to discuss the need for a taper with a pharmacist or toxicologist to avoid precipitating benzodiazepine withdrawal.

Ms. May is a 28-year-old female who comes to see you because her 58-year-old mother was recently diagnosed with breast cancer. Ms. May has no medical issues and takes no medications. A detailed family history reveals that her maternal great-aunt died of breast cancer at 65 years old. She has no relatives with ovarian cancer. Her breast examination is normal. Based on U.S. Preventive Services Task Force (USPSTF) guidelines, what would you recommend for breast cancer screening for Ms. May? A. Mammography now and then every year after the age of 40 B. Monthly breast self-examination with mammography beginning at the age of 40 C. Routine mammography every two years beginning at the age of 50 D. Shared decision-making discussion regarding initiation of mammography E. Yearly mammography at the age of 60

The correct answer is D. Ms. May is at higher risk for breast cancer based on her family history; therefore, she may be more likely to benefit from breast cancer screening beginning at the age of 40. It is recommended that providers and patients have shared-decision making discussions regarding the risks and benefits of mammography beginning at the age of 40. Breast self-examination is no longer recommended as a screening strategy. In the absence of any findings on examination, there is no indication for immediate mammography. Yearly mammograms for screening purposes are not recommended.

You are evaluating a 53-year-old male in the emergency department who is presenting with a chief concern of shortness of breath. He reports a one-day history of progressively worsening dyspnea on exertion, productive cough, and subjective fever. His past medical history includes chronic obstructive pulmonary disease (COPD) and ischemic cardiomyopathy with a stable ejection fraction (EF) of 40%. Which of the following physical exam findings is most supportive of an infectious etiology of his shortness of breath? A. Absent breath sounds on one side B. Bibasilar crackles and mild wheezes C. Diminished inspiratory effort D. Increased fremitus and egophony over part of the lung E. Tachycardia with normal breath sounds on exam

The correct answer is D. Physical exam findings most consistent with infectious etiology include rhonchi, crackles, egophony ("E" to "A" changes with auscultation due to consolidated tissue not transmitting the "E" pitch) and increased fremitus (indicates denser, more inflamed tissue) (D). Absent breath sounds (A), along with a deviated trachea, should raise concern for a possible pneumothorax. BIbasilar crackles and wheezing (B) may be consistent with congestive heart failure (CHF). Wheezing can also be heard in COPD and asthma, however the edema associated with CHF can also cause airway narrowing leading to wheezing. A diminished respiratory effort (C) in a patient with shortness of breath would prompt an evaluation for a neuromuscular etiology of their dyspnea. Tachycardia with normal breath sounds on exam (E) in a patient presenting with dyspnea may be concerning for pulmonary embolism (PE).

A 45-year-old human immunodeficiency virus (HIV)-infected male presents to the emergency department with nonproductive cough, fever, and progressive exertional dyspnea for one week. His CD4 count was 150/microliter one month ago. He has not been adherent to his antiretroviral regimen. On arrival, his temperature is 38.3 C (100.9 F), heart rate is 98 beats/minute, blood pressure is 110/70 mmHg, and oxygen saturation is 89% on room air. Physical exam is unremarkable except for oral thrush. Chest x-ray shows diffuse bilateral infiltrates with early cystic changes. Which organism is most likely causing this patient's acute symptoms? A. Aspergillus fumigatus B. Candida albicans C. Mycobacterium tuberculosis D. Pneumocystis jirovecii E. Streptococcus pneumoniae

The correct answer is D. Pneumocystis jirovecii is the most likely cause of pneumonia in this patient with a CD4 count of less than 200/microliter and characteristic chest x-ray findings. While Streptococcus pneumoniae remains the most common cause of community-acquired bacterial pneumonia in HIV patients, it typically presents with lobar or focal consolidation. Invasive aspergillus infections usually present with solitary or multiple nodules that may be cavitary, or areas of patchy consolidation or infiltrates. Candida albicans is a rare cause of primary pulmonary infections, even in immunocompromised patients. Mycobacterium tuberculosis causes a more-indolent infection than Pneumocystis jirovecii, and is less likely to present with diffuse, bilateral infiltrates on chest radiography.

A 45-year-old female presents with urgency, frequency, and pain when she urinates. She also has left-lower back pain, fever, chills, nausea, and vomiting. She has a history of type 2 diabetes mellitus and takes metformin. On physical examination, her vital signs show her temperature 38.9 C (102 F), blood pressure is 90/55 mmHg, pulse is 120 beats/minute, respiratory rate is 18 breaths/minute, and oxygen saturation is 98% on ambient air. She has left flank tenderness and lower abdominal tenderness. Laboratory studies showed an elevated white blood cell count. Urinalysis showed pyuria with white blood cell cast. Blood and urine cultures were sent. She is admitted to the hospital and started on intravenous fluids and ceftriaxone. Urine culture shows pan-sensitive E. coli. Despite three days of antibiotics, she remains febrile with a peak temperature of 38.9 C (102 F). Her abdominal pain persists, albeit is slightly better, and she continues to have nausea and vomiting. Which of the following is the most appropriate next step? A. Add vancomycin B. Change ceftriaxone to another class of antibiotic C. Consult urology D. Obtain a chest radiograph E. Obtain a renal ultrasound

The correct answer is E. Imaging studies are not routinely ordered in cystitis or pyelonephritis, but are needed to further evaluate patients who fail to respond to therapy. This patient has pyelonephritis. She was started on an appropriate antibiotic (ceftriaxone) with cultures confirming that the choice of ceftriaxone was suitable. However, she displayed minimal clinical improvement despite thee days of antibiotics. This is concerning and might indicate an obstruction, nephrolithiasis, or renal/perinephric abscess. A computed tomography (CT) scan or renal ultrasound is warranted. There is no indication for gram-positive coverage, such as vancomycin, based on the urine culture. Furthermore, pyelonephritis is most commonly caused by gram-negative organisms. Based on the urine culture showing pan-sensitive E. coli, there is no indication for changing the ceftriaxone to a different antibiotic. It will be premature to call urology without first evaluating for obstruction, nephrolithiasis, or abscess. Based on the results of the renal ultrasound, a urology consult might be warranted. She has no respiratory symptoms warranting a chest radiograph. The most probable cause of her persistent fever appears to be genitourinary in origin.

A 31-year-old male with previously diagnosed hypertension comes in for a follow-up visit. He currently takes 10 mg of amlodipine daily, 40 mg of enalapril daily, and 100 mg of atenolol daily. His pulse is 65 beats/minute. His blood pressure is 190/100 mmHg in his right arm and 188/100 mmHg in his left leg. He reports adherence to all of his medications, and he took all of them this morning. He reports no headache, vision changes, chest pain, palpitations, or shortness of breath. He does not take any over-the-counter medications. He reports no tobacco, alcohol, or drug use. His physical exam is unremarkable. His basic metabolic profile reveals: Sodium 139 Potassium 3.1 Chloride 103 Bicarbonate 21 BUN 10 Creatinine 1.1 Glucose 87 Which of the following is the most likely cause for the man's hypertension? A. Aortic coarctation B. Essential hypertension C. Pheochromocytoma D. Primary hyperaldosteronism E. Renal artery stenosis

The correct answer is D. Primary hyperaldosteronism is thought to be the most common cause of secondary hypertension with prevalence as high as 5-15% in those with hypertension. It should be suspected in younger patients with hypertension that is difficult to control (the patient in this vignette is on maximum doses of three drugs) and often presents with hypokalemia. Patients with primary hyperaldosteronism will have an elevated aldosterone and suppressed renin levels. Aortic coarctation would typically present with a pulse delay and a blood pressure differential between the right arm and the limbs. Essential hypertension is very common. However, in patients whose hypertension is difficult to control on several agents, present at a young age, or who have lab abnormalities (hypokalemia), secondary hypertension should be entertained and excluded. Pheochromocytoma will typically present with symptoms of sweating, heart racing, tachycardia, and pounding headache. This man's lack of symptoms and normal review of systems and exam make this less likely. Renal artery stenosis can also present with difficult-to-control hypertension in a young patient, but often is associated with fibromuscular dysplasia in a young female. It can also result from atherosclerosis in older patients.

You are seeing a frail 85-year-old female who was admitted to your service three months ago with a chronic obstructive pulmonary disease (COPD) exacerbation. Despite your tobacco cessation counseling, she continued to smoke after discharge. She returns with shortness of breath. Six days ago, she had sudden worsening of her shortness of breath, and has since felt progressively more breathless, and is now unable to walk more than 30 feet. She can also no longer climb the stairs to her bedroom, so she is sleeping on the living room sofa. She has no cough, fever, or new orthopnea. In addition to a COPD exacerbation, what other illness most likely explains her acute presentation? A. Acute myocardial infarction B. Idiopathic pulmonary fibrosis C. Lung cancer D. Pulmonary embolism E. Tension pneumothorax

The correct answer is D. Pulmonary embolism (PE) (D) can often coexist with COPD, and smoking is a strong risk factor for venous thromboembolism (VTE). Most of the time, PE presents with acute onset of dyspnea and/or chest pain. However, small PEs can occasionally present more subacutely. An acute myocardial infarction (A) could be a consideration, but would also be expected to present in a matter of minutes to hours. A fairly recent MI causing new onset heart failure may be a possibility, but not an acute infarction alone. Idiopathic pulmonary fibrosis (IPF) (B) and lung cancer (C) would have a more insidious onset. Tension pneumothorax (E) would likely have a more-acute presentation (minutes to hours instead of days), and is often accompanied by ipsilateral chest pain. In addition, patients with poor underlying pulmonary reserve are generally severely symptomatic and unstable.

A 50-year-old male is brought to the hospital by paramedics after a witnessed syncopal event. The event occurred at a local baseball game where he was a spectator. He was evaluated by paramedics at the scene within three minutes of the event and arrived at your hospital within 15 minutes. An intravenous (IV) line was started, but no fluid was given. The patient states that he stood up to get some lunch and suddenly became very lightheaded. He collapsed and does not recall anything until he awoke with many bystanders attending to him. He was aware of his surroundings immediately upon regaining consciousness and was not confused. Bystanders state he had a 30-second period of loss of consciousness and did not hit his head when he collapsed. His vital signs are as follows: temperature is 36.7 C (98 F), pulse is 90 beats/minute, respiratory rate is 12 breaths/minute, and blood pressure is 110/65 mmHg. What is the next best step to determine the cause of the patient's syncopal episode? A. Assessment for pain with passive ankle dorsiflexion (Homan's sign) B. Auscultation of the carotid arteries C. Cranial nerve exam D. Orthostatic blood pressure measurement E. Pulmonary exam

The correct answer is D. Syncope is caused by decreased perfusion to the brain. This patient collapsed after a sudden positional change, therefore orthostatic hypotension should be high in the differential diagnosis and should be assessed with orthostatic blood pressure measurement before fluids are given. Homan's sign is a poor test for deep-vein thrombosis (David L. Simel, Drummond Rennie. The Rational Clinical Examination: Evidence-Based Clinical Diagnosis. New York, NY: McGraw-Hill Education, 2016.) and a pulmonary embolism should give the patient more symptoms if it was large enough to result in syncope. Auscultation of the carotids is not necessary in syncope evaluation, because carotid artery stenosis alone does not result in syncope (due to effective collateral supply). A cranial nerve exam should be performed to assess for neurologic damage from a traumatic fall, but the results will not determine a cause of sudden drop in perfusion. A pulmonary exam is unlikely to help you determine a cause of syncope.

A 25-year-old fair-skinned female presents with a skin lesion she is concerned about. It is located on her upper-right back. She is a runner and runs outside often. She wears sunscreen in the summer with SPF 30. The lesion has been present for years; however, over the last few months she has noticed a change in size and color. The color was initially solid brown, however, it is now brown with a mixture of darker brown and black. It is not painful and does not itch. She describes no easy bleeding in the area. On physical exam, the lesion is 7 mm, irregular and asymmetric in appearance, and has a mixture of brown and black shades. The lesion is a flat patch without crusting or ulceration. Based on this patient's history and examination, which finding has the highest specificity for melanoma? A. Asymmetric appearance B. Color variation C. Diameter D. Evolving character E. History of frequent sun exposure

The correct answer is D. The ABCDE criteria is used to identify pigmented lesions which are more likely to be diagnosed as melanoma. They include asymmetry, border irregularity, color variation, diameter >6 mm, and evolving changes in shade, size, symptoms, and ulceration. This patient's lesion meets all five of the ABCDE criteria, however, evolving character has the highest specificity for melanoma. While a history of frequent sun exposure is a risk factor for the development of melanoma, it does not affect the sensitivity for detecting the presence or absence of melanoma.

A 52-year-old male with hypertension and hyperlipidemia comes to the clinic for a routine follow-up. He has no symptoms. His mother was recently diagnosed with diabetes, and he wants to know if he is also diabetic. He quit smoking two years ago after being admitted to the hospital with chest pain. He has gained 30 pounds since that time, and his body mass index (BMI) is currently 30 kg/m2. His last meal was breakfast two hours ago, which consisted of a bagel and orange juice. Which of the following studies is most likely to support a diagnosis of diabetes? A. Blood glucose level in the clinic of 160 mg/dL B. Blood glucose level of 190 mg/dL two hours after a 75 g glucose load C. Decreased sensation in feet by monofilament test D. Hemoglobin A1c (HbA1c) of 6.8% E. Urinalysis with glucosuria

The correct answer is D. There are four ways to diagnose diabetes: 1) HbA1c > 6.5% 2) Blood sugar > 200 mg/dL, accompanied by polyuria, polydipsia, and unexplained weight loss 3) Fasting blood sugar > 126 mg/dL on two separate occasions 4) Blood sugar > 200 mg/dL two hours after 75 g glucose load, confirmed on a second occasion. Decreased sensation in feet by monofilament and a urinalysis with glucosuria are compatible with diabetes, however neither are diagnostic.

A 70-year-old female presents to the emergency department with moderately severe chest pain. She describes the pain as a vague, deep discomfort and notes associated nausea, diaphoresis, and dyspnea. The pain has no clear aggravating or alleviating factors. Her pain began early on the morning of presentation, waking her from sleep, and has persisted for two hours. Her exam, including vital signs, is normal. What is the most important initial diagnostic test? A. Chest x-ray B. Complete blood count C. Computed tomography (CT) angiography of the chest D. Electrocardiogram (ECG) E. Troponin

The correct answer is D. The first step in evaluating this patient with severe chest pain is an ECG. An ECG can differentiate an ST-segment elevation myocardial infarction from the other acute coronary syndromes. It can also provide other evidence of coronary ischemia, including T-wave inversion and ST-segment depression. Since the outcome of a myocardial infarction depends on timely intervention, rapid identification is essential and an ECG is the most efficient diagnostic test. A chest X-ray may be reasonable for evaluating the source of the chest pain, but with normal vital signs and lung exam, and a primary complaint of anginal chest pain, an ECG should be performed first. A complete blood count would be helpful in evaluating for infection or an acute anemia which could be contributing to the patient's presenting symptoms, but it is a nonspecific test that would not be useful in the timely evaluation for acute coronary syndrome. A CT angiogram of the chest would be obtained to evaluate for pulmonary embolism. This may be appropriate if acute coronary syndrome is ruled out, and suspicion for pulmonary embolism increases. However, the lack of pleuritic chest pain, tachycardia, and hypoxia makes this diagnosis less likely. A troponin test may also be helpful in identifying an acute coronary syndrome in this patient, but the process of drawing blood and lab analysis will take time, during which she could already be receiving appropriate treatment. Additionally, since her pain began only two hours earlier, it is possible that the troponin measurement may be negative. In some instances, serum troponin elevations may not be detectable for four to eight hours after onset of anginal symptoms.

A 21-year-old male with history of hepatitis C, cholelithiasis, sickle cell disease, and alcohol use disorder presents to the hospital with one day of severe abdominal and chest pain, worse in the right-upper quadrant. A fractionated bilirubin reveals the following: Total bilirubin: 5.6 Indirect bilirubin: 4.6 Direct bilirubin: 1.0 What is the most likely cause of the bilirubin pattern in this patient? A. Alcoholic hepatitis B. Choledocholithiasis C. Non-alcoholic steatohepatitis D. Sickle cell crisis E. Viral hepatitis

The correct answer is D. The hemolysis that occurs in sickle cell anemia results in the release of large amounts of heme into the blood stream, which breaks down into bilirubin. Large releases of bilirubin can outpace the capacity of the liver to conjugate (bind bilirubin to glucuronic acid), resulting in high serum and urine levels of unconjugated bilirubin. Viral hepatitis, alcoholic hepatitis, choledocholithiasis, and non-alcoholic steatohepatitis can also cause hyperbilirubinemia, but the bilirubin is primarily conjugated (direct bilirubin).

You are interviewing a 57-year-old female in the emergency department with unilateral leg swelling. Which of the following are additional risk factors for deep-vein thrombosis (DVT) that you should inquire about in the history? A. Alcohol use of three glasses of wine per day B. Allergy to aspirin C. History of Helicobacter pylori (H. pylori) infection D. History of lupus E. Vegetarian diet

The correct answer is D. The history of lupus would put her at potential risk for DVT because of the association with antiphospholipid antibody syndrome, which is a known prothrombotic condition. H. pylori infection, alcohol use, vegetarian diet, and aspirin allergy are not known to increase DVT risk.

A 45-year-old male presents to the emergency department having ingested an unknown liquid found in his garage. He has a severe metabolic acidosis and an osmolar gap of 30. Which of the following findings on urinalysis would be characteristic of ethylene glycol toxicity? A. Calcium carbonate crystals B. Calcium phosphate crystals C. Cystine crystals D. Monohydrate calcium oxalate crystals E. Uric acid crystals

The correct answer is D. The monohydrate or "picket fence" calcium oxalate crystal (D) is pathognomonic of ethylene glycol toxicity. Ethylene glycol is metabolized in the liver by alcohol dehydrogenase to glycolaldehyde and oxalate. Glycolaldehyde is converted to glycolic acid which leads to metabolic acidosis. Oxalate precipitates with calcium in the urine as calcium oxalate crystals and can cause tubular damage to the kidneys. Calcium carbonate crystals (A) are generally found in alkaline urine and not associated with ethylene glycol toxicity. Calcium phosphate crystals (B) can be normal components of the urine. Cystine crystals (C) are a rare finding in patients with a genetic disorder that causes cystinuria and can predispose those patients to form cystine stones. Uric acid crystals (E) in the urine may be a normal variant but are most commonly seen with uric acid nephropathy.

A 49-year-old female is evaluated for fatigue, pruritus, dry eyes, and dry mouth. Her symptoms have been present for about four months and have progressively worsened. She denies any fever, chills, or weight changes. She has hyperlipidemia for which she takes atorvastatin. She has no other medical problems. Her vital signs are normal, and body mass index (BMI) is 24 kg/m2. Her physical exam is notable for excoriations on her arms, legs, chest, and back; otherwise unremarkable. Her alanine transaminase (ALT) is 76 U/L, aspartate aminotransferase (AST): 55 U/L, alkaline phosphatase (ALP): 356 U/L, total bilirubin: 1.3 mg/dL, and direct bilirubin: 0.7 mg/dL Her right-upper-quadrant ultrasound is normal.Which of the following is the most likely diagnosis? A. Cirrhosis B. Crigler-Najjar syndrome C. Gilbert syndrome D. Primary biliary cholangitis (PBC) E. Wilson disease

The correct answer is D. The most likely diagnosis is primary biliary cholangitis (PBC), previously known as primary biliary cirrhosis. PBC mainly affects middle-aged females. Most patients are initially asymptomatic, but as the disease progresses, they may present with fatigue, pruritus, dry mouth, and dry eyes. Hyperlipidemia is common. Alkaline phosphatase (ALP) is usually > 1.5 the upper limit of normal, and aspartate transaminase (AST) and alanine transaminase (ALT) less than five times (5x) the upper limit of normal. Total bilirubin levels may be normal early, but become elevated with increased conjugated bilirubin in advanced disease. In 90% to 95% of patients, antimitochondrial antibody is present. Conversely, etiologies of increased bilirubin production (e.g. hemolysis, Wilson disease), impaired hepatic bilirubin uptake (cirrhosis, Gilbert syndrome), and impaired bilirubin conjugation (Crigler-Najjar syndrome) will lead to unconjugated (and not conjugated) hyperbilirubinemia.

A 59-year-old female presents to the clinic with a four-week history of intermittent fever, malaise, and decreased appetite. One week ago, she went to the emergency department (ED) for evaluation of a new left-sided throbbing frontal headache and pain with chewing. She brings in a report of labs from the ED visit, showing a white blood cell (WBC) count of 8400 cells/mcL, hemoglobin of 11 gm/dL, platelets of 472,000 plt/mcL, a normal urinalysis, normal complete metabolic panel, and non-contrast head computed tomography (CT) report that shows no significant findings. She reports she was sent home with a prescription for sumatriptan to treat her headache and instructions to follow up with her primary care provider. Which of the following additional history questions would be most relevant in helping you differentiate among the etiologies for fever in this patient? A. Asian ancestry B. Influenza vaccination status C. Migraines as a teenager D. Vision loss

The correct answer is D. This older female presents with symptoms suspicious for giant cell arteritis (GCA). Ocular symptoms of vascular injury such as diplopia, partial or complete vision loss, or amaurosis fugax may occur with GCA. Muscular pain and morning stiffness in the neck, shoulder, or pelvic area are symptoms of polymyalgia rheumatica which occurs in a large percentage of patients with GCA. Jaw claudication is seen in almost half of patients with temporal arteritis. Influenza does not cause a four-week fever. Migraines do not cause fever, and migraines in youth do not preclude the development of GCA in older adulthood.

A 53-year-old male comes in for his yearly checkup. He has no past medical history and takes no medications. He rushes in five minutes after his appointment time because his bus ran late, and he had to run one block to get to the office. He is sweaty and still breathing heavy. His blood pressure taken by the medical assistant is 157/85 mmHg in his right arm. What is the next best step? A. Check a basic metabolic panel (BMP) for possible hypertensive renal disease B. Prescribe ambulatory blood pressure monitoring C. Recheck his blood pressure in the other arm D. Repeat his blood pressure after he has been seated quietly for five minutes E. Start him on 5 mg of amlodipine daily

The correct answer is D. This patient had his blood pressure checked very soon after coming into the office and was obviously rushing to get to the appointment. Per consensus guidelines, blood pressure should be taken after at least five minutes of entry into the office. An appropriately sized cuff should be used with the patient's arm at the level of the right atrium. Generally, blood pressure should be checked two times, with the average blood pressure used to diagnose hypertension. While this patient's blood pressure is elevated, it should not be used at this time to diagnose him with hypertension or lead to further workup or treatment. Another option in diagnosing hypertension is ambulatory or home blood pressure monitoring, which decreases the chance of elevated blood pressure being due to "white coat hypertension" or the stress of the physician's office. Finally, if an abnormality, such as coarctation of the aorta, is suspected, it would be appropriate to recheck the blood pressure in the other arm and possibly the leg, but at this point it would be better to recheck his blood pressure after he has sat and relaxed for at least five minutes.

A 24-year-old female is evaluated in the hospital for 8/10 abdominal pain. She was admitted one day ago for cholecystitis, and an open cholecystectomy was performed today. She was previously healthy. Medications are ceftriaxone and acetaminophen. Temperature is 37.5 C (99.5 F), pulse is 100 beats/minute, respiratory rate is 28 breaths/minute, blood pressure is 120/80 mmHg, and oxygen saturation is 95% on room air. On physical exam, she appears uncomfortable and anxious. Her surgical incision from this morning is intact without erythema. She is tender near the surgical incision. Lungs are clear to auscultation bilaterally. Cardiovascular exam reveals no murmur and normal S1 and S2. What is the most appropriate step in management of this patient? A. Arterial blood gas B. Chest radiograph C. Computed tomography pulmonary angiogram (CTPA) D. Intravenous (IV) morphine E. Nebulized albuterol

The correct answer is D. This patient has new tachypnea and tachycardia after a painful surgical procedure. Pain and anxiety can cause both tachypnea and tachycardia. She is clinically stable and has not received appropriate analgesia. The next step should be to treat her pain and monitor her vital signs closely. There is no evidence to support a pulmonary process such as a pulmonary embolus or pneumonia. Further evaluation with imaging or an arterial blood gas is not indicated at this time, but could be considered if her tachypnea and tachycardia do not resolve with analgesia and resolution of her pain and anxiety. Nebulized albuterol would be indicated if she had evidence of bronchospasm but there is no wheezing on exam and no asthma history.

A 52-year-old female with stage 4 chronic kidney disease, secondary to hypertensive nephropathy, presents to establish care. Her blood pressure is 154/86 mmHg on 12.5mg of hydrochlorothiazide. Her urine protein/creatinine ratio is 1. What is the best treatment for her hypertension? A. Increase the dose of the hydrochlorothiazide and add lisinopril B. Increase the dose of the hydrochlorothiazide and add losartan C. Stop the hydrochlorothiazide and start chlorthalidone D. Stop the hydrochlorothiazide and start lisinopril E. Stop the hydrochlorothiazide and start losartan

The correct answer is D. This patient is spilling protein in her urine, so an angiotensin converting enzyme inhibitor (ACE-I), such as lisinopril, or angiotensin receptor blocker (ARB), such as losartan, is required. ACE-Is are less expensive than ARBs, so should be tried first. ARBs are appropriate for use in patients who cannot tolerate ACEIs because of side effects, such as cough, or allergic reactions, such as angioedema. Thiazide diuretics such as hydrochlorothiazide and chlorthalidone are not effective with stage 4 and stage 5 chronic kidney disease (CKD).

A 33-year-old male with a history of human immunodeficiency virus (HIV) from sexual contact presents with one day of fever, cough productive of green sputum, and a left-lower-lobe infiltrate on chest radiograph. His CD4 count is 440 cells/mm3. What is the most-likely cause of his pneumonia? A. Adenovirus B. Methicillin-resistant Staphylococcus aureus C. Mycobacterium avium complex D. Pneumocystis jiroveci E. Streptococcus pneumoniae

The correct answer is E. The respiratory tract is the most-common site of invasive pneumococcal infection in HIV-infected persons. In a prospective multicenter study, bacterial pneumonia was more common than Pneumocystis pneumonia in HIV-infected persons. S. pneumoniae was the most-common pathogen isolated. Pneumococcal disease can occur at any time during the course of HIV-1 infection. (Ref: Wallace JM et al. PubMed ID: 8256894) Pneumocystis pneumonia is more common when the CD4 count drops below 200 cells/mm3, and Mycobacterium avium complex pneumonia when CD4 is <50 cells/mm3. Methicillin-resistant Staphylococcus aureus (MRSA) and adenovirus pneumonias can occur in HIV-infected persons, but are not the most common.

A 35-year-old female comes to the clinic because of a three-month history of abdominal distension. She has concerns about diffuse abdominal pain, polyuria, and polydipsia. She has lost 15 pounds over the past six months despite not changing her diet. She was previously healthy. Physical exam is significant for a distended abdomen with shifting dullness and a fluid wave. Laboratory studies are shown: Serum calcium: 12.5 mg/dL Parathyroid hormone: 8 pg/mL 1,25(OH)2 Vitamin D: 40 pg/mL 5x3 cm mass in the left adnexal area. Which of the following is the most-likely cause of this patient's findings? A. Direct production of parathyroid hormone (PTH) by the malignant cells B. Direct synthesis of 1,25(OH)2 Vitamin D by the tumor C. Osteoclast activation and release of calcium from an osteolytic tumor D. Parathyroid hormone-related protein activation of osteoclasts E. Parathyroid hormone-related protein increasing renal calcium elimination

The correct answer is D. This patient most likely has ovarian cancer. Ovarian cancer can cause hypercalcemia three ways. First, it can secrete PTH-related-peptide (PTHrP). Second, it can cause lytic bone metastases. Third, it rarely can directly produce PTH. This patient has a low PTH level and normal 1,25(OH)2 Vitamin D. In addition, she has no bone pain, making lytic bone metastases less likely. Therefore, the most likely cause of her hypercalcemia is the tumor secreting PTHrP, which activates osteoclasts. PTHrP decreases renal calcium elimination; it doesn't increase it. The vitamin D levels are normal, so direct synthesis by the tumor is unlikely.

A 46-year-old female presents to her primary care provider to discuss her risk of cardiovascular disease. She notes that her mother had a heart attack at age 66. She denies any recent symptoms of chest pain or shortness of breath. She has a history of hypertension, hyperlipidemia, and obesity. She is a never smoker and drinks a glass of wine daily. Current medications are 10mg amlodipine daily, and 40mg atorvastatin daily. Her pulse is 78 beats/minute, respiration rate is 16 breaths/minute, and blood pressure is 154/80 mmHg. Her lungs are clear, heart sounds are normal, and there is no lower-extremity edema. Her electrocardiogram (ECG) is normal. A recent fasting lipid profile obtained 2 weeks prior to this visit shows a total cholesterol of 173mg/dL, an HDL of 45mg/dL, and an LDL of 105mg/dL. Which of the following is the most appropriate management to lower her risk of cardiovascular disease? A. Limit alcohol intake B. Limit polyunsaturated fats C. Start a beta blocker D. Start a thiazide diuretic E. Take an aspirin daily

The correct answer is D. This patient should have her blood pressure medication titrated upward to optimize her risk reduction. Thiazide diuretics are first-line antihypertensive agents and should be added to her regimen to achieve goal blood pressure value < 130/80. One serving of alcohol daily in women has been shown to lower the risk of cardiovascular disease (CVD). In addition, she should be counseled on lifestyle modification to reduce her risk of developing coronary heart disease. Polyunsaturated fats have been shown to reduce the risk of CVD and should not be limited. Beta blockers have been shown to lower mortality for secondary prevention of CVD, but not for primary prevention. A low dose aspirin daily is recommended by the United States Preventive Services Task Force (USPSTF) 2016 guidelines for individuals over 50 years old who have a 10% or greater risk of CVD, have no bleeding risk, and at least a 10-year life expectancy. Those guidelines found insufficient evidence to support daily aspirin use for primary prevention in patients < 50 years. A 2018 expert analysis by the American College of Cardiology that has incorporated more recent studies of aspirin for primary prevention recommend use of daily low dose aspirin for nondiabetic patients 40-70 years if 10-year risk of CVD is 20% of higher who do not have a high bleeding risk. This patient's 10-year risk based on ASCVD score is < 10% so daily aspirin would not be recommended.

A 56-year-old male presents to the emergency department with shortness of breath with minimal activity for two days. He endorses orthopnea and paroxysmal nocturnal dyspnea. Past medical history is significant for hypertension, hyperlipidemia, and peripheral vascular disease for which he takes aspirin, amlodipine, and atorvastatin. He has a 20-pack-per-year history of smoking. Vital signs are as follows: temperature is 37.1 C (98.7 F), pulse is 102 beats/minute, blood pressure is 180/100 mmHg, body mass index (BMI) is 36 kg/m2 and oxygen saturation is 94% on two liters of air via nasal cannula. Physical exam is significant for crackles at both lung bases, jugular venous pressure of 9 cm of water, S3 gallop at cardiac apex, and bilateral pitting edema to mid-calves. A complete blood count, basic metabolic panel, troponin T assay, and b-type natriuretic peptide level return within normal limits. An electrocardiogram (ECG) shows sinus tachycardia. His chest radiograph is shown. Which of the following is the most likely cause of this patient's symptoms? A. Acute coronary syndrome B. Acute exacerbation of asthma C. Acute exacerbation of chronic obstructive pulmonary disease D. Acute exacerbation of congestive heart failure E. Pulmonary embolism

The correct answer is D. This patient's symptoms of shortness of breath with orthopnea and paroxysmal nocturnal dyspnea, and his exam findings of bibasilar crackles, elevated jugular venous pressure, S3 gallop, and bilateral pitting edema are consistent with acute exacerbation of congestive heart failure (CHF). Chest radiograph shows cardiomegaly, alveolar edema, and haziness of vascular margins. Of note, B-type natriuretic peptide level can be falsely normal in obese patients with CHF exacerbation. Asthma exacerbation is unlikely in this patient without a prior history of asthma and absence of wheezing on physical exam. While he has a history of smoking, lack of productive cough and absence of wheezing on exam makes a diagnosis of chronic obstructive pulmonary disease (COPD) exacerbation less likely. Acute coronary syndrome (ACS) can present with shortness of breath as an anginal equivalent, but this is more common in female and older adult patients. Lack of ST-T changes on an ECG and a normal troponin T level make ACS much less likely in this patient. Patients with pulmonary embolism typically present with pleuritic chest pain. Leg swelling may be present, but it is usually unilateral, and not bilateral as in this patient.

A 68-year-old female is admitted to the hospital because of increasing dyspnea and orthopnea over the past two weeks. Her medical history is significant for ischemic cardiomyopathy with an ejection fraction of 40% and low back pain for one month. She reports no chest pain, palpitations, fever, and cough. She has been adherent to a low-sodium diet. She does not use alcohol or tobacco. Daily medications are carvedilol, lisinopril, amlodipine, atorvastatin, aspirin, and ibuprofen. Which medication would most likely cause her worsening symptoms? A. Amlodipine B. Atorvastatin C. Carvedilol D. Ibuprofen E. Lisinopril

The correct answer is D. When evaluating a patient with a congestive heart failure (CHF) exacerbation, it is important to identify the trigger. Medications which can trigger CHF exacerbation include NSAIDs (e.g. ibuprofen), non-dihydropyridine calcium channel blockers (verapamil and diltiazem), and thiazolidinediones (pioglitazone). Other common triggers include non-adherence with diet or medications (e.g. diuretics), ischemia, thyroid disorders, uncontrolled hypertension, arrhythmia, and anemia. Angiotensin-converting-enzyme (ACE) inhibitors (e.g. lisionpril) and long-acting beta blockers (e.g. carvedilol) decrease mortality in CHF. Dihydropyridine calcium channel blockers (e.g. amlodipine) can cause leg edema, but do not trigger CHF. Statins do not trigger CHF.

Mr. Sands is a 63-year-old male with a 25-year history of hypertension and type 2 diabetes mellitus who presents to the office for a follow-up of his blood pressure. Given Mr. Sands' long-standing history of poor blood pressure control, you evaluate him for end-organ effects of long-standing hypertension. Which of the following findings would be most specific for possible end-organ damage from hypertension? A. Decreased monofilament sensation on bilateral feet B. Left basilar crackles on lung exam C. Left leg edema D. Paralysis of upper and lower face on right side E. Point of maximum impulse (PMI) of the heart palpated 6 cm lateral to the mid-clavicular line

The correct answer is E. A displaced PMI could represent hypertensive heart disease causing myocardial hypertrophy and possible dilation. Other signs of end-organ damage include findings of congestive heart failure, findings of vascular abnormalities (poor perfusion or pulses), hypertensive retinopathy on retinal exam (e.g. arteriovenous (AV) nicking, flame-shaped hemorrhages), and a neurologic exam suggestive of a stroke. Decreased monofilament sensation on bilateral feet are more likely to be from diabetic neuropathy and not hypertensive vascular disease. Unilateral crackles on lung exam is less likely to be from pulmonary edema and more likely due to an acute infection. Unilateral leg edema is more specific for a deep-vein thrombosis. Hypertensive heart or renal disease generally present with bilateral lower extremity edema. Paralysis of upper and lower face on one side is consistent with Bell's Palsy. A stroke will usually spare the forehead muscles as it usually only impairs the upper motor neuron.

A 21-year-old college student presents to the hospital with one day of fever, headache, and neck stiffness. His retinal and neurological examinations are normal. Ceftriaxone is given, and a lumbar puncture is performed which shows the following: Cloudy fluid White blood cell (WBC): 2,400; 85% polymorphonuclear cells (PMNs) Red blood cell (RBC): 2 mg/dL Protein: 250 mg/dL Glucose: 32 mg/dL. Gram stain: Gram-negative diplococci Which finding in the man's cerebrospinal fluid (CSF) is most specific for bacterial infection? A. Cloudy fluid B. High CSF protein C. Low CSF glucose (hypoglycorrhachia) D. Pleocytosis with neutrophil predominance E. Positive Gram stain

The correct answer is E. All of these findings can support the diagnosis of bacterial meningitis, but the most specific finding is a positive Gram stain, with specificity approaching 100%. However, laboratory errors can occur and may include technical errors, misinterpretations when Gram staining is equivocal, and clerical errors. Thus, follow-up culture and sensitivities are always performed, even if a likely organism has been identified by a Gram stain. The other findings all support the diagnosis of bacterial meningitis, but none of these individual findings is considered specific enough to make the diagnosis. Common sense dictates that when all of these findings are present (i.e. cloudy CSF with a positive Gram stain, high CSF protein, low CSF glucose, and CSF pleocytosis with a neutrophil predominance), then the diagnosis of bacterial meningitis is confirmed for all practical purposes.

A 56-year-old female with non-ischemic cardiomyopathy and hypertension presents to the office for a routine followup. Her last hospitalization for a congestive heart failure (CHF) exacerbation was two years ago. Currently, she has no shortness of breath, orthopnea, leg edema, or chest pain. She has been following a low-salt diet and does not drink alcohol. Her medications are carvedilol and a baby aspirin. Her home blood pressure measurements have ranged from 140-150/80-90 mmHg. Her exam is notable for a blood pressure of 150/90 mmHg. Her pulse is 60 beats/minute with normal S1 and S2 with no murmurs or gallops, and she has a normal respiratory rate. Her jugular venous pulse and pressure (JVP) is normal and her lungs are clear. Her point of maximal impulse (PMI) is laterally displaced. What medication should you add that can also improve her mortality from heart failure? A. Amlodipine B. Digoxin C. Furosemide D. Hydrochlorothiazide E. Ramipril

The correct answer is E. Angiotensin-converting-enzyme (ACE) inhibitors, such as ramipril, angiotensin receptor blockers (ARBs), and long-acting cardio-selective beta-blockers improve survival in heart failure patients. In advanced cases (New York Heart Association (NYHA) class II - IV and with left-ventricular ejection fracture (LVEF) of 35% or less), aldosterone antagonists and the combination of hydralazine and nitrates improve survival as well.

You are in the rheumatology clinic seeing a 32-year-old female with a recent diagnosis of systemic lupus erythematosus (SLE). She informs you that she is G3P0A3, having had three early (<10 weeks gestation) spontaneous abortions. Which of the following antibody tests is most strongly associated with an increased incidence of spontaneous abortions? A. Anti-histidyl-tRNA synthetase (anti-Jo-1) B. Anti-Ro/anti-SSA C. Antineutrophil cytoplasmic (ANCA) D. Antinuclear (ANA) E. Antiphospholipid (APLA)

The correct answer is E. Antiphospholipid antibodies (E) are associated with an increased risk of spontaneous abortions, in part due to disruption of formation of blood supply to the developing fetus. It is estimated that 7-25% of spontaneous abortions have antiphospholipid syndrome (APS, which is a clinical syndrome of antibody positivity and clinical manifestations of thrombosis) as a primary cause, however, accurate numbers are unavailable. Pregnancy morbidity is included in the diagnostic criteria for antiphospholipid syndrome (APS). Anti-histidyl-tRNA synthetase (Jo-1) (A) antibodies are seen in dermatomyositis. There is no known increase in abortion risk with these antibodies. Anti-Ro antibodies (SSA) (B) are seen in 30-40% of women with SLE. Babies of mothers with these antibodies are at higher risk for neonatal lupus and congenital heart block (so mothers with SLE should be tested); however, there is not a known increased risk of spontaneous abortion. Antineutrophil cytoplasmic antibodies (ANCA) (C) are associated with vasculitis. Various vasculidities do pose increased risk for spontaneous abortion and should receive appropriate counseling of the risk prior to conception and the importance of disease remission before pregnancy. (Article) Antinuclear antibodies (ANA) (D) are non-specific, however, they are seen in 98% of women with SLE. Twenty percent of healthy women have a weakly positive ANA, but no clinical findings. The level of ANA titer is not related to a disease state. The presence of these antibodies alone do not pose an increased risk for spontaneous abortions.

A 55-year-old male presents with acute, severe, substernal chest pressure. He has a history of peptic ulcer disease. He drinks two beers daily and smokes a half-a-pack of cigarettes daily. He has multiple family members with inflammatory bowel disease. He works long hours outdoors in construction. Electrocardiogram (ECG) confirms an ST-elevation myocardial infarction (STEMI). Which of the following is a cardiac risk factor in this patient? A. Construction work B. Drinking two beers a day C. Having a family history of inflammatory bowel disease D. Having peptic ulcer disease E. Smoking half-a-pack of cigarettes a day

The correct answer is E. Chest pain history includes assessing for cardiac risk factors. Cardiac risk factors include smoking, hypertension, dyslipidemia, diabetes, other cerebrovascular disease, or family history of coronary artery disease (CAD).

Which of the following measures is most important when entering the hospital room of a patient who is on droplet precautions? A. Gloves B. N95 respirator mask C. Paper or plastic gown covering the entire body D. Protective eyewear E. Simple mask covering the nose and mouth

The correct answer is E. Droplet precautions are intended to protect the healthcare provider's respiratory tract from becoming contaminated with respiratory secretions. A simple mask covering the nose and mouth suffices for this purpose. Instituting droplet precautions also assumes that healthcare personnel will also follow universal precautions. Hospitalized patients often require forms of isolation that are specific to their clinical syndrome. The Centers for Disease Control (CDC) and other authorities recommend different precautions for selected infectious and various clinical syndromes. Standard precautions consist of basic hygiene techniques to limit the spread of disease and should be used with every patient. These precautions include hand hygiene between each patient contact; wearing gloves when handling soiled surfaces or touching mucosal surfaces; and shielding the eyes, nose, and mouth when spray of body fluid is expected or possible. Contact precautions are required with infections that are transmitted by fecal-oral route or can survive on fomites. These precautions include wearing a gown and gloves when in contact with the patient or any potentially contaminated areas in the patient's room. Perhaps the best known use of contact precautions is for patients with suspected or known Clostridium difficile colitis. Airborne precautions are used for infectious agents that are transmissible over long distances and for highly virulent respiratory viruses of uncertain behavior. Infectious agents requiring airborne precautions include tuberculosis, smallpox, varicella zoster virus, and coronaviruses like SARS-CoV and MERS-CoV.

A 45-year-old female comes to your clinic with concerns of fatigue and shortness of breath on exertion for the past three weeks. She has a history of hypertension and obesity, and underwent gastric bypass surgery five years ago. She stopped taking her medications three years ago due to financial reasons. On physical examination, her pulse is 89 beats/minute and her blood pressure is 100/50 mmHg. She has conjunctival pallor, cheilosis, and glossitis. Cardiac examination reveals a systolic murmur over the mitral area. Neurological examination reveals loss of proprioception. What is the most likely cause of this patient's symptoms? A. Folic acid deficiency B. Hemolytic anemia C. Hypothyroidism D. Iron deficiency anemia E. Vitamin B12 deficiency

The correct answer is E. It is not uncommon for anemic patients to present with fatigue and dyspnea on exertion. This patient's history of gastric bypass surgery suggests malabsorption of nutrients needed for hematopoiesis, along with no access to vitamin supplements for three years. Cheilosis or angular cheilitis is an inflammatory lesion at the corner of the mouth, which often occurs bilaterally. It can be associated with malnutrition or deficiencies of iron or vitamin B12. Glossitis, an inflammation or infection of the tongue, is associated with iron deficiency anemia, pernicious anemia, B-vitamin deficiencies, as well as aphthous ulcers. A functional systolic murmur in the mitral or aortic area frequently occurs in anemic patients. One of the peculiar signs of vitamin B12 deficiency is the loss of proprioception and vibratory sensation which is caused by subacute combined degeneration of the spinal cord.

A 50-year-old male presents to the office with concerns of low back pain. He has a five-year history of hypertension which is well controlled on hydrochlorothiazide. A physical exam reveals an uncomfortable obese male whose pulse is 90 beats/minute, respiratory rate is 22 breaths/minute, and blood pressure is 140/85 mmHg. The back pain is reproduced by lifting the right leg 45 degrees from the horizontal table while he is reclined in the supine position. Right-posterior calf pain which radiates down to the foot also occurs with this maneuver. Which additional physical exam finding would help confirm the diagnosis? A. Seated bilateral-leg raise reproducing the pain at 25-degrees extension B. Seated left-leg raise reproducing the pain at 25-degrees extension C. Seated left-leg raise reproducing the pain at 35-degrees extension D. Seated right-leg raise reproducing the pain at 25-degrees extension E. Seated right-leg raise reproducing the pain at 35-degrees extension

The correct answer is E. Pain in the back and posterior calf with radiation of the pain to the ankle and foot on lifting the leg between 30 to 60 degrees from the horizontal table with the patient in the supine position is indicative of radicular pathology from disk compression of the nerve leaving the spine. A seated-leg-raise test can be used as a confirmatory physical exam maneuver. A positive confirmatory test occurs when the pain is reproduced when extending the same leg at the knee within 15 degrees of the straight-leg-raise test. In this case, reproduction of the pain with the patient seated and the right leg raised to 35-degrees extension (E), within 10-degrees of the supine straight-leg-raise, would confirm a radicular cause for his pain. A seated bilateral leg-raise (A) is not an indicated maneuver for confirming unilateral radiculopathy. Likewise, (B) and (C) are not indicated tests; the crossed straight-leg-raise test is done when the patient is supine, not in a seated position. A positive seated right-leg raise should reproduce the pain within 15 degrees of the straight-leg raise result, rather than the 20 degrees stated in (D).

A 63-year-old male with stage 3 chronic kidney disease (CKD), secondary to hypertension, presents for a routine follow-up. His blood pressure is 134/72 mmHg. His physical exam is within normal limits. His labs are significant for a potassium of 5.0 mEq/L and phosphorus of 5 mg/dL. What is the explanation for his hyperkalemia and hyperphosphatemia? A. Excess dietary potassium and phosphorus B. Excess intestinal absorption of potassium and phosphorus C. Impaired intestinal absorption of potassium and phosphorus D. Impaired renal absorption of potassium and phosphorus E. Impaired renal excretion of potassium and phosphorus

The correct answer is E. Patients with CKD have a decrease in nephron mass and impaired renal potassium excretion. This can lead to hyperkalemia. Hyperphosphatemia can occur with CKD due to an inability of the kidney to excrete excess dietary phosphorus. The intestinal tract functions normally in patients with CKD. CKD patients have to limit their dietary intake of potassium and phosphorus, however, hyperkalemia and hyperphosphatemia are secondary to impaired excretion and can occur even when patients are eating a "normal" amount of these electrolytes.

A 60-year-old male presents with two months of progressive, focal pain over his mid-back. He has no neurologic symptoms. He also notes night sweats, unintended weight loss of 10 pounds, and general fatigue. His exam is notable for focal tenderness to palpation over the T10 vertebra. You order magnetic resonance imaging (MRI), which shows lesions of the T9 and T10 vertebra, concerning for metastases. Which of the following malignancies is most likely to cause focal spine pain in this patient? A. Colon B. Esophagus C. Kidney D. Liver E. Prostate

The correct answer is E. Prostate cancer is a common cancer in males, and commonly metastasizes to bone. This patient has red-flag findings for a serious cause of his back pain, including night sweats and unintended weight loss. In addition, benign pain in the thoracic spine is less common than in the lumbosacral spine and should raise suspicion for malignancy or compression fracture. The most common tumors metastasizing to the spine are prostate, lung, breast, and thyroid. Among the choices listed, prostate cancer (E) is the most common cause of spine metastasis. The type of pain experienced is usually very focal. Although colon cancer (A) is one of the most common cancers in this age group, it tends to metastasize first to the liver and lungs, and is a less common cause of spine metastasis than prostate cancer. Esophageal cancer (B), renal cancers (C), and liver cancers (D) can metastasize to bone, but they are less common compared to prostate cancer.

You admit a 65-year-old male with a severe chronic obstructive pulmonary disease (COPD) exacerbation. He has a 40-pack-per-year smoking history, and quit five years ago. He has not followed up with primary care appointments in the past, so you take the opportunity to ensure he is up-to-date with preventive measures. Which of the following preventive measures should be addressed at this time? A. Dual energy x-ray absorptiometry (DEXA) scan B. Haemophilus influenzae type b (Hib) vaccination C. Hepatitis A vaccination D. Sputum cytology E. Streptococcus pneumoniae vaccination

The correct answer is E. The U.S. Centers for Disease Control and Prevention (CDC) guidelines for adults with lung disease including chronic obstructive pulmonary disease (COPD) and asthma recommend vaccinations for Streptococcus pneumoniae. Hib and hepatitis A vaccines are not recommended in this population. In addition, an abdominal ultrasound should be performed to screen for aortic aneurysm in men aged 65 to 75 years old who have ever smoked. Screening DEXA scans are recommended for women aged 65 years old or older, but not men. Sputum cytology is used to diagnose malignancy when cancer is suspected, but is not a useful preventive measure. However, given his >30 pack-per-year smoking history, with <15 years since quitting, and age within the 55-year-old to 80-year-old range, he would be a candidate for low-dose chest CT screening for lung cancer.

A 33-year-old previously healthy female is admitted to the hospital with pneumonia. Two weeks prior to admission, she developed fever and myalgias. She was diagnosed with influenza at an urgent care clinic. Five days ago, she developed a productive cough, fever, and chills. Today, her temperature is 38.9 C (102 F), pulse is 110 beats/minute, respiration rate is 22 breaths/minute, blood pressure is 90/68 mmHg, and pulse oximetry is 89% on room air. Respiratory exam reveals crackles at the lung bases bilaterally. White blood cell count is 3,400/mm3. The chest x-ray shows multilobular cavitating alveolar infiltrates. Which of the following organisms is the most likely cause of pneumonia in this patient? A. Adenovirus B. Legionella pneumophila C. Mycoplasma pneumoniae D. Pseudomonas aeruginosa E. Staphylococcus aureus

The correct answer is E. The most likely etiology would be Staphylococcus aureus, which can present in young, previously healthy patients as severe pneumonia after an influenza-like illness, and is characterized by severe respiratory symptoms, high fever, leukopenia, and hypotension, as in this patient. The chest x-ray often shows multilobular cavitating alveolar infiltrates. Adenovirus pneumonia would be less likely, as it is more commonly seen in infants and immunosuppressed patients. The patient does not have any risk factors for Legionella pneumonia such as diabetes mellitus, smoking, chronic obstructive pulmonary disease (COPD), or immunosuppression. Mycoplasma pneumonia usually presents with a more gradual, subacute course. Pseudomonas aeruginosa is an uncommon cause of pneumonia and is most often seen in immunosuppressed patients, ventilated patients, or patients with chronic lung disease.

A 34-year-old female presents to the hospital for one day of periumbilical abdominal pain that is 8/10 in severity. It radiates to her back and is associated with severe nausea and vomiting. She does not drink alcohol. She has had three Caesarean sections, gastric bypass surgery for weight loss, and a tubal ligation. On exam, her vitals show temperature is 37 C (98.6 F), pulse is 100 beats/minute, respiratory rate is 18 breaths/minute, and blood pressure is 130/70 mmHg. She has decreased bowel sounds on auscultation. On palpation, she has periumbilical tenderness to palpation and voluntary guarding. Her complete blood count (CBC) and comprehensive metabolic panel are normal. Her lipase is 520 U/L, and her urine pregnancy test is negative. She is given a liter of intravenous (IV) normal saline and has normal saline running at 250 cc/hour. Her pain is controlled with morphine. What is the next step in her evaluation? A. Computed tomography (CT) of the abdomen and pelvis B. Endoscopic retrograde cholangiopancreatography (ERCP) C. Hepatobiliary (HIDA) scan D. Magnetic resonance cholangiopancreatography (MRCP) E. Right-upper-quadrant ultrasound (RUQ US)

The correct answer is E. The patient has pancreatitis. A RUQ US is indicated to determine if she has gallstone disease. RUQ US can also be useful in determining if she has cholecystitis, though her current symptoms are not consistent with cholecystitis. A CT scan could be considered if the diagnosis of pancreatitis is uncertain, or if complications of pancreatitis such as pseudocyst, necrotizing pancreatitis, or infected necrosis are suspected. These complications of pancreatitis take time to develop, so it is too early in the clinical course to see them. ERCP is indicated in moderate-to-severe acute biliary pancreatitis if there is cholangitis, persistent biliary obstruction, clinical deterioration, or a stone detected in the biliary duct. She does not meet these criteria. MRCP is non-invasive, does not require contrast, and can be used to determine if there are stones in the common duct. MRCP can be used to determine if there is a need for ERCP in patients with persistently elevated liver tests or dilation of the common bile duct without cholangitis, if the patient is pregnant, or if there is altered anatomy that would make ERCP difficult. A HIDA scan can be used to diagnose acute cholecystitis if the diagnosis is uncertain after a right upper quadrant ultrasound is performed. In this case, the patient's symptoms are consistent with pancreatitis so a HIDA scan would not be ordered.

A 73-year-old male with a history of a recent diverticular bleed causing hemodynamic instability presents with right knee pain. He states that he was diagnosed with osteoarthritis of the knee in the past. He has tried acetaminophen, without improvement. He asks what other options are available to help control his pain and improve his mobility. What is the best initial option to treat this patient? A. Glucosamine and chondroitin B. Ibuprofen C. Naproxen D. Opioids E. Physical therapy

The correct answer is E. The patient should undergo a trial of physical therapy for pain control and mobility training. Glucosamine and chondroitin have shown no benefit in recent high-quality studies Nonsteroidal anti-inflammatory drugs (NSAIDS), such as naproxen and ibuprofen, inhibit platelet function and should be avoided due to his recent history of gastrointestinal (GI) bleeding. Opioids are not recommended as initial options for pain control in non-cancer patients.

In mid-September, a 52-year-old male presents with a three-day history of sore throat, nasal congestion, rhinorrhea, dry cough, and fatigue. He has no chronic medical conditions and does not smoke. On physical examination, vital signs are normal. There is mild, clear nasal discharge with no sinus tenderness. The oropharynx is without injection or exudate. There is no lymphadenopathy. Tympanic membranes are dull bilaterally and a small effusion is noted on the left. Lungs are clear to auscultation. What is the most likely pathogen causing his symptoms? A. Adenovirus B. Coronavirus C. Parainfluenza virus D. Respiratory syncytial virus E. Rhinovirus

The correct answer is E. The patient's findings are consistent with a viral upper respiratory infection (URI). Rhinovirus is the most common cause of viral URI, causing 30-50% of all cases annually. Rhinovirus has seasonal peaks in the spring and early autumn and is also the predominant organism causing URI during the summer months. During the winter, the influenzas, respiratory syncytial viruses, and parainfluenza viruses predominate. Adenoviruses and coronaviruses are less prevalent causes of URI year-round.

A 52-year-old female presents with one month of progressive dyspnea on exertion, a productive cough of clear sputum, bilateral leg swelling, and orthopnea. She has had no chest pain or fevers. She has a history of poorly controlled hypertension and 30-pack-years of smoking. Before beginning her physical exam, you determine that her pre-exam probability of heart failure is moderate. On exam, you find a jugular venous pulse and pressure (JVP) of 12 cm with positive hepatojugular reflux, regular heart rate, normal S1 and S2 with S3 gallop, 3/6 systolic crescendo-decrescendo murmur at the right upper sternal border, crackles at both lung bases. Which of the following physical examination findings most increases your post-exam probability of congestive heart failure? A. Bibasilar crackles B. Hepatojugular reflux C. Increased jugular venous pressure D. Systolic murmur in the aortic area E. Third heart sound

The correct answer is E. The physical exam can be considered a diagnostic test. Like other diagnostic tests, different physical exam findings have different sensitivities and specificities. Likelihood ratios incorporate both the sensitivity and specificity to tell you how strongly a test result changes the post-test (or post-exam) probability of a disease. A third heart sound (S3) has the highest positive likelihood ratio for congestive heart failure, followed by an elevated JVP and hepatojugular reflux. Bibasilar crackles and dependent edema have very low likelihood ratios. They are non-specific findings that commonly occur in many conditions other than heart failure; for example, crackles can be heard in chronic obstructive pulmonary disease (COPD), interstitial fibrosis, atelectasis, and pneumonia. A systolic murmur is not useful in diagnosing heart failure, though it can sometimes be helpful in considering the underlying cause (e.g. critical aortic stenosis).

It is the Monday after a major national holiday, and you are seeing a 58-year-old male in the emergency department with progressive shortness of breath and generalized weakness over the previous two days. He has a history of end-stage kidney disease (ESKD) due to diabetes and hypertension. He is chronically anuric and undergoes hemodialysis (HD) every Tuesday, Thursday, and Saturday. He says, "You know, Doc, I definitely indulged a bit too much at the holiday dinner." On exam, he appears short of breath, sitting upright on the gurney and speaking in 2- to 3-word sentences. His temperature is 36.1 C (97 F), pulse is 75 beats/minute, respiratory rate is 20 breaths/minute, blood pressure is 180/60 mmHg, and oxygen saturation is 94% on 10 liters/minute face mask. His exam is notable for diffuse crackles bilaterally, jugular venous pulse and pressure (JVP) of 12 cm while upright, lower extremity pitting edema to above his knees bilaterally, and a left arteriovenous (AV) fistula with a strong thrill. His labs are notable for the following: Sodium (Na): 132 mEq/L Potassium (K): 5.5 mEq/L Chloride (Cl): 111 mEq/L Bicarbonate (HCO3): 19 mEq/L Blood urea nitrogen (BUN): 48 mg/dL Creatinine (Cr): 5.9 mg/dL Phosphorus: 5.1 mg/dL Albumin: 3.4 g/dL Troponin: 0.04 ng/mL pH: 7.35 Chest x-ray shows pulmonary vascular congestion and pulmonary edema. Electrocardiogram (ECG) is normal. What is the next-best step in evaluation and/or management of this patient? A. Blood cultures and intravenous (IV) antibiotics B. Calcium gluconate IV C. Computed tomography angiography (CTA) of the chest D. Furosemide 40 mg IV push and reevaluation in four hours E. Nephrology consult for urgent hemodialysis

The correct answer is E. This patient has ESKD and volume overload with pulmonary edema due to dietary indiscretion, which is an indication for urgent hemodialysis. He has ESKD rather than AKI but the indications for urgent dialysis apply to both ESKD and AKI patients. A helpful mnemonic for remembering indications for urgent hemodialysis is "AEIOU": Acidosis - milder acidosis can sometimes be managed with bicarbonate replacement, but more severe or refractory acidosis requires hemodialysis for correction. Electrolytes - particularly hyperkalemia refractory to conservative measures. Ingestion - of toxins or medications that can be removed with hemodialysis. This includes toxic alcohols, salicylates, phenobarbital, and lithium. Overload - volume overload that is either refractory or cannot be managed with conservative measures. This patient is significantly volume overloaded, manifested by respiratory distress from acute hypoxia, crackles on lung exam, elevated JVP, and leg edema. Uremia - based on clinical assessment. Findings of symptomatic uremia include altered mental status, nausea, generalized pruritus, uremic pericardial friction rub, etc. Calcium gluconate can be given acutely in hyperkalemia to stabilize the cardiac membrane, but does not lower the total body potassium level and is therefore a temporizing measure. This patient's potassium was within the upper limit of normal with a normal ECG, and his hypoxia should be addressed first. The history, exam, and imaging are all supportive of pulmonary edema from volume overload; this is much more likely than a pulmonary embolism (PE), and therefore delaying treatment for a CTA would not be appropriate. In addition, the IV contrast dye required for a CTA would give him additional intravascular volume, exacerbating his volume overload. The patient is anuric (does not make urine), so treatment with furosemide and re-evaluating four hours later is not correct and would result in a delay of more-appropriate treatment. There is no evidence of an active infection currently that would prompt the need for blood cultures and antibiotics.

A 43-year-old healthy female comes to the clinic to discuss a cancer screening. Her family history is significant for a first cousin with colon cancer diagnosed at 48 years of age. There is no history of cancer in her immediate family. She does not smoke or consume alcohol, and she exercises three times a week. Vital signs show her pulse is 68 beats/minute, blood pressure is 132/72 mmHg, and body mass index (BMI) is 23.8 kg/m2. Physical examination is normal. With regards to colon cancer screening, what would you recommend for this patient? A. Computed tomography (CT) colonography now and then every 5 years B. Fecal immunochemical testing now and then every year C. Fecal occult blood testing at age 50 and then every 3 years D. Flexible sigmoidoscopy now and then every five years E. Routine colonoscopy at age 50 and then every 10 years

The correct answer is E. This patient is at average risk for colon cancer. Her cousin is not considered a first-degree relative, so she does not fall into the category of high risk. The only recommended screening test schedule among the choices is beginning colorectal cancer screening at age 50, and if using colonoscopy, the screening interval is every 10 years if the colonoscopy is normal. Flexible sigmoidoscopy, fecal immunochemical testing, and CT colonography are all options, but wouldn't be indicated until age 50. If using fecal occult blood testing, it should be repeated annually.

A 45-year-old female presents with a 12-day history of fever to 39.2 C (102.6 F), unilateral face and tooth pain, a frontal headache, and purulent nasal discharge. She has no chronic medical conditions. On examination, temperature is 38.4 C (101.1 F), pulse is 96 beats/minute, respiratory rate is 16 breaths/minute, and blood pressure is 116/74 mmHg. There is tenderness to palpation over the left maxillary sinus. Nasal examination shows inflamed turbinates bilaterally with purulent discharge on the left. The remainder of the exam is normal. What is the most-likely pathogen causing her symptoms? A. Adenovirus B. Moraxella catarrhalis C. Parainfluenza virus D. Staphylococcus aureus E. Streptococcus pneumoniae

The correct answer is E. This patient most likely has acute bacterial sinusitis as evidenced by her high fever and unilateral face pain lasting more than 7 to 10 days, Streptococcus pneumoniae and Haemophilus influenzae are the most common causes of acute bacterial sinusitis and account for >50% of infections. Adenovirus infection presents with severe sore throat and cough because it injures the respiratory epithelium. Moraxella is less common in adults, but causes approximately 25% of acute sinus infections in pediatric patients. Parainfluenza infection causes more tracheal and laryngeal inflammation and edema resulting, in dysphonia or croupy cough. Staphylococcus aureus (Staph aureus) can colonize the anterior nares and is associated with chronic sinusitis, but infrequently causes acute sinusitis.

A 49-year-old female returns to the emergency department (ED) with a five-week history of fever, night sweats, decreased appetite, weight loss, and dry cough. She has a history of human immunodeficiency virus (HIV) and hyperlipidemia and has been off medications for more than one year since she was released from prison. Two weeks ago, she went to the ED for these symptoms. Labs from the ED visit revealed a white blood cell (WBC) count of 10,600 cells/mm3, a CD4 count of 90 cells/mm3, hemoglobin of 11,100cells/mm3, platelets of 492,000 plt/mm3, a normal urinalysis (UA), normal complete metabolic panel, negative blood cultures, and a chest x-ray with moderate pleural effusions with mediastinal lymphadenopathy. Hospital admission was recommended, but she left against medical advice. Which of the following is the next best step in evaluating this patient? A. Biopsy the mediastinal lymph nodes B. Obtain blood cultures C. Send pleural fluid for analysis and culture D. Send serum for an interferon gamma-release assay (IGRA) E. Send sputum sample for an acid-fast bacillus (AFB) smear

The correct answer is E. This patient with HIV has recently been incarcerated and thus is at risk for mycobacterium tuberculosis (MTB). Mycobacterium avium complex would be a consideration if her CD4 were less than 50. The patient has had a previous evaluation in the ED and blood cultures were negative at that time and she had known pleural effusion and mediastinal lymphadenopathy. As her TB status is unknown, sputum for AFB and culture and a serum assay for interferon gamma are indicated. The results will not be immediately available but this is the easiest and least invasive way to make the diagnosis of TB. Patients with an undiagnosed pleural effusion warrant investigation. She should undergo thoracentesis and the fluid should be sent for the usual tests to determine if it is transudative or exudative, as well as bacterial cultures. As the diagnosis of MTB is a possibility, adenosine deaminase of the pleural fluid may be helpful. However, AFB smears and mycobacterial cultures of the pleural fluid are often negative for MTB and diagnosis of a tuberculous pleural effusion requires making the diagnosis of TB in another way (sputum or other sources) or by tissue culture (obtained by pleural biopsy). Biopsy of the mediastinal lymph nodes is very invasive and should be done only after other less-invasive attempts to make a diagnosis have been unsuccessful.

A 63-year-old male comes to you for a new-patient visit. He has a past medical history of a myocardial infarction (MI) and hyperlipidemia one year ago. He ran out of his previous medications several weeks ago and cannot remember what they were. He is asymptomatic and has a normal physical exam except for a blood pressure of 165/92 mmHg. His labs come back with a hemoglobin A1c of 5.4%, an elevated LDL, and normal electrolytes and renal function. In addition to a statin and daily aspirin, what other medication would be the best to start him on? A. Amlodipine B. Diltiazem C. Hydrochlorothiazide D. Lisinopril E. Metoprolol

The correct answer is E. This patient with a history of coronary artery disease (CAD) and hypertension should be treated with a beta-blocker as it is the preferred agent for patients with a history of myocardial infarction (MI). Beta-blockers have been shown to decrease cardiovascular mortality in patients with CAD. Lisinopril (an angiotensin-converting-enzyme (ACE) inhibitor), hydrochlorothiazide​ (a thiazide diuretic), and amlodipine (a dihydropyridine calcium-channel blocker) are typically good first-line agents for hypertension, but in patients with CAD, it would be best to start him on a beta-blocker first for the mortality benefit. Diltiazem is a non-dihydropyridine calcium channel blockers, and while it can be used for hypertension it is more commonly used for rate control in patients with atrial fibrillation. Again, it does not have a compelling indication in this man.

A 38-year-old female with human immunodeficiency virus (HIV) infection presents to her primary care doctor with concern for daily intermittent fevers to 38.6 C (101.5 F) for four weeks. She also has night sweats and some dyspnea, especially when hiking near her home in Louisville, Kentucky. She doesn't have any additional symptoms. She reports no recent dental work or travel. She has no pets. Her physical exam is unremarkable. Laboratory testing reveals a CD4 count of 220 cells/mcl, normal basic metabolic panel, and a complete blood count (CBC) remarkable for stable mild thrombocytopenia (125,000 mm3). Bacterial blood cultures demonstrate no growth after five days. Chest x-ray shows diffuse pulmonary infiltrates. Which of the following would be the most appropriate test to order for this patient? A. Echocardiogram B. Funduscopic examination C. Lymph node biopsy D. Serum cryptococcal antigen E. Serum histoplasma antigen

The correct answer is E. This patient's fever and pulmonary infiltrates are due to histoplasmosis. Her CD4 count is above 100 cells, and her physical exam is unremarkable. She lives in the Ohio River Valley, where histoplasmosis is endemic. She has no risk factors or physical findings consistent with endocarditis, so doing an echocardiogram would not be appropriate at this time. Performing a lymph node biopsy without presence of lymphadenopathy, or before any imaging or less invasive testing, would not be appropriate at this point in the evaluation of fever. With a CD4 count > 100, she is less likely to have cryptococcus, particularly with normal mentation and lack of a headache. Cytomegalovirus (CMV) will often manifest with CMV retinitis, but with a CD4 count > 100, it is less likely.

A 45-year-old male with a history of alcohol use disorder is admitted to the hospital with delirium. His brother, with whom he lives, states that the patient normally drinks a 12-pack of beer per day, but for the past two weeks has been trying to quit by drinking a large glass of water every time he craved a beer. Vital signs are normal. On exam, his mucous membranes are moist, lungs are clear, and jugular venous pulse and pressure (JVP) is normal. He is alert and oriented to self and place, but not to time. He can follow one-step commands but is unable to name the days of the week forward or backward. His serum sodium is 120 mEq/L. His urine sodium is 5 mEq/L, and urine osmolarity is low. Which of the following is the best approach to treat his hyponatremia? A. Dextrose in water (D5W) at 100 cc/hour B. Hypertonic (3%) saline intravenous (IV) at 100 cc/hour C. Hypotonic (0.45%) saline IV at 100 cc/hour D. Normal (0.9%) saline IV at 100 cc/hour E. No IV fluids

The correct answer is E. This patient's hyponatremia is due to beer potomania with polydipsia; he has low sodium stores due to excessive beer intake, and he is diluting those stores with extra water intake. Because of the excess water intake, his urine is diluted (low sodium and osmolarity). Stopping the excess fluid intake may be enough to restore his serum sodium to normal. If this were not sufficient, the next step would be to administer 0.9% saline. Providing D5W or 0.45% saline will further dilute his serum sodium and worsen his hyponatremia. Providing hypotonic solutions could become necessary if he begins to rapidly correct on his own with fluid restriction. If his serum sodium levels decreased with fluid restriction to severe hyponatremia, or he developed severe neurologic symptoms such as seizures, then treatment with hypertonic saline infusion in an intensive care setting with frequent lab monitoring would be indicated.

A 75-year-old male presents to the clinic for the follow-up of a skin biopsy of a nevus. The pathology report says that histology is consistent with a junctional nevus. Which of the following would be the expected histologic findings in this patient? A. Atypical melanocytes B. Hemosiderin deposition C. Melanocytes primarily within the dermis D. Melanocytes located in both the dermis and dermal-epidermal junction E. Nests of melanocytes at the dermal-epidermal junction

The correct answer is E. This patient's pathology report has determined that he has a benign nevus. A junctional nevus would histologically show nests of melanocytes at the dermal-epidermal junction (DEJ). A dermal nevus would have melanocytes primarily within the dermis, while a compound nevus would have a combination of melanocytes located within the DEJ and dermis. Atypical melanocytes would be found in patients with melanoma. Hemosiderin deposition may be noted in postinflammatory hyperpigmentation.

A 66-year-old female is evaluated in the emergency department with a two-day history of right-sided pleuritic chest pain and intermittent shortness of breath. Past medical history is significant for hypertension, chronic kidney disease, and depression. Medications include lisinopril, baby aspirin, and oral hormone replacement therapy. Physical examination shows her temperature is 37.8 C (100.1 F), pulse is 112 beats/minute, respiratory rate is 22 breaths/minute, blood pressure is 158/76 mmHg, body mass index (BMI) is 32 kg/m2 and oxygen saturation is 89% on room air. Her lungs are clear and heart sounds are normal. Her left leg is edematous and tender to palpation. An electrocardiogram (ECG) shows no ST or T wave changes. Chest x-ray is normal. Laboratory studies are unremarkable except for a creatinine of 2.1 mg/dL. Which of the following is the most appropriate diagnostic test at this time? A. Computed tomography (CT) of the chest B. D-dimer test C. Troponin T test D. Venous ultrasonography of left leg E. Ventilation perfusion (V/Q) scan

The correct answer is E. This patient's symptoms of acute pleuritic chest pain and dyspnea, and her physical exam findings of tachycardia, low-grade fever, hypoxia, and left leg swelling are all consistent with the diagnosis of pulmonary embolism (PE). Obesity and hormone replacement therapy are additional risk factors for PE. A ventilation-perfusion scan can confirm the diagnosis of PE by showing an area of the lung that is normally ventilated but poorly perfused. Chest CT is an excellent test for diagnosis of PE. However, intravenous contrast administered during the test can worsen renal function and would be contraindicated in this patient with a creatinine of 2.1 mg/dL. D-dimer is a sensitive, but non-specific, test for PE. It is helpful in ruling out PE in a patient with low pretest probability of PE. This patient has a high pretest probability for PE with a simplified Wells score of 3 (1 point each for tachycardia; signs and symptoms of deep vein thrombosis; and alternative diagnosis being less likely than PE). A definitive diagnostic test such as a CT scan or a V/Q lung scan is recommended as the next-best step in patients with a high pretest probability of a PE (simplified Wells score >1). A troponin T level is helpful in detection of myocardial injury. If the patient has a PE on V/Q, then a troponin T would be helpful in determining if it is a submassive PE, which might benefit from thrombolytics. However, this patient is unlikely to be suffering from an acute coronary syndrome, based on the duration of symptoms, pleuritic nature of her chest pain, and lack of ECG changes, so it is not necessary as a first test to diagnose PE. Venous ultrasonography of the left leg could confirm the diagnosis of deep-vein thrombosis and could be considered if results of the V/Q scan return indeterminate or if V/Q scan is not available. However, a V/Q scan is a better test to make a diagnosis of PE in this patient with chest pain and shortness of breath.

Ms. Azarola is a 46-year-old female with a past surgical history of an exploratory laparotomy for endometriosis, two Caesarean sections, and a tubal ligation. She presents to the emergency department with eight hours of intermittent, diffuse, severe abdominal pain with nausea and vomiting. On physical exam, her vital signs show temperature is 37.2 C (99 F), pulse is 110 beats/minute, respiratory rate is 18 breaths/minute, and blood pressure is 148/90 mmHg. Her abdomen reveals high-pitched bowel sounds and moderate distension. There is a hollow sound when you tap on her abdomen. On palpation, she has guarding, but no rigidity. An abdominal x-ray reveals air fluid levels and distension in the small bowel with minimal gas in the large intestine. You place orders for routine labs, non per os (NPO), intravenous (IV) fluids, and insert a nasogastric tube, placing it to suction. What should your next action be? A. Barium enema B. Follow-up abdominal plain film C. Nutrition consultation D. Right-upper-quadrant ultrasound E. Surgical consultation

The correct answer is E. You need to request a surgical consultation. This patient has an acute small-bowel obstruction. While this can often be managed conservatively, it is important that a surgical consultation is obtained in case the obstruction does not resolve with conservative treatment. A barium enema should not be performed in small-bowel obstruction. A follow-up abdominal film can be a part of the follow-up plan for this patient, but only after a surgical consultation is requested. If she remains NPO for several days, then nutrition consultation may become appropriate, but is not needed acutely. A right-upper-quadrant ultrasound is the test of choice for presumed cholecystitis, but this patient does not fit that illness script.

A 35-year-old female presents with two days of constant, sharp, left-sided chest pain. There was no clear precipitant for her pain, and she does not identify any alleviating or aggravating factors. On physical exam, she appears comfortable and has normal vital signs. Cardiac exam demonstrates regular rate and rhythm, normal S1 and S2, and no murmurs. She has point tenderness over the left-upper-sternal border. Electrocardiogram (ECG) showed normal sinus rhythm with no ST or T wave changes. What is the most likely diagnosis? A. Acute coronary syndrome B. Costochondritis C. Panic disorder D. Pneumothorax E. Pulmonary embolism

There is no single best answer. Chest wall tenderness increases the likelihood of the etiology of a musculoskeletal etiology, such as trauma, rib fracture, or costochondritis. Although acute coronary syndrome (ACS) can occasionally cause reproducible chest tenderness, the long duration of constant chest pain without any findings of infarction or ischemia on ECG makes ACS very unlikely. A negative troponin would rule out ACS completely. Chest pain can be associated with anxiety or panic disorder, but the patient is not presenting with panic symptoms, and other life-threatening etiologies should be ruled out first. A pneumothorax significant enough to cause pain for two days is typically associated with tachycardia and respiratory symptoms, including tachypnea, hypoxia, or both. Chest pain due to pulmonary embolism can cause reproducible tenderness with palpation, but it is typically worsened with deep breathing, and is associated with tachycardia.


Related study sets

Chapter 19---The menstrual cycle

View Set

Unit 6: Palliative Online Courses

View Set

Chapter 19, Documenting, Reporting, Conferring

View Set

Adult cognitive disorders Neuropsych assessment post brain injury

View Set